Location via proxy:   [ UP ]  
[Report a bug]   [Manage cookies]                

مذكرة مايوSMLE-2019

Download as pdf or txt
Download as pdf or txt
You are on page 1of 514

‫‪-SMLE-2019‬مذكرة مايو‬

‫مالحظات‬ ‫‪2019/5/27 11:10 am‬‬ ‫أخر تحديث للملف‬

‫تم التصحيح الى صفحة ‪291‬‬ ‫قروب أبريل ‪1‬‬

‫تم التصحيح الى صفحة ‪312‬‬ ‫قروب أبريل ‪2‬‬

‫تم إنشاء وتصحيح المذكرة بمشاركة أعضاء قروب أبريل‬

‫مالحظة‪ :‬المذكرة تحت التحديث اليومي‪ ،‬لرؤية أخر التحديثات والتعديالت بشكل حصري‬
‫الرجاء االنضمام الى قروبات أبريل عن طريق ليدر القروب الدكتور محمد والمشاركة‬
‫بالتصحيح‪ ،‬حيث أنه لن تنشر المذكرة لباقي القروبات إال بعد االنتهاء منها وإعادة تصحيحها‬
‫لمنع التشتت بكثرة الملفات‪.‬‬

‫التظليل الخاص بالتصحيح هو اللون األخضر‬


Infant abandoned by his mother after delivery, pediatric surgeon reviewed the
case and it was hernia, he recommended to do surgery now, what should you do?
- refuse to do surgery without consent
- consult ethics committee in hospital
- do surgery without consent
- fourth option can not remember it

Child is treated for eczema with topical steroid, comes to clinic with itching and
pustule lesions on top of his eczema, arranged in grape like pattern,
What is the most likely organism that causes his superimposed infection :
Herpes simplex
Staph
Group a strept

Woman admitted with DVT diagnosis and was started on enoxaparin


The same night she develops SOB, ct confirms lower pulmonary artery thrombus,
what is the appropriate action ?
Change enoxaparin to sodium heparin
Start thrombolytic therapy
Child come with fever and headache, there is menengeal irrtation sings on
examination, Lp done with results :
Glucose 2.9 Normal 2.4-4.1
Protein 560 normal range 150-450

What ttt is best :


Ampicillin + vancomycin
Vancomycin + rifampin
Ampicillin + something
Vancomycin + ceftriaxone
The most common presentation in compartment syndrome is severe pain, what
mechanism is responsible for this ?
- skin hypoxia
- nerve hypoxia
- bone hypoxia
- muscles engorgement
78 years old male admitted to ICU for MI that was complicated by pneumo nia in
hospital, he received etazocine for 12 days, now complains of right upper
quadrant pain with tenderness, us was done with report of thick gallbladder wall
with fluid collection and no stones
What is the management?
-Emergency cholecystectomy
- ERCP drainage
- us guided drainage
- continue same medication

Pt DM, HTN with chronic heart failure, comes to clinic for medication counseling,
he takes insulin l, furosemide, enalapril, digoxin
Some tests values where provided
K = 5.9
Glucose high
Bp normal
What drug should he stop? enalapril
New test was introduced to 800 of patients who was proven the have liver
cirrhosis
And other 800 who didn't have liver disease
200 of first group tested positive and 100 from 2nd group tested positive
What is the sensitivity of this test ?
25%
33%
57%

Woman come to clinic with history of multiple abortions, now she is 11 week
pregnant, what is the best predictor of complication in pregnancy ?
Abd pain
Vaginal bleeding
Other options

Old man recently diagnosed with Parkinson and he wants to begin treatment,
what is the best drug to give?
Levodopa
Surgeon was doing lab cholecystectomy and faced difficult anatomy there was an
injury and did x- ray showed air in abdominal cavity and thorax, what organ was
damaged?
Esophagus
Stomach
Pancreas

Child come with cola urine , pt. has hx of tonsillitis, PSGN is suspected, what test
would help you confirm your diagnosis ?
- blood culture
- anti-streptomyces
- c3 low

Women pregnant 34+2 weeks had cesarean section past pregnancy due to non-
ushering fetal monitoring, present now for external cephalic version , us amniotic
fluid index 14, responsive ctg
What would be absolute contraindication for ecv ?
Previous cs
Ctg results
Us findings,
fourth option cant remember
Child come to clinic with cough fever(38.6)for3dayswithrash started on face then spread to trunk and
white pustule on erythematous base on the buccal

mucosa(oppositethelower1st&2ndmolars),dx?

-measles

-rubella

-scarlet fever

Other option can't remember

Case about some labs with high calcium and normal PTH(1.1)phosphate was not given, renal functions
were elevated slightly and asking about what is the

diagnosis ?

-renal failure

-2ryhyperparathyroidism

-primary hyperparathyroidism

-tertiary hyperparathyroidism
30 y old women comes with painless vaginal mass under urethral orfice that
bleeds when touched , picture was provided
What is dx
Bartholin abscess (wrong option )
Other options can't remember

Pt come to clinic with leg painless swelling , no hx of trauma, on examination leg


is pale cold and blueish discoloration is noted, peripheral pulses are intact, what is
the appropriate method to confirm diagnosis ?
- ct angio
- MRI
- arterial duplex
- Venus duplex

Case about teenager had argument with her mother and swallowed 20 pills of
acetaminophen come to emergency the second day having Abd. Pain and nausea
vomiting, asking about stage ?
Stage 1
Stage 2
Stage 3
Stage 4
10 year old boy come with hx of diarrhea more than one month, there is
abdominal pain with blood sometimes and mucous sometimes, the child hasn't
been loosing weight, the child is reporting that he feels incomplete evacuation
even when there is nothing left in his bowel,
What would be appropriate to try first ?
- metronidazole
- gluten free diet
- ct
- some other option

Same case but there was bleeding and repaired


After surgery what would you do ?
Tell the patient what happened
Tell the patient lawyer to prepare for lawsuit
Tell hospital administrators to take action against you
Don't tell any body
Case about child with lower limb movement loss after URTI, child describes his
problem started from down and ascending upward, there was something about
his sensation in lower limb but as I remember it wasn't lost completely, asking
about diagnosis?
- GBS
- Transverse myelitis
- other options weren't related to neuro

Case about hip joint avascular necrosis, what is management ?


I chose surgical repair

Case about septic arthritis and asking about diagnosis ?


I chose fluid aspiration

Case about thyroid nodule , FNA was done and result were inadequate, asking
what to do?
Thyroid iodine scan
Repeat FNA
Thyroidectomy
Woman lactating come to clinic complaining of right breast mass, firm with
erythema and tenderness, no discharge asking about diagnosis ?
- abscess
- fibroadenoma
- duct ectasia

Child 5 years old diagnosed with tonsillitis, and asking about his brother (2 years)
prophylaxis ?
I chose penicillin
Other cant remember

Case of pregnant women with UTI, culture was done and result was E.coli
sensitive for TMP/Smx and ciprofloxacin and nitrofurantoin, and asking what is
the best drug to give ?
Options were the same
- cipro
- tmp/smx
- nitrofurantoin
1- Couple come by infertility after investigation done show bilateral fallopian tube
obstruction what will do
Tell wife
Tell husband
Tell couple

2- child with eczematous lesion and mother do good hygiene by warm bath what
will do next:
That enough for him
Olive oil may decrease his symptoms
Add strong soap .......

3- pt c/o burn 25% her weight 70 how to manage


Normal saline 2l/k now and 2L after 6 hr
Ringer lactate 5 L / k after 6 hr and 5 L after 16 hr
Normal slain 5L after 6 hr and 5L after 8 hr
Ringer lactate 2.5 now and 2.5 after 8 hr

4-CTG early decelerations causes


Head compression
Placenta insufficient
Cord prolapse
5- child activity in day:
60 min
90
100
120 min

6- LMC 17may 2019 what is the expected day


24 February2020

7- celiac disease what’s the late prognosis?

8- GBS the expected prognosis:


Permanent paralysis
Full recovery

9- baby born for hb +antigen mother


and received vaccine and antigen and mother worry about breastfeeding for him
because she have history of baby death after breastfeeding what will do :
Wait 12 hr and then breastfeeding
Don’t lactated until do antigen test
10 child 4 months c/o hypospadias in midshaft come by here mother for
circumspection what will do :
Wait at 6 months an do it
No relationship between circumcision and hypospadias
Wait after done from surgery
This type of hypospadias not need surgery so do circumcision

11
‫صورةالطفل جاي باحمرار بدون حكة و طالبين التشخيص‬
Atopic Dermatiti
Erythema .......
candidiasis

12- scenario of hemolytic uremic syndrome

13- scenario of B12 deficiency anemia


14 - first line treatment of malaria in KSA
Quinine
Sulfa dioxin
Chloroquine

female G6 the last delivery before 3 mouths C/o stool incontinence and flatus per
her vaginal what is the diagnosis
Partial pre Neal tear
Anus vaginal fistula

Abruptio placenta what is the acid base balance of fetus??


Asphyxia
Res acidosis
Meta acidosis

Surgeon doing restriction for cancer mass , anastomosis and he call the vascular
surgeon for bleeding control , the case under control , whole minute the blood
pressure fall down , what you do ?
1- Stabilize the pt on table
2- Re opening and suture the injury
3- Clap above the injury on aorta
4- Clap down the injury on aorta
2.Man in ER with pain in the right upper abdomen with vomiting ultrasound
shows thick wall
of gallbladder and fluid accumulation around management
A - lap cholecystectomy after 12 weeks
B - percutaneous drain
C - open cholecystectomy

What the contraindication of liver transplantation ?


1- Acute liver cirrhosis
2- Liver cirrhosis with alcohol
3- End stage liver with ascites
4- End stage liver with encephalopathy

Ectopic pregnancy done salpingectomy how to follow up:


a) weekly BHCG until unremarkable
b) Outpatient visit (US) after six weeks
Infant abandoned by his mother after delivery, pediatric surgeon reviewed the
case and it was hernia, he recommended to do surgery now, what should you do?
- refuse to do surgery without consent
- consult ethics committee in hospital
- do surgery without consent
- fourth option can not remember it

The most common presentation in compartment syndrome is severe pain, what


mechanism is responsible for this ?
- skin hypoxia
- nerve hypoxia
- bone hypoxia
- muscles engorgement

Women pregnant 34+2 weeks had cesarean section past pregnancy due to non-
usshuring fetal monitoring, present now for external cephalic version , us
amniotic fluid index 14, responsive ctg
What would be absolute contraindication for ecv ?
Absolute contraindication:
Previous cs Previous scar on the uterus (UNLESS low transverse CS)
Placenta praevia
Ctg results Unexplained APH
Pre-eclampsia severe may go to eclampsia , HEELP .
Multiple pregnancy
Us findings,
fourth option cant remember Relative contraindications:
Rhesus isoimmunisation AB + indirect coomb >we don’t want to reach Critical
value . If AB - give rhogam then do conversion BCS of chance of FMH .
Elderly primigravida precious pregnancy .
IUGR
Oligohydramnios
Polyhydramnios May return back
Child come to clinic with cough fever (38.6) for 3 days with rash started on face
then spread to trunk and white pustule on erythematous base on the buccal
mucosa (opposite the lower 1st & 2nd molars) , dx ?
- measles
- rubella
- scarlet fever
Other option can't remember

Pt come to clinic with leg painless swelling , no hx of trauma, on examination leg


is pale cold and blueish discoloration is noted, peripheral pulses are intact, what is
the appropriate method to confirm diagnosis ?
- ct angio
- MRI
- arterial duplex
- venus duplex

Child come with cola urine , pt has hx of tonsillitis, PSGN is suspected, what test
would help you confirm your diagnosis ?
- blood culture
- anti-streptomyces
- c3 low
10 year old boy come with hx of diarrhea more than one month , there is
abdominal pain with blood sometimes and mucous sometimes, the child hasn't
been loosing weight, the child is reporting that he feels incomplete evacuation
even when there is nothing left in his bowel,
What would be appropriate to try first ?
- metronidazole
- gluten free diet
- ct
- some other option

Surgeon was doing lab cholecystectomy and faced difficult anatomy there was an
injury and did x- ray showed air in abdominal cavity and thorax, what organ was
damaged?
Esophagus
Stomach
Pancreas

Same case but there was bleeding and repaired


After surgery what would you do ?
Tell the patient what happened
Tell the patient lawyer to prepare for lawsuit
Tell hospital administrators to take action against you
Don't tell any body
Case about some labs with high calcium and normal PTH (1.1) phosphate was not
given, renal functions were elevated slightly and asking about what is the
diagnosis ?
- renal failure
- 2ry hyperparathyroidism
- primary hyperparathyroidism
- tertiary hyperparathyroidism

Case about hip joint avascular necrosis, what is management ?


I chose surgical repair

Case about septic arthritis and asking about diagnosis ?


I chose fluid aspiration

Case about teenager had argument with her mother and swallowed 20 pills of
acetaminophen come to emergency the second day having abd.pain and nausea
vomiting, asking about stage ?
Stage 1
Stage 2
Stage 3
Stage 4
Case about thyroid nodule , FNA was done and result were inadequate, asking
what to do?
Thyroid iodine scan
Repeat FNA
Thyroidectomy

Case about child with lower limb movement loss after URTI, child describes his
problem started from down and ascending upward, there was something about
his sensation in lower limb but as I remember it wasn't lost completely, asking
about diagnosis?
- GBS
- Transverse myelitis
- other options weren't related to neuro

30 y old women comes with painless vaginal mass under urethral orfice that
bleeds when touched , picture was provided
What is dx
Bartholin abscess (wrong option )
Other options can't remember
Woman lactating come to clinic complaining of right breast mass, firm with
erythema and tenderness, no discharge asking about diagnosis ?
- abscess
- fibroadenoma
- duct ectasia

Child 5 years old diagnosed with tonsillitis, and asking about his brother (2 years)
prophylaxis ?
I chose penicillin
Other cant remember

Infant abandoned by his mother after delivery, pediatric surgen reviewed the case
and it was hernia, he recommended to do surgery now, what should you do?
- refuse to do surgery without consent
- consult ethics committee in hospital
- do surgery without consent
- fourth option can not remember it
Child is treated for eczema with topical steroid, comes to clinic with itching and
pustulw lesions on top of his eczema, arranged in grape like pattern,
What is the most likely organism that causes his superimposed infection :
Herpes simplex
Staph
Group a strept

The most common presentation in compartment syndrome is severe pain, what


mechanism is responsible for this ?
- skin hypoxia
- nerve hypoxia
- bone hypoxia
- muscles engorgement

Woman come to clinic with history of multiple abortions, now she is 11 week
pregnant, what is the best predictor of complication in pregnancy ?
Abd pain
Vaginal bleeding
Other options
Surgeon was doing lab cholecystectomy and faced difficult anatomy there was an
injury and did x- ray showed air in abdominal cavity and thorax, what organ was
damaged?
Esophagus
Stomach
Pancreas

Case about some labs with high calcium and normal PTH (1.1) phosphate was not
given, renal functions were elevated slightly and asking about what is the
diagnosis ?
- renal failure
- 2ry hyperparathyroidism
- primary hyperparathyroidism
- tertiary hyperparathyroidism

Same case but there was bleeding and repaired


After surgery what would you do ?
Tell the patient what happened
Tell the patient lawyer to prepare for lawsuit
Tell hospital administrators to take action against you
Don't tell any body
Child is treated for eczema with topical steroid, comes to clinic with itching and
pustulw lesions on top of his eczema, arrranged in grape like pattern,
What is the most likley organism that causes his superimposed infection :
Herpes simplex
Staph
Group a strept

Child come with fever and headache, there is menengeal irrtation sings on
examination, Lp done with results :
Glucose 2.9 Normal 2.4-4.1
Protein 560 normal range 150-450

What ttt is best :


Ampicellin + vancomycin
Vancomycin + rifampin
Ampicill + somthing
Vancomycin + ceftriaxon

Woman come to clinic with history of multiple abortions, now she is 11 week
pregnant, what is the best predector of complication in pregnancy ?
Abd pain
Vaginal bleeding
Other options
New test was introduced to 800 of patients who was proven th have liver cirrhosis
And other 800 who didn't have liver disease
200 of first group tested positive and 100 from 2nd group tested positive
What is the sensitivity of this test ?
25%
33%
57%

Woman admitted with DVT diagnosis and was started on enoxaparin


The same night she develops SOB, ct confirms lower pulmonary artery thrombus,
what is the appropriate action ?
Change enoxaparin to sodium heparin
Start thrombolytic therapy

78 years old male admitted to ICU for MI that was complicated by pneumonia in
hospital, he received tazocin for 12 days, now complains of right upper quadrent
pain with tenderness, us was done with report of thick gallbladder wall with fluid
collection and no stones
What is the management?
-Emergency cholecyctectomy
- ercp drainage
- us guided drainage
- continue same medication
Pt DM, HTN with chronic heart failure, comes to clinic for medication counciling,
he takes insulin l, furocemide, enalapril, digoxin
Some tests values where provided
K = 5.9
Glucose high
Bp normal
What drug should he stop? enalapril

Infant abandoned by his mother after delivery, pediatric surgen reviewed the case
and it was hernia, he recommended to do surgery now, what should you do?
- refuse to do surgery without consent
- consult ethics committee in hospital
- do surgery without consent
- fourth option can not remember it

The most common presentation in compartment syndrome is severe pain, what


mechanism is responsible for this ?
- skin hypoxia
- nerve hypoxia
- bone hypoxia
- muscles engorgement
Women pregnant 34+2 weeks had cesarean section past pregnancy due to non-
usshuring fetal monitoring, present now for external cephalic version , us
amniotic fluid index 14, responsive ctg
What would be absolute contraindication for ecv ?
Previous cs
Ctg results
Us findings,
fourth option cant remember

Child come to clinic with cough fever (38.6) for 3 days with rash started on face
then spread to trunk and white pustule on erythematous base on the buccal
mucosa (opposite the lower 1st & 2nd molars) , dx ?
- measels
- rubella
- scarlet fever
Other option can't remember

Pt come to clinic with leg painless swelling , no hx of trauma, on examination leg


is pale cold and blueish discoloration is noted, peripheral pulses are intact, what is
the appropriate method to confirm diagnosis ?
- ct angio
- MRI
- arterial duplex
- venus duplex
Child come with cola urine , pt has hx of tonsilitis, PSGN is suspected, what test
would help you confirm your diagnosis ?
- blood culture
- anti-streptomyze
- c3 low

10 year old boy come with hx of diarrhea more than one m onth, there is
abdominal pain with blood somtimes and mucous sometimes, the child hasn't
been loosing weight, the child is reporting that he feels incomplete evacuation
even when there is nothing left in his bowel,
What would be appropriate to try first ?
- metronidazole
- gluten free diet
- ct
- some other option

Surgeon was doing lab cholecyctectomy and faced difficult anatomy there was an
injury and did x- ray showed air in abdominal cavity and thorax, what organ was
damaged?
Esophagus
Stomac
Pancreas
Patient with fatigue weakness and tingling diagnosed megaloblastic anemia

deficiency of which vitamin

B1

B2

B6

B12

Trauma to the knee lateral inferior caused fracture to the head of fibula what

nerve is affected :

-Tibial

-common peroneal

-deep peroneal

-superficial peroneal
Epigastric pulsating mass which investigation :

a) Abdominal US

b) ct

Vaginal discharge white fishy odor what is the diagnosis :

-bacterial vaginosis

-Trichomoniasis

Long case Patient with signs and symptoms of Eclampsia what is the emergency

medication for bp:

-methyldopa

-hydralazine

-Na nitroprusside
UTI Medication contraindicated in pregnancy:
-amoxicillin
-ciprofloxacin

Long Case of irritable bowl syndrome what to role out :


-thyroid disease
-celiac disease
-h. Pylori

Long Case of myasthenia gravies what is the medication : -pyridostigmine

-Eczema and thrombocytopenia diagnosis :


Weskit Aldrich syndrome

-pain and mass in anal average no fever:


Anal fissures
Anal abscess
All ethics repeated from the file above

-hernia bilaterally surgery to do :


Laparoscopic surgery not open because bilateral

-ketoacidosis with hypotension 80/60 and hyperglycemia important next step:

Fluid replaced ✅
Insulin infusion

-low osmolarity blood and low Na and in urine high osmolarity:


SIADH

-ECG :
Inferior MI

-Q about pericarditis can’t remember

-greenish discharge (trichomonas) Rx:


Metronidazole

-asthma management moderate asthma :


LABA and ICL
-child immunocompromised CI :
Varicella vaccine

-pregnant outbreaks for her child in the school what to give her :
Influenzas vaccine

- pregnant tender uterus and bleeding heavy :


Abruption placenta

-sometimes bloody diarrhea :


Chron

-Q postprandial abdominal destination and diarrhea:


Celiac disease

-Q about sickle cell anemia I can’t remember

-labs of anemia and TIBS normal :


Thalassemia

Anal hematoma ✅

Woman lactating come to clinic complaining of right breast mass, firm with
erythema and tenderness, no discharge asking about diagnos is ?
-

abscess
- fibroadenoma
- duct ectasia
1.Man with blunt trauma , what site of aorta is injured ?

A- aortic arch

B- distal part of aorta in subclavian site

C- proximal part of aorta

2.Man in ER with pain in the right upper abdomen with vomiting ultrasound shows thick wall
of gallbladder and fluid accumulation around management

A - lap cholecystectomy after 12 weeks

B - percutaneous drain

C - open cholecystectomy

3.Appendicitis without pus management

A- lap appendictomy after 12 weeks

B- open appendictomy after 12 weeks

C - immediate appendictomy

4.Symptom confirm diagnosis of eosinophilic esophagitis

A- dysphagia

B- stucking of food in esophagus

C - chest pain centrally located not responed to anti acid

D - confirmation by biopsy and endoscopy

5.Most important prognostic and preventing factor post MI :

A- Decrease lipid

B - stop smoking

C- decrease weight

D - exercise

6.Patient with fever right upper abdominal pain +ve serology of ecchinococcus ( hydatid
disese) **picture management

A- albenadzole

B- cyst. ???

C- Laparoscopic cystectomy

D - percutaneous drainage

7.pt. -> apendectomy

Most common infection post op.

A- fragilis bacterial

B- staph

C- shigella

D- pseudomonas

8.protin +2, glucose high, pregnant 37 wks + 8 days,

management ?

A- induction of labour

B- CS

9.treatment of chlymedia

A- azithromycin

B- doxy cyclin

10.child frequent eye blinking, normal all

A- Tic disorder

B- eye dryness

11.6 months baby have oral candida

A- oral anti fungal

B- systemic anti fungal

C- oral and systemic anti fungal

12.pregnant 2nd trimester -> BHCG, indicate what?

A- normal pregnancy

B- complete …?

C- Down

13._ ‫ وحده شربت لبن وبعده‬fever, abdominal pain, low back pain

brucella, ttt 6 weeks ??

14. prevlence of HCV in needle stick

30%

3%

.03%

15.pt will do thyroidectomy suffer from MR give?

A- amoxicillin for endocarditis

B- cefazoline for wound infection

C- amox & cefa together

16.pt with constipation for 2 days suffer from sever anal pain ex at 6 & 12 o’clock

A- not laceration

B- int sphincter abcess

C- anal fissure

D- fistula

17.post menopausal flushing cause

A- high FSH

B- VD

C- VC

18.You was talking to patient to get details on the history but the patient interrupted u and start
to talk about his point of veiw regarding his diagnosis and the reason of his disease . How u
would act :

A. Refer him to another doctor

B. Let him finish his point of veiw

c. Apologize to him and start with close ended questions d. I forget this one

19.hormonal replacement therapy ?

‫ الزم نعرف عندها‬uterus or post hystrectomy

20.how to diagnose recovery from preeclampsia

A- BP

B- protinurea 3 test

C- protinurea 5 test

21.hypertensive lady controlled in pregnancy may complicated by

A- abnormal fetal presentation

B- abnormal placenta

C- preterm

22.40 yr parathyroid adenoma

‫ متى اضطر أعمل‬parathyroidectomy

A- age > 50

B- osteoprosis

23.TIA

‫ممكن تعمل ايه في ال‬kidney

Acute tubular necrosis ?

24.‫
مريض بيعمل صبغة‬
precaution ?

-Diuretic (before )

-Nacl (After )

25.swelling med line under thyroid bone, motile

thyroglossal cyst

26.Neck mass ; biopsied showing (thyroid folicullar cells)

A- thyroid follicular cell

B- ectopic thyroid

27.thyroid mass

cytology -> medullary thyroid carcinoma, rt lobe ‫


سليم‬
A- total

B- subtotal

C-radiotherapy

D-chemotherapy

28.Most common cause of breast bloody discharge?

1- Fibroadenoma

2- Ductal Papilloma

3- Mammary Duct Ectasia

4- Abscess

29.23 Years old woman with cyclic bilateral nodularity in her breast since 6 months. On
examination there is tender 3 cm mobile subareolar mass on her right breast. What will you do
next? 

1- FNA with cytology 

2- Mammogram 

3- Follow up for next cycle 

4- Observation

30.Pectoralis major muscle was removed during mastectomy. Which action will be lost?

1- Abduction

2- Adduction 

3- Extension 

4- Internal rotation or flexion

31.24 Years old girl came to you with a painless mobile breast mass that does not change with
menstrual cycle. It started increasing since 3 months. Now it’s 3 cm in size. What is the most
likely cause?

1- Fibroadenoma 

2- Ductal Papilloma 

3- Paget’s disease of nipple

32.They want to stop screening for breast cancer for women under 47 to decrease
unnecessary anxiety to the public. You didn’t agree because there is a gene that cause cancer
in young women. What is the gene? 

1- APC 

2- BRCA2 

3- CA125

33.Which of the following suggest ductal papilloma on breast examination?

1- Blood stain on nipple 



2- Hemorrhagic discharge 

3- Serous discharge 

4- Pus from nipple

34.What will tell you tell a young lady in regard to breast cancer? 

1- Self exam is obsolete now 

2- Self exam as well as mammography are important 

3- Only mammography 

4- CT scan

35.What is the best study to determine cystic breast lesion? 



1- Biopsy 

2- FNA 

3- Mammogram 

4- U/S

36.Patient presented with hard mass on the outer upper area of the breast. which lymph node
you have to examine? 

1- posterior axillary 

2- anterior axillary 

3- lateral axillary

4- medial axillary

37.What is the most common site of breast cancer?

1- Subareolar 

2- Upper Lateral Quadrant 

3- Upper Medial Quadrant

38.A lactating woman 10 days after delivery complaining of fever and rigors. On examination:
tender left breast and nodules in upper outer area , What is the most likely the diagnosis? 

1- Postpartum sepsis. 

2- Breast abscess. 

3- Inflammatory breast cancer.

39.What is the cause of skin dimpling in breast cancer?

1- Cooper ligament 

2- Lactiferous duct 

3- Peau D’orange

40.30 years old female dancer with breast mass that disappeared by aspiration ? 

1- Fibroadenoma

2- Fibrocystic Change 

3- Ductal papilloma 

4- Galactocele

41.Bilateral breast cancer with : 



1- paget disease

2- lobular carcinoma

3- mucinous carcinoma

4- ductal carcinoma

1._ lesion in upper thigh ( irrigular margin )

A- excision biopsy

B- bunch biopsy

2.‫ هنا غالبا بيقولك‬old pt had bilateral shoulder pain now bilateral hip pain

3.Celiac disease biopsy done , showing villous atrophy ,

What else could you finding?

B- subtle crypt lengthening

C- high lymphocyte

4.laparoscopic cholecystectomy come with discharge from middle

mangement ?

A- daily dressing

B- wound inspect

C- exploration

5.sickle cel anemia, splenomegaly, hypersplenism

CBC ‫
كل حاجة قليل‬
A- exchange transfusion

B- hydroxy urea

6.meningitis -> LP

G+ve, b B hemolytic +ve, coagulase +ve

ttt?

A- Ampicillin

B- cefapime

7.? _ tension pneumothorax, best ttt ?

A- thoracocnthesis

B- tube thoracotomy

8.pulmonary edema CHF with poor prognosis best ttt with diuretic

9.Parkinson disease Dx ->


Clinical diagnosis CT

10.cystic fibrosis or celiac

‫ اجتمعوا مع الدكتور عشان يعرفوا كيفية التعامل‬+ prognosis


اللي ف الجلد اسمها إيش ؟‬
A- summarisation

B- verbal communication

11.- pic of fracture -> debridement and surgery

12._ pic of HSP


Ttt ? Suppurative treatment

13.pic of vulva and papule ->

reassurance

More investigation

14.3 yo child :

A) draws triangle

B) use stairs

15.smoking + pregnancy ….

16.pic or discerption -> cauliflower -> condyloma accuminata

17.case of hyperkalemia.. ttt -> Ca gluconate

18.Physical activity in adult

A- 1/2 hour in 5 days

B- 1/3 hour in 6 days

19.hematemesis -> chronic liver disease

Esophageal varices

20.hematemesis -> endoscope -> resuscitation

Before endoscope what you’ll give

A- vasopressin

21.hematemesis, no h/o

peptic ulcer

or Erosive gastritis

or Mallory weiss syndrome

22.CTG, deceleration

Late -> placenta insufficincy

Early -> head compression

Variable -> cord compression

23.seizure in pregnancy

A- Mg sulfate

24.child have abdominal pain, testes tender, horizontal testis above the other one

A-US

B- scrotal exploration

C- angiography

25.ttt of testicular torsion ?

Surgery

26.egg allergy vaccine -> yellow fever

27.Which vaccine cause GBS ?

28.vaccine contraindicated in pregnancy ?

29.child with low immunity, which vaccine we shouldn’t give ?

Varicella

30._ ‫ طفل ياخذ ستيرويد من فترة طويلة وصار عنده‬cushinoid app


وعايزين نعطيه تطعيمة نعمل ايه ؟‬
A- no problem to give him vaccine

B- give immunoglobulins

C- give him vaccine after 1 month

31.case of shock ( cardiogenic ) increase capillary pressure

With fever

A- bacteremia sepsis

B- septic shock

32.48 y.o female, without any family history of Breast CA with Normal mammogram when to
repeat it?

1- 2 years

2- 3 years

3- 5 years

33.Case scenario about patient with breast lump in upper outer quadrant which is tender and
increase within days from her period asking about the diagnosis?

1- Fibroadenosis

2- Fibroadenoma

3- lactational mastitis

34.In simple mastectomy what structures are to be excised ?

1- all breast tissue, skin, pectoralis muscle, axillary nodes

2- all breast tissue, skin, and axillary nodes

3- all breast tissue and skin

35.31 years old lady, found a lump in the upper outer area of her right breast, it was large, firm
with irregular border, not tender, no palpable lymph node, what should you do?

1- FNA.

2- lumpectomy and radiation.

3- mastectomy.

4- Core Biopsy

36.Patient after radical mastectomy developed winged scapula, why?

1- due to excision of Pectoralis muscle

2- Due to injury to the axillary nerve

3- Due to injury the long thoracic nerve

37.Patient after radical mastectomy can't raise her hand above the shoulder?

1- due to excision of Pectoralis muscle

2- Due to injury to the axillary nerve

3- Due to injury the long thoracic nerve

38.Best prognostic indicator for breast cancer?

1- Estrogen receptor

2- Stage and grade

3- Involvement of axillary LN

39.Most specific predictor of local recurrence in breast cancer.

1- size.

2- Lymph Node Metastasis.

3- site.

4- Age.

40.What is the commonest nerve injured after thyroidectomy?

•Superior Laryngeal Nerve

•External Laryngeal Nerve

•Recurrent Laryngeal Nerve

41.45 years old female presented with neck swelling and anxiety. On examination the swelling
is moving with swallowing and lateral to midline. What is the most likely diagnosis?

•Thyroglossal cyst

•Branchial cyst

•Thyroid nodule

42.which thyroid cancer have the best prognosis?

•Papillary Carcinoma

•Follicular Carcinoma

•Atypical Carcinoma

•Medullary Carcinoma

43.mass in the neck in cervical area, the best initial investigation?

•Chest X-ray

•U/S

•FNA

•CT

44.In thyroidectomy which nerve injury lead to horseness of voice?

•Superior Laryngeal Nerve

•External Laryngeal Nerve

•Recurrent Laryngeal Nerve

45.Generalized cervical lymphadenopathy + mild tenderness + low grade fever. What's the
most likely diagnosis:

•Viral Infection

•lymphocytic lymphoma

•hodgkin's lymphoma

46.Lump in throut barium enima and endoscopy normal?

•Squamous cancer

•Gerd

•Peptic ulcer

•Globus pharyngeos
1- Doctor ask resident to give abx but resident think pt had sensitivity to it :

A-Ask doctor to sign the order

2-Family think vaccine harm son :

A-Explain to them to advatages of vaccines

3- Elective surgey to pt , pt don’t want to know details but anesthesiologist insist to tell her :

A- Tell pt anesthesiologist decision

B- Reschedule surgery until see onther anesthesiologist

4-4- 15 years old male come to er . l.p should done to r/o meningitis :

A- Consent from parents

B- No consent it is an er

C- Consent from pt

D- Consent parents and ancent pt .


5- Pt insisted to take HRT and you tell her it is not fit for her :

A-Refuse to give .

6- Couples ask for surrogacy :

A-Tell them it might be illegal in s.a

7-Increase domestic violence in ?

A- pregnancy

8-When to start education about breast feeding ?

A-2nd trimester

B-3rd trimester

C-after delivery

D-before preg

9-Asthmatic pt not controlled on SABA ?

A- ADD ICS

10-Sign of of acute severe asthma , pt best pef 400 :

A-rr>25

B-hr >100

C-o2 sat <95%

D-pef <300

11-Pt known COPD , come to ER drowsy , o2 sat 84 % , ABGS : I think resp.acidosis :?

A-o2 by mask

B-mechanical ventilation

C-nebu

12-Type of SBG in excessive vomiting ?

metabolic alkalosis with hypokalemia

13-Pt known case of PUD diagnosed 8 years ago come with vomiting 2 weeks ago on exam
dehydrated and abd squash test + what expected abg :

A-metabolic acidosis

B-Metabolic alkalosis

C-compensated

14-Umbilical hernia baby 10 months ?

A-observation

15-2 days old baby , seizure , lp analysis nl , Dx ?

A-neonatal sepsis

B-asphyxia

16-Child with pneumonia ( in rt middle lobe ) with x ray , vital signs stable ?

A-admission for iv abx

B- prescribe amoxicillin for 7 days and discharge

17-Pt had retained placenta and has bleeding what type of PPH ?

A-primary

B- secondary

18-Pt in labot dilatation 5cm since 2 hour , effac 100 , station 0 ?

A-observe

B-give oxytocin

18-Pt in labot dilatation 5cm since 2 hour , effac 100 , station 0 ?

A-observe

B-give oxytocin

19-Case of pph what to give ?

A-oxytocin

B-ergot

C-carboprost

20-Pt want smoking cessation with HTN and epilepsy what contraindicated ?

A-bupropion

21-Pt has hx of recurrent PPH what to do ?

A-active management of 3rd stage

22-Pt prev c/s has abd pain (36+ weeks ) vitals : hypotensive , tachycardia dx ?

A-uterine rupture

B-abruptio

23-Yellowish greenish vaginal discharge increase with intercourse ?

A-trichomonas

B-bact vaginosis

24-Pt had menopause at age of 40 ?

A- increase risk for osteoporosis

25-Anal itching and pain after defecation ?

A- anal fistula

B-anal fissure

27-Newborn with hypoglycemia what to give ?

A-2ml/kg of 10% dextrose

B-2ml/kg of 12.5% dextrose

C-4ml/kg of 10% dextrose

D-4 ml/kg 12.5% dextrose

28-Newborn of diabetic mother from where take sample

A-central venosuline

B-peripheral

C-nasogastric

D-orogastric

29- 20 years old male with meningitis and seizure rx ?

A-vanco and ceftr

B-vanco + ceft + steroid

30- 50 years old male with meningitis most common pathogen ?

A-strep p.

B-listeria

31-22 weeks gestation , os dilated 1.5 com ?

A-cervical incompetence

32-1st trimester , os closed , mild spotting . type of abortion ?

33-1st trimester, os closed and speculum heavy bleeding, type of abortion?

34-Ectopic pregnancy treated with salpingostomy what next ?

A-no follow up needed

B-follow up bhcg after a week and till zero,

35-Pt has MI 6 wweks ago come for elective laprascopic cholecystectomy ?

A-do it now

B-delay 6 months

36-Child with jaundice and anemia , blood film , coombs direct and indirect + , retic high ?

A-autoimmune hemolytic

B-spherocytosis

37-Inguinal hernia reducable rx ?

A-simple repair

B-open

C-laprascopic

D-observe

38-what’s hemosiderin disposition in macrophages in the lung in indicative of?

A-CMV

B-chronic lung infection C. Heart failure

C-pneumocystis jiroveci

39- which group of drugs is contraindicated in diabetic patients with heart failure?

A-biguanides

B- dipeptidyl peptides IV inhibitors

C- GLP-1 analogous

D- SGLT-2 inhibitors

E- thiazolidinediones
18-year-old healthy male was playing baseball and suddenly he felt abdominal
pain. On examination he has para-umbilical mass. His vital signs are: BP 100/76,
RR 30 , HR 100 , O2 sat is 95 % with 100 % oxygen mask , what is your next step in
the management?
A)Abdominal US
B)CT scan.
C)Erect Chest x-ray
D)Reassure and send home

Post herniorraphy , wound showed pus , mesh come out side the wound
A laparoscopic drainage
B surgical wound exploration
C Debridement, dressing change

48 years old female with abnormal uterine bleeding on u/s there was subserous
fibroid, endometrial thickning was 4mm
A fibromyoma
B fibrosarcoma
C myosarcoma

1- Female patient has pain on mid


Planter line increases e walking
A-planter fasciitis
2- female patient 2 symptoms with pain increases through the day:
- Abs against choli??
3- came with diarrhea associated with dehydration leading to weakness , flat T
wave on ecg. What the cause?
- hyponatremia
- hypokalemia
- hypomag
- hypocalcemia
4- came with loin pain on IVU, non opaque patche on
Us hyperdence echo

5- bacterial vaginosis
6- truchimonas
7- old age came with hx of hepatitis c , on US

8- came with obs jaundice associated with wt loss


- parabilliary carcinoma
- k..... syndrome

9- fistula on anal opening ,....... what management?


Fistiolotomy
Fistulogram

11- pt with pe controlled symptoms was on iv heparin for 5 days. Inr 1.2 what to
do?
- stop heparin
- change to enoxaprin
12- inferior MI , post emergent management?
- thrombolytics
- pci with stent
- heparin

١٠-Pt e herniotomy c/o of pus discharge from Mish come out of it


A-wound drainage and mish removal
B- open wound
C-systemic Abx

Pt diagnosed e retroprotneal cancer what TNM about


A-tansmition to lymphatic
B-metastasis
C-‫مش فاكره الباقي‬
‫ وانا شايفه انو‬a‫و‬b ‫نفس الفهم‬

Croup case mangment


Abx
O2
I think the 2not mangment of croup
But o2 may be !!
Intussuption mangment ?!
Saline enema
nasogastric tube
Also the 2option not right
I.v fluid and surgery

Parmenstal syndrom➡irritability

.....HCV in KSA

Dr wrong applied forcep during vaginal delivary crush of sternomastoid foramen


and its content what you expected to find in this baby :

a. Unable to close the eye ✔


b. Loss of taste in ant 2/3 of tongue
c. Unable to open the eye d. Sensation in face lost
‫قريب للسؤال‬
Fascial palsy

16y boy come e dysuria then after 3day develop foul smelling what the organism
A-porteus
B-klebsella
C-
While make cholecystectomy you find mass what will do
A-cholecystectomy only
B-cholecystectomy and remove the mass
C-off the procuder

Elderly c/o dysuria and frequency


What inves to do
A-urine analysis
B-CT Abd

85 years male c/o of ..... sever headache while lefting some thing
A-CT brain ,cervical spine
B-MRI

Case of ASD what true about it


A-it will done about 5 to 6 year
B-must make it at 2 year

Primagravida come e threatend miscarriage after 2 year of infertility what to do


A-ask about infertility
B-pelvic U/s
C-conforming her preg
Child come to ER e meningitis e his parent you want to do csf analysis
A-do it without consent
B-consent from parents abisttant from child
C-consent from parents abisttant from parents
D-consent from both
‫ يعني ايه‬abisttant

Old age of Adenxal mass ..... by u/s what to do next


A-surgical excetion
B-refer for gynecologist work up
C-cytology

Asthmatic pt on short acting and low dose steroid and still has daily symptoms
about 5day at night what to add
A-theophylline
B-leukoterian inhaler
C-no thing

IBS come e abd pain x-Ray show thumb points


A-ischemic colitis
B-ulcerative colitis

C-conns
Newborn of diabetic mother from where take sample
A-central venousline
B-peripheral
C-nasogastric !!
D-oro…
Some NEW Questions from
my exam today ...
MAY 2019
>>26 yrs Man è Hbsag +ve ,hbeag +ve on routine screening ..what is next
;
-observe
-HBV DNA study
-Interferon
-take liver biopsy

>>Trauma è skin erythema and Partiel skin loss


,ttt
-H2o irrigation
-Bicarb. Irrigation
-Surgical debridment
>>Old female,obese on diet control ,polydepsia , invest. Hba1c 7.5 ,all
(random,Fasting, post prandial ) sugar are mild elevated urine ketone (+) ttt
-Insulin
-pioglitazone
-Thiazolidinediones
-fourth i forgot (not Metformin nor sulfonylurea)

>>Contraindicated in asthmatic female ē PPHge;


-Oxytocin
-Prostaglandin F2 (or alfa2 not sure)
-Methergine
-FFP

>>Day to day variation of this not suitable for patients under warfarin therapy:
-retinols
-Fresh fruits and vegitables
-Meet and paultry
-Old cheese

>>We advise Heart failure pt to


-markedly decrease salt in food √
>>Best indicator for CHRONIC RENAL FAILURE
-24 hrs urine ptn
-creatinine clearance
GFR

>>50 children are already diseased..35 of them are positive via a screening
test..what is this test sensetivity ?
30%
50%
70% √
90%

>>Alcoholic 6 yrs è persistant vomiting fatigue epigastric pain ,sense of dull aching
epigastric mass :
_choledocal cyst
_Pancreatic pseudocyst
-Pancreatic cystadenoma
>>Acne ē cyst formation(some thing like that) ,marked scarring ,ttt
-Oral isoretinoid (spelling not sure)
-Oral minocycline
-Topical steroids
-Topical erythromycin

>>7 yrs old boy with yellowish discoloration of teeth and caries ttt
-Mouth antiseptic wash
-Diet control
-Local oral antibiotics
-(Oral systemic) Flouride

>>62 yrs male ē idiopathic subvalvular hypertrophic aortic stenosis going for
dental extraction :
-Risk of endocarditis is 50%
-Risk of endocarditis is 12%
-Prophylactic AB not needed
-After procedure AB is sufficient
>>4yrs child ē anisocytosis and occasional macrocytes ,wbcs 11.5 ,Plt 65…Bone
marrow examination will show what ?:
-hypocellular
-hypercellular with microcytic activity
-hypercellular with megaloblastic activity(my answer)
-hypercellular with myeloblastic activity

>>7 yrs child asthmatic managed by Daily PEFR , daily oral systemic steroids(sure)
and Short b2 agonists ..his asthma severity is ?:
-Mild intermittant
-Mild persistant
-Moderate
-Severe
✓ A 57 year-old ship builder presents with increasing shortness of breath on exertion.
Chest x-ray reveals diffuse bilateral streaking shadows and pulmonary function testing
demonstrates a severe restrictive ventilatory defect as well as reduced gas transfer.
What is the most likely diagnosis? A. Berylliosis
B. Asthma
C. Asbestosis
D. Mesothelioma

2- A five year-old has had a painless limp for over one week. X-ray of the hip reveals severe
avascular necrosis of the left proximal femoral epiphysis. What is the most appropriate way to
treat
this patient? A. Physical therapy
B. Surgical correction
C. Serial x-rays of hip
D. No weight bearing for 6 months

3- A paediatric patient presented five days ago with viral gastroenteritis. After five days, the
diarrhoea and emesis resolved but the patient remains anorexic. What dietary advice should
be given?
✓ Chopped pears, yogurt, and bran cereal
✓ Bananas, rice, and apple sauce
✓ Granola, whole wheat toast and beans
✓ Ice cream, oranges and broth

4- A 66 year-old patient who has a cardiac pacemaker develops severe central sleep apnoea.
Although the patient has attained ideal weight, has altered sleep positions, and uses
continuous positive airway pressure, the condition has not improved. Which of the
following is now indicated?
A. Atropine
B. Atrial over-drive pacing
C. Nasal surgery
D. Pharyngeal surgery
5- What is the most common adverse effect of labour induction with oxytocin?
A. Maternal hypernatremia
B. Maternal hypertension
C. Foetal acidosis
D. Uteroplacental hypoperfusion

6- A 19 year-old presents with fever, sore throat, malaise, arthralgia, and a rash. The rash
involves the entire body, including the palms of the hands and soles of feet, but excluding the
face. The symptoms developed eight weeks after having an unprotected sexual exposure.
What is the most likely diagnosis?
A. Chancroid
B. Donovanosis
C. Syphilis
D. Chlamydia trachomatis

7- A patient present with multiple maculopapular rashes on their face, ears, wrists and elbows.
A skin biopsy shows numerous acid fast bacilli within macrophages in the dermis. What is the
most likely diagnosis?
A. Tuberculosis
B. Leprosy
C. Syphilis
D. Psoriasis

8- A patient suffering from epilepsy states, "I feel a seizure coming," then becomes rigid for 30
seconds, followed by a 90 second convulsion. Which of the following emergency measures
would most likely benefit the patient and reduce the risk for injury?
A. Use a wooden mouth gag
B. Hold the tongue with forceps
C. Maintain the airway
D.Apply physical restraints
-
9- A 30 year-old has a large indirect inguinal hernia sac that is reducible with mild effort. Bowel
sounds are heard in the hernia sac. This hernia is the result of which of the following?
A. A defect in the abdominal wall with protrusion of abdominal contents through the internal
inguinal ring, canal and external ring
B. A traumatic defect in the abdominal wall
C. A congenital defect in the abdominal wall allowing protrusion of abdominal contents
through the inguinal canal
D. A defect in the abdominal wall caused by constipation

10- A 55 year-old patient presents to the emergency room with a sudden onset of a very red
and painful eye with reduced visual acuity. The symptoms began while the patient was inside a
dimly lit movie theatre. Which of the following is the best explanation of the patient's
symptoms?
A. Iris inflammation
B. Iris dilitation
C. Cataract debris
D. Iris hyperpigmentation

11- A 50 year old female presents with complaints of anxiety. Two months ago during a
meeting at her job, she became very anxious, sweaty, and short of breath. She had to leave
the meeting in order to calm down. Since then, she has avoided the meeting room out of fear
that she will have a recurrence of the symptoms. What is the most likely diagnosis?
A. Panic disorder
B. Specific anxiety disorder
C. Post-traumatic stress disorder
D. Generalized anxiety disorder

12- A patient returning to the hospital three days after being discharged complains of a fever,
headache, retrobulbar pain, conjunctival suffusion, and a severe backache. Flavivrus infection
is diagnosed. What should be most appropriate public health measure?
A. Eradication of the animal reservoir
B. Appropriate cooking of food
C. Improvements in water supply
D. Isolation of infected patients
13- As the symptoms subside in a patient with Crohn's disease, in long-term management,
what the physician should monitor? A. Lupus-like syndrome development
B. Serum sickness-like reactions
C. Local and systemic infections
D. Immune and bone marrow suppression

14- A 31 year-old man kicked in the chest, presents two hours later with increasing left-sided
chest pain and difficulty breathing. Pulse is 110 beats/minute, respiratory rate is 40/minutes,
blood pressure is 100/70 mm, and oxygen saturation of 95% (see image). What is the most
appropriate immediate treatment?

A. Chest tube insertion


B. Thoracotomy
C. Needle aspiration
D. Pericardiocentesis
-
15- A 23 year old presents with a 10-day history of a pruritic skin eruption of well-
circumscribed wheals on an erythematous base. Examination reveals periorbital swelling, an
enlarged supraclavicular lymph node, an enlarged liver and an enlarged spleen. What is the
most likely diagnosis ? A. Cholinergic urticaria
B. Angioedema
C. Rheumatoid disease
D. Lymphoma

16- A patient present with sudden painless loss of vision. The retina is opaque, edematous and
has a cherry red spot on the fovea. What would be most appropriate treatment?
A. Elevate the head of the bed
B. Ocular massage
C. Ask the patient to roll the eyes
D. Dilate the pupils

17- A 25 year-old female presents with a two-week history of vaginal discharge due to
chlamydial infection. What is the most appropriate treatment?
A. Azithromycin
B. Doxycycline
C. Fluconazole
D. Metronidazole

18- A nulligravida 17 year-old complains of excruciating pain when menstruating. Menarche


was at age 14, her periods are regular, and flow is normal. The pain is midline, begins with the
flow, is cramp-like, may radiate to her thighs, and is occasionally associated with nausea. In the
presence of a normal examination, what is the most likely pathophysiology?
A. Uterine distension (hematometria)
B. Hypogastric serotonin resistance
C. Functioning endometrial tissue within the myometrium D. Endometrial prostaglandin
release

19- After surgery for resection of a stage III colon cancer, when a patient should have
chemotherapy?
A. As soon as possible
B. When lab values have normalized
C. After psychological evaluation
D. If less than 60 years old
20- Long-term, high-dose use of some opioids can lead to which of the following? A.
Neuropathic pain
B. Bell's palsy
C. Ischemic pain
D. Renal pain

21- A 61 year old patient constantly thinking about aliens landing in the backyard, especially
when away from home. Despite knowing that aliens do not exist, the patient feels
overwhelmed with the idea that they have landed. The patient fears this means the patient is
"going insane". What is the most likely diagnosis?
A. Obsession
B. Compulsion
C. Delusion
D. Hallucination

22- A pale and diaphoretic patient presents with severe left flank pain and vomiting. The blood
is concentrated and red cells appear normal. What is the most likely diagnosis?
A. Zollinger-Ellison syndrome
B. Fibrocystic nodulosis
C. Renal calculi
D. Sickle cell disease

23- A 10 year-old has just recovered from Rheumatic Fever. Because of rapid diagnosis and
completed treatment, there was no resulting heart disease. The family is advised that antibiotic
prophylaxis should be continued for period of time. For how long should this antibiotic be
taken?
A. 6 months
B. 3 years
C. 6 years
D. 15 years

24- What describes the most effective exercise plan for patients with coronary heart disease?
A. Isotonic exercises
B. Yoga exercises
C. Isometric exercises
D. Anaerobic exercises
-
25- A family brings a 65-year-old in because of a noticeable decline in intellectual function.
Examination shows no evidence of illness and psychological testing does not reveal depression.
The family asks for advice to slow the progress of suspected dementia (see table).
What the physician should advise?
A. Regular exercise may reduce the risk of developing dementia
B. Leisure activities involving mental analysis will prevent dementia
C. New methods of completing his activities of daily living are now indicated D. Homocysteine
lowering medication will protect from further decline

26- A patient has been diagnosed with an overdose. There are dilated pupils, convulsions and
hallucinations. What is the most likely cause?
A. Anticholinergics
B. Amphetamines
C. Anticonvulsants
D. Phenothiazines

27- A 25 year-old patient presents to the clinic with a history of persistent inability to attain
sufficient sexual arousal since getting married two years ago. On physical and gynaecological
examination are remarkable. What would be most appropriate treatment?
A. Bupropion
B. Estrogen
C. Sildenafil
D. Tibolone

28- A 42 year-old patient in the second trimester of pregnancy is concerned that her fetus may
have Down syndrome. What is the most appropriate investigation to rule out Down Syndrome?
A. Triple test
B. Amniocentesis
C. Cordocentesis
D. Chorionic villous biopsy
29- A 17 year-old football player presents with an area macular hypopigmentation on the trunk
and arms. The areas seem even lighter with sun exposure. What is the most appropriate
treatment regimen for this patient?
A. Topical steroid cream
B. Selenium sulphate
C. Oral antibiotics
D. Topical antibiotics

30- A 20 year-old with anaemia, pancytopenia and an enlarged spleen, develops a gradual
deterioration of mental facilities. Bone marrow examination shows the presence of large cells
with a crumpled tissue paper-like cytoplasm. What is the most likely diagnosis?
A. Glycogen storage disease
B. Niemann-Pick's disease
C. Alkaptonuria
D. Gaucher's disease

1c 2b 3b 4b 5d 6c 7b 8c 9a 10 b
11 a 12 a 13 d 14 a 15 d 16 b 17 a 18 d 19 a 20 a
21 a 22 c 23 d 24 a 25 a 26 b 27 c 28 b 29 d 30 d
-
1- Pregnant, 16 weeks, discovered to be RH –ve, what to do next?
A. ultrasound
B. amniocentesis
C. IV RH antibodies

2- Patient in 3rd trimester have high blood blood glucose despite close observation,
What is the suspect cause?
A. neonate hyperglycemia
B. neonate hypoglycemia
C. mother hyperglycemia

3- what is true about Pap smear?


A. 21 till 65 yrs repeated every year
B. 21 till 65 yrs repeated every 5 yrs
C. 21 till 35 repeated every 3 years
D. 30 till 65 repeated every 5 yrs

4- First sign of puberty for female?


A. Adrenarche
B. Thelarche
C. Pubarache
D. Menarche

5-Female present scanty vaginal bleeding she noticed that post coidal, what is you
action?
A. Pelvic us
B. CBC
C. assess the vagina and cervix

6-40yr.old woman asking about ca cx screening, pap smear when to do and asking for
ur advice ?
A. no need now
B. can be done 1 year starting from now.
C. can be done every 5 year from now
D. if 3 negative , no need to further test for screening.

7- pregnant lady in labor with active regular contractions every 3 mins and cervix dilated
5 cm After 4 hrs still the same and CTG sows grade 1 heart activity , What will u do?
A. oxytocin
B. instrumental delivery
C. CS
8- 36 week pregnant with history of
smoking came complaining of vaginal bleeding with uterine tenderness .diagnosis ?
A.Abruptio placenta
B.placenta previa
C. Vasa previa
D. Placenta accreta

9- Pregnant lady with no past history of chicken box .best to protect her from disease?
A. avoid contact with patient
B. IVIG
C. Acyclovir

10-pregnant has abruptio placenta what is the acid base balance of fetus?
A. asphyxia
B. respiratory acidosis
C. metabolic acidosis
-
Young 21 years old wife came complaining that her husband has bite her hand, what is the
reason behind this abuse?
1. Socio-economic status of husband.
2. Cultural background
Question was confusing

Surgeon done appendectomy, however during the surgery find out that the appendix is normal
and removed it, what does the surgeon suppose to do now?
1. Tell the patient that the appendix was normal only.
2. Make another surgeon inform the patient.
3. Inform health communication workers
4. Tell the patient that you followed the protocol that all surgeons and institutions follow in
these situations.

Old postmenopausal women asking you to give her Hormonal therapy?


1. Refuse to give her.

Female with symptoms of Cushing Syndrome, what to do?


1. Brain CT.
2. Give dexamethasone and observe response.
3. 24 hour collecting urine test.
4. Measure cortisol levels in blood.

Male with left leg pain, swelling (non-pitting), pulse intact, what is the most beneficial to make
the diagnosis?
1. CT angio
2. Duplex US
3. MRV
Elderly male present at night to ER with dyspnea, chest tightness, you did all investigation
including Echo, he did Cardiac surgery before, what is reason that might cause death to this
patient?
1. PE
2. Stroke
3. Pulmonary HTN

1. Pregnant women with strong indication for CS, she refuse the CS and want to deliver
vaginally, What to do?
A. Do general anesthesia and CS.
B. Take consent from husband regarding CS.
C. Try vaginal delivery.
D. Forget this option
The q was confusing.

2. 5 cases about COPD and ASTMA patients with severe acute attacks asking about
management which is if
- Comatose —> mechanical ventilation
- Non-comatose —> noninvasive ventilation

3. Pregnant in 2nd trimester diagnosed to have hyperthyroidism and prescribed to


hyperthyroidism medications, however refuse to take any medications, what to do?
- Force her to take the medications.
- Talk to her husband.
- Ignore her.
- Explain to her the consequences of untreated hyperthyroidism on her fetus.

26 yrs Man è Hbsag +ve ,hbeag +ve on routine screening ..what is next ;
-observe
-HBV DNA study
-Interferon
-take liver biopsy
-

>>Trauma è skin erythema and Partiel skin loss ,ttt


-H2o irrigation
-Bicarb. Irrigation
-Surgical debridment

>>Old female,obese on diet control ,polydepsia , invest. Hba1c 7.5 ,all (random,Fasting, post
prandial ) sugar are mild elevated urine ketone (+) ttt
-Insulin
-pioglitazone
-Thiazolidinediones
-fourth i forgot (not Metformin nor sulfonylurea)

>>Contraindicated in asthmatic female ē PPHge;


-Oxytocin
-Prostaglandin F2 (or alfa2 not sure)
-Methergine
-FFP

>>Day to day variation of this not suitable for patients under warfarin therapy:
-retinols
-Fresh fruits and vegitables
-Meet and paultry
-Old cheese

>>We advise Heart failure pt to


-markedly decrease salt in food √

>>Best indicator for CHRONIC RENAL FAILURE


-24 hrs urine ptn
-creatinine clearance
GFR
>>50 children are already diseased..35 of them are positive via a screening test..what is this
test sensetivity ?
30%
50%
70% √
90%

>>Alcoholic 6 yrs è persistant vomiting fatigue epigastric pain ,sense of dull aching epigastric
mass :
_choledocal cyst
_Pancreatic pseudocyst
-Pancreatic cystadenoma

>>Acne ē cyst formation(some thing like that) ,marked scarring ,ttt


-Oral isoretinoid (spelling not sure)
-Oral minocycline
-Topical steroids
-Topical erythromycin

>>7 yrs old boy with yellowish discoloration of teeth and caries ttt
-Mouth antiseptic wash
-Diet control
-Local oral antibiotics
-(Oral systemic) Flouride

>>62 yrs male ē idiopathic subvalvular hypertrophic aortic stenosis going for dental extraction
:
-Risk of endocarditis is 50%
-Risk of endocarditis is 12%
-Prophylactic AB not needed
-After procedure AB is sufficient
-
>>4yrs child ē anisocytosis and occasional macrocytes ,wbcs 11.5
Plt 65..Bone marrow examination will show:
-hypocellular
-hypercellular with microcytic activity
-hypercellular with megaloblastic activity(my answer)
-hypercellular with myeloblastic activity

>>7 yrs child asthmatic managed by


Daily PEFR , daily oral systemic steroids(sure) and Short b2 agonists ..his asthma severity :
-Mild intermittant
-Mild persistant
-Moderate
-Severe

DKA pation came ti Hospital , insulin and fulid given in hospital


Pation lab hyponatrima
What the cause
A-insulin
B - fliud
C- notrin antidiurtic hormone syndrom

Patient go to opertation Rome the surgeon said should be done ambutation the pation refuse
What should to do
A- do the surgery
B- till the hospital etihi communite
C - make the pation DAMA
1- A 57 year-old ship builder presents with increasing shortness of breath on exertion. Chest
x-ray reveals diffuse bilateral streaking shadows and pulmonary function testing demonstrates
a severe restrictive ventilatory defect as well as reduced gas transfer. What is the most likely
diagnosis?
A. Berylliosis
B. Asthma
C. Asbestosis
D. Mesothelioma

2- A five year-old has had a painless limp for over one week. X-ray of the hip reveals severe
avascular necrosis of the left proximal femoral epiphysis. What is the most appropriate way to
treat this patient?
A. Physical therapy
B. Surgical correction
C. Serial x-rays of hip
D. No weight bearing for 6 months

3- A paediatric patient presented five days ago with viral gastroenteritis. After five days, the
diarrhoea and emesis resolved but the patient remains anorexic. What dietary advice should
be given?
A. Chopped pears, yogurt, and bran cereal
B. Bananas, rice, and apple sauce
C. Granola, whole wheat toast and beans
D. Ice cream, oranges and broth

4- A 66 year-old patient who has a cardiac pacemaker develops severe central sleep apnoea.
Although the patient has attained ideal weight, has altered sleep positions, and uses
continuous positive airway pressure, the condition has not improved.
Which of the following is now indicated?
A. Atropine
B. Atrial over-drive pacing
C. Nasal surgery
D. Pharyngeal surgery

5- What is the most common adverse effect of labour induction with oxytocin?
A. Maternal hypernatremia
B. Maternal hypertension
C. Foetal acidosis
D. Uteroplacental hypoperfusion
6- A 19 year-old presents with fever, sore throat, malaise, arthralgia, and a rash. The rash
involves the entire body, including the palms of the hands and soles of feet, but excluding the
face. The symptoms developed eight weeks after having an unprotected sexual exposure.
What is the most likely diagnosis?
A. Chancroid
B. Donovanosis
C. Syphilis
D. Chlamydia trachomatis

7- A patient present with multiple maculopapular rashes on their face, ears, wrists and elbows.
A skin biopsy shows numerous acid fast bacilli within macrophages in the dermis.
What is the most likely diagnosis?
A. Tuberculosis
B. Leprosy
C. Syphilis
D. Psoriasis

8- A patient suffering from epilepsy states, "I feel a seizure coming," then becomes rigid for 30
seconds, followed by a 90 second convulsion. Which of the following emergency measures
would most likely benefit the patient and reduce the risk for injury?
A. Use a wooden mouth gag
B. Hold the tongue with forceps
C. Maintain the airway
D.Apply physical restraints

9- A 30 year-old has a large indirect inguinal hernia sac that is reducible with mild effort. Bowel
sounds are heard in the hernia sac. This hernia is the result of which of the following?
A. A defect in the abdominal wall with protrusion of abdominal contents through the internal
inguinal ring, canal and external ring
B. A traumatic defect in the abdominal wall
C. A congenital defect in the abdominal wall allowing protrusion of abdominal contents
through the inguinal canal
D. A defect in the abdominal wall caused by constipation

10- A 55 year-old patient presents to the emergency room with a sudden onset of a very red
and painful eye with reduced visual acuity. The symptoms began while the patient was inside a
dimly lit movie theatre. Which of the following is the best explanation of the patient's
symptoms?
A. Iris inflammation
B. Iris dilitation
C. Cataract debris
D. Iris hyperpigmentation
11- A 50 year-old female presents with complaints of anxiety. Two months ago during a
meeting at her job, she became very anxious, sweaty, and short of breath. She had to leave
the meeting in order to calm down. Since then, she has avoided the meeting room out of fear
that she will have a recurrence of the symptoms. What is the most likely diagnosis?
A. Panic disorder
B. Specific anxiety disorder
C. Post-traumatic stress disorder
D. Generalized anxiety disorder

12- A patient returning to the hospital three days after being discharged complains of a fever,
headache, retrobulbar pain, conjunctival suffusion, and a severe backache. Flavivrus infection
is diagnosed. What should be most appropriate public health measure?
A. Eradication of the animal reservoir
B. Appropriate cooking of food
C. Improvements in water supply
D. Isolation of infected patients

13- As the symptoms subside in a patient with Crohn's disease, in long-term management,
what the physician should monitor?
A. Lupus-like syndrome development
B. Serum sickness-like reactions
C. Local and systemic infections
D. Immune and bone marrow suppression

14- A 31 year-old man kicked in the chest, presents two hours later with increasing left-sided
chest pain and difficulty breathing. Pulse is 110 beats/minute, respiratory rate is 40/minutes,
blood pressure is 100/70 mm, and oxygen saturation of 95% (see image). What is the most
appropriate immediate treatment?

A. Chest tube insertion


B. Thoracotomy
C. Needle aspiration
D. Pericardiocentesis
15- A 23 year-old presents with a 10-day history of a pruritic skin eruption of
well-circumscribed wheals on an erythematous base. Examination reveals periorbital swelling,
an enlarged supraclavicular lymph node, an enlarged liver and an enlarged spleen. What is the
most likely diagnosis ?
A. Cholinergic urticaria
B. Angioedema
C. Rheumatoid disease
D. Lymphoma

16- A patient present with sudden painless loss of vision. The retina is opaque, edematous and
has a cherry red spot on the fovea. What would be most appropriate treatment?
A. Elevate the head of the bed
B. Ocular massage
C. Ask the patient to roll the eyes
D. Dilate the pupils

17- A 25 year-old female presents with a two-week history of vaginal discharge due to
chlamydial infection. What is the most appropriate treatment?
A. Azithromycin
B. Doxycycline
C. Fluconazole
D. Metronidazole

18- A nulligravida 17 year-old complains of excruciating pain when menstruating. Menarche


was at age 14, her periods are regular, and flow is normal. The pain is midline, begins with the
flow, is cramp-like, may radiate to her thighs, and is occasionally associated with nausea. In
the presence of a normal examination, what is the most likely pathophysiology?
A. Uterine distension (hematometria)
B. Hypogastric serotonin resistance
C. Functioning endometrial tissue within the myometrium
D. Endometrial prostaglandin release

19- After surgery for resection of a stage III colon cancer, when a patient should have
chemotherapy?
A. As soon as possible
B. When lab values have normalized
C. After psychological evaluation
D. If less than 60 years old

20- Long-term, high-dose use of some opioids can lead to which of the following?
A. Neuropathic pain
B. Bell's palsy
C. Ischemic pain
D. Renal pain
21- A 61 year-old patient constantly thinking about aliens landing in the backyard, especially
when away from home. Despite knowing that aliens do not exist, the patient feels
overwhelmed with the idea that they have landed. The patient fears this means the patient is
"going insane". What is the most likely diagnosis?
A. Obsession
B. Compulsion
C. Delusion
D. Hallucination

22- A pale and diaphoretic patient presents with severe left flank pain and vomiting. The blood
is concentrated and red cells appear normal. What is the most likely diagnosis?
A. Zollinger-Ellison syndrome
B. Fibrocystic nodulosis
C. Renal calculi
D. Sickle cell disease

23- A 10 year-old has just recovered from Rheumatic Fever. Because of rapid diagnosis and
completed treatment, there was no resulting heart disease. The family is advised that
antibiotic prophylaxis should be continued for period of time. For how long should this
antibiotic be taken?
A. 6 months
B. 3 years
C. 6 years
D. 15 years

24- What describes the most effective exercise plan for patients with coronary heart disease?
A. Isotonic exercises
B. Yoga exercises
C. Isometric exercises
D. Anaerobic exercises

25- A family brings a 65-year-old in because of a noticeable decline in intellectual function.


Examination shows no evidence of illness and psychological testing does not reveal depression.
The family asks for advice to slow the progress of suspected dementia (see table).
What the physician should advise?
A. Regular exercise may reduce the risk of developing dementia
B. Leisure activities involving mental analysis will prevent dementia
C. New methods of completing his activities of daily living are now indicated
D. Homocysteine lowering medication will protect from further decline

26- A patient has been diagnosed with an overdose. There are dilated pupils, convulsions and
hallucinations. What is the most likely cause?
A. Anticholinergics
B. Amphetamines
C. Anticonvulsants
D. Phenothiazines

27- A 25 year-old patient presents to the clinic with a history of persistent inability to attain
sufficient sexual arousal since getting married two years ago. On physical and gynaecological
examination are remarkable. What would be most appropriate treatment?
A. Bupropion
B. Estrogen
C. Sildenafil
D. Tibolone

28- A 42 year-old patient in the second trimester of pregnancy is concerned that her fetus may
have Down syndrome. What is the most appropriate investigation to rule out Down
Syndrome?
A. Triple test
B. Amniocentesis
C. Cordocentesis
D. Chorionic villous biopsy

29- A 17 year-old football player presents with an area macular hypopigmentation on the
trunk and arms. The areas seem even lighter with sun exposure. What is the most appropriate
treatment regimen for this patient?
A. Topical steroid cream
B. Selenium sulphate
C. Oral antibiotics
D. Topical antibiotics

30- A 20 year-old with anaemia, pancytopenia and an enlarged spleen, develops a gradual
deterioration of mental facilities. Bone marrow examination shows the presence of large cells
with a crumpled tissue paper-like cytoplasm. What is the most likely diagnosis?
A. Glycogen storage disease
B. Niemann-Pick's disease
C. Alkaptonuria
D. Gaucher's disease

1c 2b 3b 4b 5d 6c 7b 8c 9a 10 b
11 a 12 a 13 d 14 a 15 d 16 b 17 a 18 d 19 a 20 a
21 a 22 c 23 d 24 a 25 a 26 b 27 c 28 b 29 d 30 d
SMLE 5th of May 2019
1- Female around 22 years has pain on mid plantar line increased withwalking. Dx?
••Plantar fascitis!!!

2- Female patient 25 years with muscle pain increased throughout theday. What to do?
••Ach esterase antibodies!!!

3- Case of diarrhea + dehydration + loss of deep tendon reflexes + flatT wave on ECG
+ muscle weakness. Cause of these muscle and ECGfindings?
••HypoMg!!!
••HypoNa!!!
••HypoCa!!!
••HypoK!!!

4- Case of loin pain, IVU showed non opaque patches on renal pelvis,US showed
hyperdense echoic patches. Dx?
••Renal stone!!!
••Tumor!!!
••Rena papillae atrophy!!!

5- Case of bacterial vaginosis.

6- Case of trichomonas vaginitis.

7- Case of elderly with HCV Hx. US shows hypervascular mass on rightlobe of liver.
Dx?
••HCC!!!
••Cholangiocarcinoma!!!
••Hamartoma!!!

8- Case of elderly with weight loss and obstructive jaundice (high totaland direct
bilirubin) with mild liver enzymes elevation. Dx?
••Para-ampullary carcinoma!!!
••Klessler syndrome!!!

9- Fistual in anus opening the posterior wall. TTT?


••Fistulotomy!!!
••Fistulogram!!!
10- Pt underwent herniorrhapphy. Post operative he developed pusdischarge from
surgical opening and mesh came out of it.Management?
••Wound drainage and mesh removal!!!
••Systemic ABx!!!
••Open the wound!!!

11- Pt with PE, given IV Heparin for 5 days, INT 1.2, Symptomscontrolled. What to do?
••Stop Heparin!!!
••Change to Enoxaparin!!!

12- Inferior MI. What is the post emergency management (post MONAand ECG and
liver enzymes)?
••TPa thromolysis!!!
••PCI with stent!!!
••Heparin!!!

13- Postmenopausal women come to take HRT. You said she didn'tneed it. She
insisted to take it. What to do?
••Prescribe HRT to her!!!
••Don't give it!!!
••Consult Hospital authority!!!
••Refer to another Gynecologist!!!

14- 55 years old female with no past or family history of BC.Mammogram done and
was negative. You will do mammogram every?••1 year!!!
••2 year!!!
••3 year!!!

15- 55 years old male complained of sudden severe headache whilelifting something.
What to do?
••MRI brain and cervical spine!!!
••CT brain and cervical spine!!!
16- Elderly female discovered to have cancer. Her sons said to you
don't tell her she has cancer. What to do?
••Tell pt!!!
••Don't tell pt!!!
••Consult health authority!!!

17- Pregnant 36 weeks with HTN and plasma protein is high. Dx?••Preeclampsia!!!
••Eclampsia!!!
••Gestational HTN!!!

18- 48 years old female came with abnoral uterine bleeding. TVU showssubserous
fibroid and endometrial thickness is 14 mm. Dx?••Fibrosarcoma!!!
••Myosarcoma!!!
••Fibromyoma!!!

19- Female patient diagnosed with retro peritoneal sarcoma. What istrue regarding
sarcoma?
••Transmitted to nearby lymphatics!!!
••Metastasis!!!

20- Newborn of diabetic mother needs to take glucose infusion. Whereto give glucose
through?
••Central venous line!!!
••Peripheral venous line!!!
••Nasogastry!!!
••Orogastric!!!

21- Newly married young female came to police to complain that shehas bitten by her
husband. What is the cause of bite do you think?••Husband culture!!!
••Socioeconomic state!!!
••Man roles over women!!!

22- 74 years old male known case of IHD cane with Abdominal pain. X-
ray shows thumb print sign. Dx?
••Ulcerative colitis!!!
••Crohn disease!!!
••Ischemic colitis!!!
23- Female complaining of vomiting when she goes in airplane. Mostappropriate
antiemetic?
••Ondesterone!!!
••
••

24- 31 years old female did pap smear annually in last 5 years and wasnegative.
Recommend pap smear in future?
••Annually!!!
••Every 2 years!!!
••Every 3 years!!!

25- When you were doing cholecystectomy you found a mass. What todo?
••Cholecystectomy only!!!
••Stop procedure!!!
••Cholecystectomy and remove mass!!!

26- 16 years old male was complaining of dysuria, after 3 days theparents said that his
urine became foul smelling. Gram negative bacilliwas found in urine. What is the
causative?
••Klebsiella!!!
••Proteus marbilaris!!!

27- During ventose instrumental delivery there was an injured tostylomastoid foreamin.
What will happen to baby?
••Incomplete eye closure!!!

28- Croup case. Management?


••O2!!!
••Antibiotics!!!

29- Intussucception case. Management?


••Saline enema!!!
••Nasogastric tube!!!
30- What is the most important symptom to affect the patient inpremenstrual
syndrome?
••Pain!!!
••Depressed mood!!!
••Irritability!!!

31- Best prophylaxis against peripheral atrial disease?


••Programmed aerobic exercise!!!
••Aerobic exercise!!!
••Something exercise!!!

32- Which of the following is true about ASD in a 6 years old boy?
••It will close by 5 years!!!
••Surgery should be done at 2 years!!!

33- Case of PCOS, normal FSH, high LH, testosterone high. Whatinvestigation to do
next?
••Glucose and lipid!!!
••Thyroid function!!

34- Case about 21 years old primigravida with 2 years history ofinfertility now came
with spotting of blood (threatened abortion case).Nex step?
••Confirm pregnancy by BhcG!!!
••Pelvic US!!!
••Ask about cause of infertility!!!

35- Elderly female came with dysuria and frequency. What investigationyou will do?
••Urine analysis!!!
••CT abdomen!!!
••Something invasive!!!

36- 3 weeks old infant with blood drops in diapers noticed 2 times bymother when she
change diapers. Dx?
••Juvenile polyps!!!
••Mickels diverticulum!!!
37- 36 weeks pregnant lady has features of abrupto placenta, severebleeding,
hypotensive, tachycardic and anemia. What to do to safe live?
••ICU admission andmultiteam work!!!
••Insert 2 large bore peripheral cannulas and blood transfusion!!!
••Delivery room!!!

38- Child brought by his parents With meningitis. You need to doemergency LP. What
to do?
••Do LO without consent!!!
••Take consent from parents!!!
••Take consent from parents and child!!!
••Abstent from child, consent from parents!!!
••Abstent from parents, consent from child!!!

39- Asthmatic patient on short acting bronchodilator and low dosesteroid inhalation.
Still complaining of daily symptoms about 5 days perweak and night symptoms about 2
days per weak. What to add? (NoLABA in choices)
••Leukotriene!!!
••Theophylline!!!
••Nothing to add!!!

40- While you are doing a surgery, you made a surgical mistake thenyou corrected it.
What to do?
••Tell patient!!!
••Don't tell patient!!!
••Write report on the case!!!
SMLE 1st of May 2019

1- Baby delivered at home. Dx?


••Hemorrhagic disease of newborn (factor 10 deficiency)!!!

2- Surgeon wii do appendectomy, he opened and found appendixnormal, then he


removed it. What to do with the patient as a doctor?••Tell the patient!!!
••Don't tell the patient!!!

3- Surgeon diagnosed a patient as a case of appendicitis, he decided todo


appendectomy, when he opened the patient, he didn't find theappendix. How to
identify appendix?
••Follow tenia coli!!!
••Follow right colonic artery!!!

4- A Surgeon decided to do resection anastomosis surgery for coloncancer. During


the operation the Surgeon noticed the surgical fieldbecame red (bleeding) and BP
dropped. Surgeon called the vascularSurgeon to interfere. What is the injured artery in
such case?
••

5- A Surgeon decided to do resection anastomosis surgery for coloncancer. He called


the vascular Surgeon to attend cuz he was excpect tohave a bleeding during
operation. During the operation the Surgeonnoticed the surgical field became red
(bleeding) and BP dropped.Surgeon and vascular Surgeon tried to control the
bleeding and theydid packing. Then after finishing the anastomosis, BP dropped
again.What is the next step in management?
••Remove Packing and suture injured vessel!!!
••Clamp aorta proximally!!!
••Clamp aorta distally!!!
••Stabilize patient on table!!!

6- Case scenario of female patient get menopaused (cycles stopped).Then she


developed dysfunctional uterine bleeding for the last 3
months. TVU found endometriosis (Jnadi think this is endometrialthickness not
endometriosis) with thickness about 19 mm. What to offerfor this women?
••Ablation of endometriosis!!!
••Total abdominal hysterectomy!!!
••Laparoscopic hysterectomy!!!
7- Case scenario of female patient menopausal now 58 years old. Shedeveloped
dysfunctional uterine bleeding for the last 2 months. TVUfound multiple endometriosis.
What to offer for this women?
••Ablation of endometriosis!!!
••Total abdominal hysterectomy!!!
••Laparoscopic hysterectomy!!!

8- Female patient diagnosed as a case of ovarian cancer. What is themost important


tumor marker for follow up?
••C125!!!
••AFP!!!

9- Question directly related to Saudi vaccination guidelines, child aged1 years, asked
about what vaccine in this age?
••
••

10- Many Qs about Ulcerative colitis.

11- Many Qs about Crohn disease.

12- Many Qs about biliary diseases.

13- Pt was self dependent in eating, drinking and wearing clothes. Thenhe developed
dementia and some abnormal behaviors and agitations.How to manage?
••Respiredon!!!

14- Q about erythema nodusum and pyoderma gangrenosum of CD andUC, with their
pictures.

15- Child aged 3 years old brought by his mother with episodes ofcrying, fever,
productive cough and drooling of saliva. 1-2 weeks ago
mother reported that her child was complaining of bilateralconjunctivitis. Which of
the following is the most causative organism?••Mycoplasma pneumonia!!!
••Adenovirus!!
••Chlamydia trichomonas pneumonitis!!
16- 36 week pregnant with history of
smoking came complaining of vaginal bleeding with uterine tenderness.Diagnosis?
••Abruptio placenta!!!
••Placenta previa!!!
••Vasa previa!!!
••Placenta accreta!!!

17- Contraindications of ECV?


••Low lying placenta "placentae previa"!!!

18- Young married female came to hospital complaining that herhusband bite her. What
do you think the cause of this problem?••Husband culture!!!
••Husband socioeconomic state!!!
••Husband education ‫!!!تربية‬

19- Surgeon diagnosed a pt with appendicitis then he decided to dosurgical


appendectomy. Intraoperatively he found appendix normal.Then doctor removed the
appendix. What is the most appropriateaction with regards to this decision?
••Tell the patient that Dr found appendix normal and removed itaccording to
guidelines!!!
••Don't tell patient!!!
••Call another Surgeon to tell him!!!
••Inform the ethical communication in Hospital!!!

20- Surrogacy?
••Not allowed, non ethical in KSA!!!

21- HRT?
••Refuse!!!
22-
••Measure cortisol level!!!
••CT brain!!!
••Low dose dexamethasone!!!
••24h!!!

23- Case of male patient with joint and abdominal pain, splenomegaly,not responding
to NSAID. Dx?
••SLE!!!

24- Pregnant women at 15 weeks presented with nausea andheadache. On exam the
BP was high "168/100", otherwise healthy. Dx?••Preeclampsia!!!
••Gestational HTN!!!
••Pregnancy induced HTN!!!

25- Pregnant women at 22 weeks presented with nausea andheadache. On exam the
BP was high "168/100", otherwise healthywithout protein in urine. Dx?
••Preeclampsia!!!
••Gestational HTN!!!
••Pregnancy induced HTN!!!

26- Pregnant women at 22 weeks presented with nausea andheadache. On exam the
BP was high "168/100", otherwise healthywithout protein in urine. TTT?
••Hydralazine!!!
••Methyl dopa!!!
••Nigedipine!!!
•••Nitrates!!!

27- Most common site of colon cancer metastasis?


••Liver!!!
••Lung!!!
••Prostate!!!
••Stomach!!!
28- Surgeon did laparoscopic procedure in the liver for a certainreason. Then 10 days
following procedure the patient presented withGERD like sensation, air or gases
coming out of the mouth. What couldbe the injured structure due laparoscopy?
••Trachea!!!
••Esophagus!!!
••Stomach!!!
••Duodenum!!!

29- Pt has thigh swelling, diagnosed as sarcoma. How to knowmetastasis?


••MRI!!!
••CT!!!
••CT angiogram!!!
••Bone scan!!!

30- Child can sit in tripod position, can move from back to abdomenand from
abdomen to back, can hold things in hand but can't controlthing in his hand between 2
fingers "no pincer grip". Developmentalmilestones age?
••6 months!!!

31- Healthy 35 years old male doing routine investigations thenaccidentally solitary
nodule found on CXR. What to do?
••Follow up after 6 months!!!
••CT!!!
••Take biopsy!!!
••Resection!!!

32- The most important precaution in patient with Pulmonary TB is?


••Airborne infection!!!
••Droplets infection!!!

33- Neoborn delivered at hospital, healthy, but mother reported thatthe previous baby
died due to immunodeficiency. What is the best
action regarding Saudi immunization program?
••Don't give BCG!!!
••Give BCG with close observation!!!
••Refer baby!!!
34- A child has immunodeficiency. Which vaccine is contraindicated?••Varicella!!!
••Pneumococcal!!!
••Influenza!!!
••Injectable polio!!!

35- Pregnant women, her child is in school, she is afraid of flaring, whatnext?
••Take influenza!!!

36- Women planned to be pregnant in winter. What vaccine to take?••Influenza!!!

37- Mother came to hospital with her healthy normal weight 6 weeksold child, she said
that with each feeding the child passing informedstool. What to do?
••Change Milk formula!!!
••Give dehydration solution!!!

38- Child 1 years old on normal formula but passing bloodless diarrhea,vomiting and
abdominal pain since. What to do?
••Change formula to cow free milk!!

39- Child 2 months, cry 1-2 days per week, not sleep well for 1 month.Normal weight.
Otherwise healthy. Dx?
••Infantile colick!!!

40- Male patient with left leg pain and swelling, edematous, non pittingedema, intact
pulse. Most benifit test to confirm diagnosis?
••CT angio!!!
••MRI venography!!!
••Doplex US!!!

41- Elderly male came to ER with SOB and difficulty sleeping at night.He mentioned
Hx of heart surgery few years ago. You did investigationsand ECHO. What is the
most important thing you will be afraid of thatmight kill patient?

••Pulmonary HTN!!!

••Stroke!!!

••PE!!!

••Cardiogenic manifestations!!!
42- Elderly male came with weight loss, headache, RUQ tenderness,LFT all high. Dx?
••HCC!!!

43- A case of acute pancreatitis in which after 10 days from acutepancreatitis the pt
came with epigastric vague pain and fullness. Dx?••Pancreatic pseudocyst!!!

44- Pulsating epigastric mass. Dx?


••AAA!!!

45- AAA. Best to confirm?


••CT!!!
••US!!!
••MRI!!
••X-ray!!!

46- Bilateral reducable uncomplicated inguinal hernias. Pt notcomplaining of any


complications. What to do?
••Lap mesh repair!!!
••Open mesh repair!!!

47- Pt with Normal sodium, low potassium, and has HTN. Dx?••Pheochromocytoma!!!
••Hyperaldosteronism!!!

48- Pt known case of DM and HTN. He has uncontrolled HTN onmultiple medications.
What is the cause of uncontrollable HTN?
••Taking food salts!!!
••Hyponatremia!!!
49- Child diagnosed as a case of nephrotic syndrome. Anyway now theonly complaint
is generalized edema. Management?
••Frusemide!!!

50-A Case scenario of K/C DM pt came complained of fatigue, labshows low


bicarbonate, normal Na and low k. What to give?
••Give bucarb infusion!!!
••Give IV insulin with dextrose!!!
••Give normal fluid!!!

51- A case scenario of patient K/C of non hodgkin lymphoma came withfeathers of
tumor lysis syndrome. What electrolytes abnormalities youwill find?
••HypoCa!!!
••HyperCa!!!

52- A 12 years old girl who is a known case of DM 1 diagnosed 2 yearsago. Now came
to do routine checkup. She has no features of puberty.What test to do annually?
••Ophthalmology!!!
(Should be done after 5 years from Dx of type 1 DM then annually)
.••GH assay!!!

53- A young female complaining of abdominal pain that increased withcycles, stay for 5
days, first 2 days of cycle she missed school duesevere pain (dysmenorrhea). First next
step in TTT?
••NSAID!!!

54- Women with PCOS, not planning to get pregnant soon. TTT?••Metformin!!!
••COCP!!!
••Progesterone only pills!!!

55- 33 years old female morbidly obese want to do surgical weight lossoperation. What
investigation you should do before surgery?
••Abdominal US!!!
••CT!!!
••MRI!!
••C-ray!!!
56- Pt with all the viral hepatitis markers negative except positive HBsurface
antibodies. Dx?
••Previous immunization!!!
••Acute resolved infection!!!

57- What is the contraindication of liver transplantation?


••End stage liver disease with ascites!!!
••End stage liver disease with encephalopathy!!!
••Acute liver cirrhosis!!!
••Alcoholic liver cirrhosis!!!

58- Contraindicated vaccine in pregnancy?


••HPV!!!
••Zoster!!!
••Pneumococcal!!!
••Influenza!!!

59- 1 year old child not given any vaccine before because motherrefused due fear of
harm to her child. What to do?
••Talk with parents about benefits and side effects of vaccines!!!

60- What is the most common organism to cause Post prosthetic valve
(following valve replacement) endocarditis?
••Staph epidermedis!!!
••Staph aureus!!!
•••Strept viridans!!!

61- Case scenario of patient complaining of SOB, with Severe MR +mild LVH TTT?
••Mitral valve replacement!!!
••Give medication!!!

62- Case scenario of patient with syncopy and SOB. Dx?


••Aortic stenosis!!!
63- When to do surgery for child with congenital atrial septal defect?
••Certain size!!!
••Certain site!!!

64- A mother brought her child to hospital. Surgeon discovered herniaand decided to
do surgery immediately. Mother refused. What to do?••Do surgery whatever mother
decision!!!
••Inform hospital police!!!
••Don't do surgery!!!

65- Rheumatoid arthritis patient with multiple joints pain, given NSAIDbut not
responding, best drug?
••Methotrexate!!!

66- Infliximab or Retuximab (Rheumatoid arthritis).

67- Case of child with skin rash in lower limbs, ankle and knee jointspain, fever. Dx?
••HSP vasculitis!!!

68- Case of abdominal pain, bloody diarrhea, Hb low. Dx?


••HUS!!!

69- Low Hb, MCV low, RDW high. Dx?


••IDA!!!
••Aplastic anemia!!!
••Thalassemia!!!

70- Low Hb, high MCV. Dx?


••Folic acid, B12 deficiency anemia!!!

71- Pt with recurrent chest infections and low platelets. Family historyof similar
condition. Dx?
••Weskot Aldrich Syndrome!!!
◼Case: old age with obstructive jaundice( increase total, direct bilirubin, mild
increase in liver enzymes) associated with weight loss

➖Para ampullary carcinoma

➖Cristlar syndrome
‫مش فاكره الباقي‬

◼Anal fistula open in post wall, managment

➖Fistulotomy

➖Fistulogram
‫مش فاكره الباقي بردو‬

◼Patient after herniorhaphy complain of pus discharge at the opening of


operation and the mesh come out, Managment?

➖Wound drainge and mesh removal

➖Open wound

➖Systemic antibiotic
◼Patient with PE(pulmonary embolisn) with controlled sypmtoms on IV heparin
for 5days INR 1.2 What to do?

➖Stop heparin

➖Change to enoxaprin

‫➖في حاجتين تانين استبعدتهم‬

◼Inf wall MI, Post emergent management?

➖Thrombolysis by TPA

➖PCI with stent

➖Heparin

◼Female patient with pain in mid planter line increase with walking

➖PLANTER FASCIITIS ‫مش فاكره باقي الخيارات‬

◼Female 25years with pain in muscles mainly increase through day

➖‫ اللي فاكراه‬Antibodies against cholinestrase


◼ Case of Diarrhea, dehydration, lead to muscle weakness, loss of reflexes
specially deep, flat T wave in ECG what is the cause of these symptoms?

➖Hyponatremia

➖Hypokalemia

➖Hypocalcemia

➖Hypomagnesemia

◼Case of loin pain IV urography show non opaque patches in the renal pelvis
US shows hyperdense echoic patches

➖Ureteric stones

➖Renal papillae atrophy

➖Tumor

◼ Case of Bacterial vaginosis

◼Trichomonus

◼Case: old age HCV US shows hyper vascular mass on right lobe of liver,
Diagnosis:

➖HCC

➖Hamartoma

➖Cholangiocarcinoma
◼Female post menopausal come to take HRT but u said she dont need but she
insisted on taking it what to do:

➖Refer to another gynacologist

➖Predcribe HRT to her

➖Dont prescribe

➖Consult hospital committee

◼3wks infant mother found drops of blood twice in his diaper,

➖Mechels diverticulum

➖Juvenile polyp

◼Abruptio placenta with sever bleeding, 36 wks


hypotension, tachycardia, anaemia
What to do to save life?

➖ICU admition with multi team work

➖2 large pores and blood transfusion

➖Admit to ICU
◼Child with meningitis come to ER with his parents you need to do CSF ananlysis
emergently what to do?

➖Do without consent

➖Consent from parents,abistent from child

➖Consent from child and abistant from parent

➖consent from both

◼Old age female with adenexl mass in US 5*5


What to do next?

➖Surgical excision

➖Refer for gynacological work up

➖Cystectomy

◼Asthmatic patient on short acting bronchodilator and low dose steroid


inhalation but still have daily symptoms 5days /wk and night symptoms 2days/wk
what to do?

➖Leukotriene inhibitors

➖Theophylline

➖No thingto add


◼Newborn of diabetic mother need infusion of glucose
how to take?

➖Peripheral venous line

➖Central venous line

➖Nasogastric tube

➖Orogastric tube

◼Recently married woman beaten by her husband and went to police to


complain
What is the cause?

➖Previous culture

➖Socio economic state

➖Thoughts of men roles over women

◼74 yrs IHD patient with abdominal pain x ray shows thumb prints

➖Ischemic colitis

➖Ulcerative colitis

➖Crohns
◼Female complains of vomiting in flights
Most appropriate antiemetic?

➖Ondansteron
‫مش فاكره باقي الخيارات‬
◼31 yrs female pap smear is _ve in the last 5 years what do u recommed for the
next pap smear

➖Annually

➖Every 2 years

➖Every 3years

◼55 yrs female no hx of breast cancer, _ve mammogrm should do mammogram


every?

➖1 year

➖2 years

➖3 years

◼Best prophylaxis against peripheral atrial(not arterial) disease

➖Programmed aerobic exercise

➖Aerobic exercise
‫ مش فاكره باقي الخيارات بس كلها‬exercise
◼Case with ASD secondum what is true about ASD?

➖Close spontaneously by 5 years

➖Need surgery at 2 years


‫مش فاكره باقي الخيارات‬

◼PCO case with normal FSH, high LH, testosterone


Further investigations?

➖TSH, T3, T4

➖Glucose tolerance,lipid profile


‫مش فاكره باقي الخيارات‬

◼21 yrs primigravida after 2 years of infertility complaining of spotting of blood


What to do?

➖Ask about cause of infertility

➖Pelvic US

➖confirm pregnancy 1st


◼Old female complaining of dysuria, frequency
Investigations?

➖Urine analysis

➖CT abdomen

Invasive investigation‫➖مش فاكراها‬

◼55 yrs male complainig of sudden sever headache while lefting something.

➖CT brain, cx spine

➖MRI

◼Elderly with cancer her sons asked u not to tell their mother
What to do?

➖Tell patient

➖Not tell her

➖Consult the health committee

◼Pregnant 36wks HTN with high plasma proteins


diagnosis?

➖Pre eclampsia

➖Eclampsia

➖Gestational HTN
◼During cholecystectomy u find a mass what to do?

➖Cholecystectomy only

➖Cholecystectomy and remove the mass

➖Stop the procedure

◼16 yrs male complaining of dysuria after 3 days urine has foul smelling, urine
culture shows gram negative bacilli
What is the organism?

➖Proteus mirabilis

➖Klebsilla
‫تانين مش فاكراهم‬2 ‫في‬

◼During delivery with ventouse injury of the sternomastoid foramen


What u find?

➖Incomplete closure of the eye

◼Croup case ttt

➖Antibiotic

➖Oxygen
‫مش فاكره الباقي‬
◼Intussusception, currrnt jelly stool
Managment?

➖Saline enema

➖Nasogastric tube
‫اجابتين تانين مش فاكراهم‬

◼Pre menstrual syndrome, which symptom mostly affect the patient?

➖Depressing mood

➖Irritability

➖Pain

◼48yrs female with abnormal uterine bleeding US shows subserous fibroid,


endometrial thickness 14mm
Diagnosis?

➖Fibromyoma

➖Fibrosarcoma

➖Myosarcoma

◼Female has retroperitoneal sarcoma


Which is true about sarcoma?

➖Transmit to near organs via lymphatics

‫➖مش فاكرة باقي االختيارات‬


◼During surgery u did a mistake and corrected it during the operation

➖Tell the patient

➖Not tell the patient

➖write a report about the case

Diapetic pt complains of fatigue , lab invist : hco3 decreased , k decreases , normal


Na - what will do ?
- give hco3 infusion
- iv insulin dextrose
- normal fluid ( normal saline )

Tumor lysis syndrom case ( pt with non hodgking lymphoma what is the meniral
abnormality)
- hypocalcemia
- hypokalemia.
- hypernatremia
12 yo child with DM1 , come for routine check up , she came 3 months ago for
follow up , has no any signs of puberty - which test she should do annually?
- ophthalmology
- growth hormone

Female complains of abdominal pain, pain increase before menarche and last for
3-5 days whats the first step u will do ?
- NSAID

Pt diagnosed with PCO , she is not paln to be pregnant , which drug she should
take ?
- cocp
- metformin
- progestron only pills

36 yo female patient with morbid obesity she is going to do briatric surgey , what
invistigations she should do be the procedure? ;
( ‫) وش الشي اللي الزم تتاكد منه قبل تدخل العمليه‬
-abdominal us
- ct
- mri
- x ray
Surgeon doing resection for colon cancer and the cardiovascular doctor with him
trying to stop bleading by packing the injury , then the pt became a stable , after
while the blood pressure decreased ( hypotension) , what they should do !!! ( not
clear )
- on table
- remove the packing and suturing the injury
- doing clamp ( ‫) يعملها باالورتا خيارين مختلفين واحد من فوق والخيار الثاني من تحت‬

Arthritis: pt with multiple joints pain he is on NSAID but with no benefit,what is


the drug u will give ?
- methotrexate

Pt with joint pain ( he is on some medication ) she chose give him influximab

Child with ankle and knee joint pain +fever +rash in the lower extremities
- vesiculitis ( henoch ! )

HUS : abdominal pain , tender , has organomegaly ( not sure what ) , bloody
diarrhea, had an infection before ,
Invist : hg was low ,
Child with fatigue and lw concentration , lab results: Hg decreased,mcv dec , RDW
high
- IDA
- aplastic
- thalassemia

Pt with macrocetic RBC , no more detalis !! What will u give ?


- folic acid

- wiskott syndrom ( recurrent infection - family hx +ev - his brother and aunt)

Contraindication of liver transplantation ?


- acute liver cirrhosis
- liver cirrhosis +alcoholic pt
- end satge liver +asitice
- end stage liver +encephalopathy

**2 Ethic q were repeated


Pregnant vaccination : contraindication ?
- zoster
- h influnza
- hpv

Baby with hernia ! The doc said we must do it immediately, but her mother
refused .. what u will do ?
- ignore the mother decision
- eithic committe
- police ??

Young 21 years old wife came complaining that her husband has bite her hand,
what is the reason behind this abuse?
1. Socio-economic status of husband.
2. Cultural background
Question was confusing
Surgeon done appendectomy, however during the surgery find out that the
appendix is normal and removed it, what does the surgeon suppose to do now?
1. Tell the patient that the appendix was normal only.
2. Make another surgeon inform the patient.
3. Inform health communication workers
4. Tell the patient that you followed the protocol that all surgeons and institutions
follow in these situations.

Old postmenopausal women asking you to give her Hormonal therapy?


1. Refuse to give her.

Female with symptoms of Cushing Syndrome, what to do?


1. Brain CT.
2. Give dexamethasone and observe response.
3. 24 hour collecting urine test.
4. Measure cortisol levels in blood.
Male with Joint and abdominal pain, splenomegaly, and not responding to
NSAIDs, what is the DX?

Male with left leg pain, swelling (non-pitting), pulse intact, what is the most
beneficial to make the diagnosis?
1. CT angio
2. Duplex US
3. MRV

Elderly male present at night to ER with dyspnea, chest tightness, you did all
investigation including Echo, he did Cardiac surgery before, what is reason that
might cause death to this patient?
1. PE
2. Stroke
3. Pulmonary HTN
1. Pregnant women with strong indication for CS, she refuse the CS and want to
deliver vaginally, What to do?
A. Do general anesthesia and CS.
B. Take consent from husband regarding CS.
C. Try vaginal delivery.
D. Forget this option
The q was confusing.

2. 5 cases about COPD and ASTMA patients with severe acute attacks asking
about management which is if
- Comatose —> mechanical ventilation
- Non-comatose —> noninvasive ventilation
3. Pregnant in 2nd trimester diagnosed to have hyperthyroidism and prescribed
to hyperthyroidism medications, however refuse to take any medications, what to
do?
- Force her to take the medications.
- Talk to her husband.
- Ignore her.
- Explain to her the consequences of untreated hyperthyroidism on her fetus.

1)case / delivery at home(not clear)


why at home ? DX ? :
_neonatal hemorragic disease*
_factorX
4)Case / ca resection and general-surgeon ask for vascular-surgeon to assist
him..he did anastomosis ..then pt collaps e severe hypotension.what to do ??
---------------
5)case /post-menopausal lady . co bleeding since 3 or4 months .US found
endometriosis 19mm.what to do ?
-partial ablation
-abdominal hysterectomy
-endoscopic hysterectomy ?
----------------
6)another case of post-menopausal lady . co bleeding and US shows multiple
endometrisis.
mx ?
---------------------
7)case /ovarian cancer . witch tumor marker ??
-alpha-fetoprotein
-C125
------------------
8)case/ 1 yo vaccinatin schedule ?
------------------
9)case /old age with dementia , agitation , ttt ?
-risperidone
- ….
----------------
10)case /ulcerative colitis
----------------
11)case /crohn . extra intestinal manifestations of crohn disease :
-erythema mindosa
-hepatiform gangren
----------------
12)Case/mother of 3mo baby she tell her son crying e drowling saliva ,pain ,
produtive cough , H/O bilateral conjonctivitis 2weeks ago , and
pt has some problem in his lungs..dx
-adenovirus
-mycoplasma
-clamydia trichomonas pneumonitis

Case of hepatitis B virus :


All labs finding are negative except anti HBsAg :
Immunization
Post infection
Acute infection
Chronic infection
_______________
Case of GIT :
Old age male with severe loos of weight, headache, tenderness on RUQ, liver
enzymes and ALP are very ELEVATED what's the dx?
Hepatocellular carcinoma

_______________

Case of pancreas :
After performing a resection surgery he complains of gastric pain, U/S showed
collection of fluids or cyst what's the dx?
Psudopancriatic cyst

________________

Pt. With pulsatile feeling above the umbilicus ( epigastric region)


Whats the dx?
AAA or ruptured aortic aneurysm
How to confirm the dx?
CT
MRI
U/S
CXR
adult with bilatral inguinal hernia, reducible, no complains rather than swelling
seen at cough
What's the managment?
Laparoscopic repairs ( mesh)
Surgical repair

Observation
________________
Ask about minerals that increase or decrease in different metabolic and
endocrinal diseases.
Hypertention

Case of pt. With hypotension with normal Na and decreased potassium


What's the dx?

Hyperaldosterinism
Pheochromocyroma
‫مش فاكره الباقي‬
_________________

Pt, with DM and HTN uncontrolled on many classes of different medication


What's the etiology of being uncontrolled?

Decreased Na
Using salts in dite
_________________

Child with post streptococcus GN complain from edeoma, protienuria +3


What's the management?
Frusemide and ‫مش فاكره الباقي‬

________________

Child 2 years old with ASD


indication of ASD repair?
Location
Size
‫مش فاكره الباقي‬
Pt. With mitral valve prolapse and he will perform valve replacement
What's the comments organism could affect this valve after replacement?

Staphylococcus
Streptococcus viridance
‫مش فاكره الباقي‬

________________

A case with recurrent attacks of syncope


What's the dx?
Aortic stenosis

_________________

A case with severe Mr and mild LVH


What's the ttt?
Mitral valve replacement. ????
Pregnant lady in 15 week , completely healthy, came with headache and nausea
no other complaints , when her blood pressure measured it was 169/100 , so her
condition is :
A) preeclampsia
B) pregnancy induced hypertension
C) gestational hypertension
D- pre existing hypertention
___________
Pregnant lady in 22 week, she had hypertension but without proteinurea (
gestational HTN) , which medication supposed to take?
A) methyldopa
B) Hydralazine
C ) Nifidipine
—————————————
First and most common site of metastasis in colon cancer:
A) liver
B) Lung
C) prostate
D) gastric
Patient did laparoscopic liver procedure after 10 days or one week came and
complain of heartburn, what is could be injured here ?
A) trachea
B) gastric
C) esophagus
D) duodenum
———————————————
Patient came with swelling in his thigh diagnosed as sarcoma , what to do to
follow the metastasis?
A) Bone scan
B ) MRI
C) MMR
D) CT or CT angio
————————-
Alot of Qs from Saudi vaccination schedule
——————————-
Milestone Qs
35 Y/ O male patient, healthy, came to do routine investigations , chest xray
shows small size solitary nodule in the lung so what to do?:
A ) take biopsy
B ) follow up after 6 months
C ) CT
D ) Resection
—————————————-
Proper precaution in TB patient:
A) Airborne
B ) droplets infection
——————-/—————
Mother delivered healthy baby , there’s hx of death of his immunodeficiency
brother,what to do regarding the Saudi vaccination schedule ?
A) give BCG and follow the baby
B) don’t give BCG
C ) refer to neonatal department
__________
Contraindicated vaccines in immunodeficiency child ?
A ) pneumococcal
B) vericella
C) influenza vaccine
D ) injectable polio
Pregnant lady, her boy in school, afraid of flaring, suppose to take ?
Influenza vaccine
—————————

Female plan to be pregnant in winter ❄


Which vaccine supposed to take?
Influenza vaccine
——————————————-
Mother came & complain that her 6 weeks baby boy after each feeding develops
unformed stool ? What to do ??
A) change milk formula
B) give dehydration solutions
———————-
Cow free milk, 1 y/o baby on norml formula milk , pass bloodless diarrhea,
vomiting and abdominal pain?
A ) change the formula to cow free milk
—————-
Mother bring her 2 months baby boy, healthy, and complain that he is crying
during sleeping and sometimes couldn’t sleep well, this occurs 2 times per week
for 2 months, what’s could be the cause?
A ) infantile colic
1. Pregnant women with strong indication for CS, she refuse the CS and want to
deliver vaginally, What to do?

A. Do general anesthesia and CS.


B. Take consent from husband regarding CS.
C. Try vaginal delivery.
D. Forget this option
The q was confusing.

2. 5 cases about COPD and ASTMA patients with severe acute attacks asking
about management which is if
- Comatose —> mechanical ventilation
- Non-comatose —> noninvasive ventilation
3. Pregnant in 2nd trimester diagnosed to have hyperthyroidism and prescribed
to hyperthyroidism medications, however refuse to take any medications, what to
do?

- Force her to take the medications.


- Talk to her husband.
- Ignore her.
- Explain to her the consequences of untreated hyperthyroidism on her fetus.
DKA pation came ti Hospital , insulin and fulid given in hospital
Pation lab hyponatrima
What the cause
A-insulin
B - fliud
C- notrin antidiurtic hormone syndrom

Patient go to opertation Rome the surgeon said should be done ambutation the
pation refuse
What should to do
A- do the surgery
B- till the hospital etihi communite
C - make the pation DAMA

Patin asthma and pregnant on delivery what is CI ??


Mitorlopolo
Chronic pressure
Gastisional precclamcy

Chest xary for baby


( glass appertains )

Newborn has family sisters day fir immunodeficiency what is should to do


:‫تعمله اختبار للمناعه‬
‫وال تعمله تاخذ عينه وتعطيه اللتطعيم وبعدين تستنى التحليل يطلع‬
‫وال ما تعطيه تطيعم الين تظهر النتيجه اول‬

‫ الفرق بين‬DI central and ne and


nephrology
9 wks gastional mid vaginal bleediny cervical os closed

Bilateral. Inguinal hernia


Surgery >>>
A-open repair
B - open with mish repair
C- laparoscopy
D- simple repair
Surgrical lateral meatue

Pregnant with heavy VB what do for her to safe her life


A . Blood tansfustion
B- us abd
C- call obstatric
D - transfer her to osbstatric department

Baby can trun head cant set without supporting


A-4 month
B-3moths
6-months

Most commn site of diverticulitis !!


Iullm iluucecal
sigmoid
Corn syndrom
Cushing syndrom
Wipple diseases
DM type 2 what u see in urin

38 wks gastional effaent 50٪


Biophoso scor 8 cervical dilation 4
No ROM all this was 4 hours
What have to do
A oxytocin
B cs
C what 2 hours

When we give RoHM for baby


When mother rh - baby rh+
Or opposite
Male pt 35 healthy with regular visit, after doing CXR dr found small solitary nodule , what you do ?

1- Take biopsy
2- Follow up after 6 month
3- Chest CT
4- Restriction

Pulmonary TB after caution:

1- Air bourn
2- Droplet

Newborn of mother with hxafraid because last her baby dead by immunedeficinecy what you must to
do by Saudi schedule immunization ?

1- Don’t give him BCG


2- Give him BCG and follow up

Newborn immune deficiency what the vaccine contraindication for him ?

1- Varicella
2- Pneumococcal
3- H.influenza
4- Injectable polio

Pregnant woman going to school for her visit her child , she afraid to inflame what that vaccine can to
give her ?

1- H.influenza

Woman planning to pregnant at winter weather , what that vaccine to give her ?

1-H.influenza

Mother came to ER with her neonate 6 weeks c/o in front stool ( not compatible with taking breast
feeding ) , what to do ?

1- Give him milk formula


2- Dehydration solution

Child one year old came to ER with her mother c/o diarrhea with less blood and vomiting and
abdominal pain because he drink normal formula , what to do ?

1-change it to cow free milk

Child 2 month and mother saying her child always he isn’t sleep at night and crying , his well as weight
, what the cause ?
1- Infantile colic

Male pt c/o left leg pain and swelling and edematous non pitting the pulse is intact , what is must
beneficial investigation to confirm your diagnosis for him ?

1- CT angio
2- Doplex
3- MRV

Elderly pt came to ER c/o SOB and dyspnea /PND and yoydoing Initial assessment for him , also there hx
of heart surgery , what most thing is concern about him ?

1- Pulmonary edema
2- Pulmonary hypertension
3- Stroke

Elderly pt came to ER c/o weight loss and headache and RUQ tenderness , investigation shows Alpha
feto protein is high , also Liver enzyme is very high , what the diagnosis ?

1- HCC

Elderly pt when doing laparoscopy they found a cyst around the pancreas , what the diagnosis ?

1- Pancreatic pseduocyst

Pulsatile feeling above umbilical region , what the diagnosis ?

1- Aortic aneurysm

Pulsatile feeling and mass above umbilical region , what the investigation that confirm the diagnosis ?

1- Abdominal U/S
2- Abdominal xray
3- MRI

Adult patient came to clinic with Hernia bilateral reducible without pain and increase when he
coughing , what you do ?

1- Surgical Mish repair


2- Reassurance
3- Laparoscopic repair

Case of pt minerals disturbing normal NA low K , 1hat the diagnosis ?

1- Pheochromocytoma
2- Hyper aldosterone
HTN pt and also DM he is uncontrolled of HTN , he is take many medication , what the reason of his
uncontrolled ? hypo NA

1- Because he’s used the salt

Child came to ER with protein in urine +++ and generalized edema and hypertension diagnosed as post
sterptocaccal glomerulonephritis , what the treatment ?

1-fureosmide

Surgeon doing restriction for cancer mass , anastomosis and he call the vascular surgeon for bleeding
control , the case under control , whole minute the blood pressure fall down , what you do ?

1- Stabilize the pt on table


2- Re opening and suture the injury
3- Clap above the injury on aorta
4- Clap down the injury on aorta

Ethic question : you doing operation for transplantation of kidney and you forget what that kidney left
or right , what you do ?

1-tell the surgeon your concern

Mother came to your clinic with her child 2 year age , don’t receive any vaccine , because the mother
afraid from the vaccine , what you do ?

1-talk with her about the benefit from vaccine

What the contraindication of liver transplantation ?

1- Acute liver cirrhosis


2- Liver cirrhosis with alcohol
3- End stage liver with ascites
4- End stage liver with encephalopathy

Mother came to ER with her child and surgeon discover there is hernia , and the surgeon decide
operation emergency ( lifesaving ) , and mother refuse it what to do ?

1- Do operation without consent from mother


2- Call the police to take the permission
3- Call ethic hospital committee
Contraindication vaccine to pregnant woman ?

1- Herpes zoster
2- Pneumococcal
3- H.influenza
4- HPV

About Hepatitis B serology , AB of Surface + , what the diagnosis ?

1- Natural Immunization
2- Infection acute
3- Chronic infection
4- Vaccinated

When doing operation for child with ASD ?

Picture with HSP , with petechial rash and bleeding with hx of respiratory infection , doing with LP
result is high protein and low glucose , he is close contact with his big brother 6 year , but the mother
concern about that boy 6 year prophylaxis , what to do ?

1- Oral rifampicin

CHF stable on statin and aspirin furosmide what to add ?

1- Enalprail
2- Spironlactone
3- Propanolol

Question about allowing of surrogacy

Question about pt sensitive to drug prescribed by nephrologists , what you do ?

Dm pt want to do hernia ( reducible ) operation , he threatened the doctor if he not do it he well


complain , with no contraindication what you do ?

1- Do the operation
2- Call the hospital administration
3- Ignore him

Male came to regular follow up the doctor tell him , he discover a lung cancer , but patient refuse that
result , what the act of pt ?

1- Denial
2- Anger
Pregnant woman came to ER 36 week after deliver the doctor tell is her baby dead , she refusing that
and say to doctor to do resuscitate , what the act of pt ?
15YO , his mother said he is depressed
1- his
For 3month because Denial
friend die and he is not in his normal ordinary state, and tell her he wish the dead but never act or blan to die what to do?

Talk to him about


Childsducidal
15 yearplan
camewill to
increase risk ofhis
clinic with sucid ideation
mother c/oand attemptfor
sadness . 3 month because his best friend dead , and

he saying wish to die but he not doing anything just saying , what you do ?
Ignore and dont talk to hom at all .
1- Take detail about sadness
Talk to hin with details will protect him.
2- Natural sadness ( bereavement ) let him alone
This is normal greave and let him alone.
Mother came to you to do operation for her baby c/o inguinal hernia bilateral reducible ,what to do ?

1- Herinotomy
2- Open mish repair

Sclerosing injection dye for ?

1- External hemorrhoids
2- Internal hemorrhoids
3- Thrombsed pill
4- Anal fistula

Anal fistula pain at ?

1- 6 clock
2- 3 clock

Pain at 6 clock what the diagnosis ?

1- Anal fissure
2- Anal fistula

Woman with past medical hx 4 NVD / at labour with infant wt 2.9 kg and normal presentation , fully
dilated , the doctor palpitate the cord at cervix , what you do ?

1- c/s
2- deliver by instrument
3- oxytocin

woman 29 weeks GA came to regular ANC visit with closed cervix , with contraction no Ruption
membrane variable deceleration, what you do ?

1- oxytocin
2- tocolytic ( terbutaline )
3- Dexamethasone
4- AB
pt with dysmenorrheal pain and menorrhagia suspension as endometriosis , how to confirm ?

1- Hysteroscopy
2- Colpscopy

How to diagnosis pt with COPD

Elderly pt came to ER with his son drowsy severe productive cough ( lap shows Respiratory Acidosis )
oxygen saturation 87% , many days he can't sleep and very tired . what you do ?

1- Nebulizer ( Oxygen musk )


2- Mechanical ventilation
3- Endotracheal intubation

What the management of moderate attack of asthma ?

1- Inhaled SABA
2- Inhaled SABA + ICS
3- LABA
4- LABA +ICS

Child came to ER with his mother c/o of attack of asthma 3 times night per week , there is history
coming to ER twice last month with 2 attacks , he on ventoline , what you add ?

Pt diagnosed as bronchiectasis he on steroid , what you do after ?

1- Special position
2- Postural drainage
3- Chest physiotherapy

X gamma globinemia ?

Child came to clinic with his mother c/o Multiple café latu lesion on his back also that lesion with some
member of family , what to do ?

1- Send him to genetic counseling

Child came to clinic with his mother c/o Multiple café latu lesion on his back also that lesion with some
member of family , what the mode of inheritance ?

1- Autosomal dominant

Picture of woman face ( colosma )

Lesion ( condyloma lata ) at inguinal area hyperpigmented


Woman came to clinic c/o left hand erythmatous pain , fever , hx of knife wound few days , what the
diagnosis ?

1- Carbuncle
2- Cellulites
3- Necrotizing fascia

Child c/o hematuria , fever with Picture of X-ray of cystourethrogram ?

1- Vesicoureteral reflux

child came to ER with respiratory symptoms last day ?

1- IgA Nephropathy

2- Post streptococcal glomerularnephritis

What the pathophysiology in glomerular nephritis ?

1- Cresentic formation shape

JRA

What the management of woman with lupus nephritis ?

1- Cyclophosphmide + steroid
2- Cyclophosphmide + methotrexate

Woman came to ER c/o proximal joint pain + swelling + morning stiffness , RF –ve , what the diagnosis ?

1- Osteoarthritis of hand
2- SLE
3- SERO –ve rheumatoid arthritis

Oseophageal CA pt came to ER c/o dysphagia start from solid then liquid , what the best invest?

1- Upper GI Endoscopy

Child with celiac disease he well until came to ER after eating something c/o diarrhea , dehydration ,
postprandial pain , mother give him the prescription of last visit ( ORS ) after that he develop the
symptoms , what you will advice her ?

1- Return to normal diet


2- Food with gluten free
Child with cardiac problem ( I think its ASD ) and need to operation , what the best time to do ?

1- Immediately after birth


2- From 3-5 years age

Pregnant woman GA 15 wks came to regular ANC the Blood pressure 169/100 she is healthy , what the
diagnosis ?

1- Eclampsia
2- Gestional HTN
3- Pregnancy induce HTN if there is chronic HTN choose it
4- Some syndrome

Pregnant woman GA 20 wks came to regular ANC the Blood pressure 169/100 she is healthy , what the
management ?

1- Methyldopa
2- Hydralizine
3- Nifedipine

Most common site of colon cancer ?

1- Liver
2- Lung
3- Prostate
4- Gastric

Pt do operation by laparoscopy , with opictfation on liver after 10 that develop what a like GERD
regurge and vomiting blood , in operation the surgeon accidently injured him there is air come by his
mouth , what the organ injured ?

1- Oseophegus
2- Gastric
3- Duodenum
4- Trachea

Swelling in thigh , they diagnosis him as sarcoma , how you will follow up ?

1- MRI
2- CT angio
3- Bone scan

CT if there choose it **
Baby his swings between his back and his stomach but he can't catch thing between his finger ( finger
brence ) , what his age ?

1- 6 month
2- 4 month
3- 3 month

Dm pt came c/o fatigue , he is control lab shows bicarb low K low NA normal level , what you do ?

1- Bicarb infusion
2- IV dextrose +Insulin
3- Normal fluid

Tumor lysis syndrome , all mineral high , what mineral will abnormality ?

1- Hypo Calcemia

Girl 12 year old diagnosed as DM type 1 recently , came to regular follow up , last follow up 3 month
ago , doctor notice no signs of puberty , what the test you do ?
!!!!‫عليه خالف‬
1- Growth hormone assay
2- Ophthalmic refer ( 3-5 year first time of discover )

Girl came with dysmenorrheal + abdominal pain pre menstrual and continue until 5 days of cycle , she
absent 2 days from school because severe pain , what the first medication to give ?

1- NASID

WOMAN diagnosis as PCO , and planning to NOT get pregnancy , what you do ?

1- COC
2- Metoformin
3- Progesterone

Morbid obese 30 yrs lady came to operation decrease of weight , what the investigation to check ?

1- Abdominal U/S
2- X-RAY
3- MRI
4- CT

Child came c/o joint pain , skin rash in lower limb , fever ?

1- Vacuities
Abdominal pain , with FLU , organomegly ( spleen or liver ) hg low , bloody diarrhea ?

1- HUS

Child with low HG MCV low ( 60 ) RDW high , fatigue , low concentrated ?

1- Aplastic anaemia
2- IDA

Pt with cbc shows Macrocytic cell MCV more than 100,how to treat ?

1- Folic acid ( no choice of B12 )

Wiskott Aldrich syndrome , chest infection for child , with hx of all male family member , what the
diagnosis ?

Most organism came to valve , and doing replacement ?

1- Staph
2- Strepto viridance

Severe mitral reguge , left vent. Hypertrophy , c/o SOB , what you do ?

1- Replacement of valve

Pt c/o syncope , what diagnosis ?

1- Aortic stenosis

DM type1 , FBS283 , after lunch 95 , 3pm 184


The difference between the Somogyi effect and the dawn
Increase short acting insulin
phenomenon is that the Somogyi effect is a response to low blood
sugar during the night. Testing blood sugar levels at 3:00 a.m. and
Decrease short acting insulin again in the morning can help distinguish between the types of
changes.
Increase long acting insulin
Blood sugar that is low at 3:00 a.m. indicates the Somogyi effect,
while high or normal levels at that time suggest that the dawn
Decrease long acting insulin
phenomenon is causing high morning blood sugar.

1- Female patient has pain on mid

Planter line increases e walking

A-planter fasciitis

2- female patient 2 symptoms with pain increases through the day:


DM type1 , FBS283 , after lunch 95 , 3pm 184 Increase short acting insulin
Decrease short acting insulin
Increase long acting insulin
Decrease long acting insulin

explanation for Q
- Abs against choli??

3- came with diarrhea associated with dehydration leading to weakness , flat T wave on ecg. What the
cause?

- hyponatremia

- hypokalemia

- hypomag

- hypocalcemia

4- came with loin pain on IVU, non opaque patche on

Us hyperdence echo

5- bacterial vaginosis

6- truchimonas

7- old age came with hx of hepatitis c , on US

8- came with obs jaundice associated with wt loss

- parabilliary carcinoma

- k..... syndrome

9- fistula on anal opening ,....... what management?

Fistiolotomy

Fistulogram

11- pt with pe controlled symptoms was on iv heparin for 5 days. Inr 1.2 what to do?

- stop heparin

- change to enoxaprin
12- inferior MI , post emergent management?

- thrombolytics

- pci with stent

- heparin

١٠-Pt e herniotomy c/o of pus discharge from Mish come out of it

A-wound drainage and mish removal

B- open wound

C-systemic Abx

Pt diagnosed e retroprotneal cancer what TNM about

A-tansmition to lymphatic

B-metastasis

C-‫مش فاكره الباقي‬

‫ وانا شايفه انو‬a‫و‬b ‫نفس الفهم‬

Croup case mangment

Abx

O2

I think the 2not mangment of croup

But o2 may be !!

Intussuption mangment ?!
Saline enema

nasogastric tube

Also the 2option not right

I.v fluid and surgery

Parmenstal syndrom➡irritability

.....HCV in KSA

Dr wrong applied forcep during vaginal delivary crush of sternomastoid foramen and its content what
you expected to find in this baby :

a. Unable to close the eye ✔


no confirmed answer
b. Loss of taste in ant 2/3 of tongue

c. Unable to open the eye d. Sensation in face lost

‫قريب للسؤال‬

Fascial palsy

16y boy come e dysuria then after 3day develop foul smelling what the organism

A-porteus IF THERE IS E COLI CHOOSE IT

B-klebsella

C-

While make cholecystectomy you find mass what will do

A-cholecystectomy only
B-cholecystectomy and remove the mass

C-off the procuder

Elderly c/o dysuria and frequency

What inves to do

A-urine analysis

B-CT Abd

85 years male c/o of ..... sever headache while lefting some thing

A-CT brain ,cervical spine

B-MRI

Case of ASD what true about it

A-it will done about 5 to 6 year

B-must make it at 2 year

Primagravida come e threatend miscarriage after 2 year of infertility what to do

A-ask about infertility

B-pelvic U/s

C-conforming her preg

Child come to ER e meningitis e his parent you want to do csf analysis


A-do it without consent

B-consent from parents abisttant from child

C-consent from parents abisttant from parents

D-consent from both

‫ يعني ايه‬abisttant

Old age of Adenxal mass ..... by u/s what to do next

A-surgical excetion

B-refer for gynecologist work up

C-cytology

Asthmatic pt on short acting and low dose steroid and still has daily symptoms about 5day at night what
to add

A-theophylline
no confirmed answer
B-leukoterian inhaler

C-no thing

IBS come e abd pain x-Ray show thumb points

A-ischemic colitis

B-ulcerative colitis

C-conns

Newborn of diabetic mother from where take sample

A-central venousline
B-peripheral

C-nasogastric !!

D-orogastric !!!

C/o of vomiting on flights what antiemetic to give??

31 years Pap smears -ve what is recommended

A-annually

B-2 year

B-3 year

Abruptio placenta e sever bleeding 36wk hypertension tachycardia Anemia what to do to save life

A-admit to ICU and multidisciplinary team

B- 2 wide bore canule and transfusion blood

C-admit to ICU

3wk e (drop and blood in diaper)!! 2 attak

A-meckels diverticum

B-juvenile polyp

Dr wrong applied forcep during vaginal delivary crush of sternomastoid foramen and its content what
you expected to find in this baby :

a. Unable to close the eye ✔

b. Loss of taste in ant 2/3 of tongue

c. Unable to open the eye d. Sensation in face lost


A 51-year-old businessman complains of dyspnoea on exertion. He recently returned

from a business trip to the USA. He has distant heart sounds on auscultation of the chest.

A chest radiograph reveals that there is a thin rim of calcification surrounding the cardiac

outline.

Which of the following conditions is most likely responsible for these findings?

1- Uraemia

2- Tuberculosis

3- Group B coxsackie virus

3- Sarcoidosis

- 48 years old women complains of abnormal uterine bleeding. On U/S there was fibroid ( endometrial
thickness was 4mm). What is the dx?

Pt diagnosed e retroprotneal cancer what TNM about

A-tansmition to lymphatic

B-metastasis

C-‫مش فاكره الباقي‬

‫ وانا شايفه انو‬a‫و‬b ‫نفس الفهم‬

Patient diagnosed with retro peritoneal what?

Croup case mangment

Abx

O2

I think the 2not mangment of croup

But o2 may be !!
Intussuption mangment ?!

Saline enema

nasogastric tube

Also the 2option not right

I.v fluid and surgery

Parmenstal syndrom➡irritability

.....HCV in KSA

Elderly c/o dysuria and frequency

What inves to do

A-urine analysis

B-CT Abd

85 years male c/o of ..... sever headache while lefting some thing

A-CT brain ,cervical spine

B-MRI

16y boy come e dysuria then after 3day develop foul smelling what the organism

A-porteus

B-klebsella
While make cholecystectomy you find mass what will do

A-cholecystectomy only

B-cholecystectomy and remove the mass betweem A OR B

C-off the procuder

While make cholecystectomy you find mass what will do


A-cholecystectomy only
B-cholecystectomy and remove the mass
C-off the procuder

Elderly c/o dysuria and frequency


What inves to do
A-urine analysis
B-CT Abd

85 years male c/o of ..... sever headache while lefting some thing
A-CT brain ,cervical spine
B-MRI
1- Child asthmatic came to ER with him father c/o SOB because the father smoking near to him
and advice the father many times , what you do ?
1- The parent should be advised to quit smoking in front of the child
2- Tell the family and child protection team
3- To treat the child without caring about the father's situation

2- Child newly discovered of asthma , what you do ?


1-Parents advise to quit smoking
2-prescribe a Neublizer

3- Inguinal hernia complication after mish repair , there pus at site of mish , what to do ?
1-clean the site of mish without remove it
2- remove it

4- Complication after mish repair , pt felt of pain at leg with hyperparathesia at thigh , what to
do ?
1- Remove the mish
2- Remove it with neuroectomy

5- DVT pre operative management 3 questions

6- 2 question of Adrenal tumor seen at CT what to do ( investigation / management ) ?contain


of fat , in choices of management observation

7- Many question in pediatric ( diarrhea / pneumonia )


8- MI treatment without ECG

9- CHF management pt take statin ,fureosmide what add?


1-spirnolactone
2-enalpril
3-propanolol

10- Mitral regurge diagnosis

11- ASD management ?


1-observation
2-wait baby until 5 years
3-no need treat
12- TOF with peripheral cyanosis what the management

13- Male 35 year came to


Case of ASD what true about it
A-it will done about 5 to 6 year
B-must make it at 2 year
22/5

1- Couple come by infertility after investigation done show bilateral fallopian tube
obstruction what will do
Tell wife
Tell husband
Tell couple

2- child with eczematous lesion and mother do good hygiene by warm bath what
will do next:
That enough for him
Olive oil may decrease his symptoms
Add strong soap .......

3- pt c/o burn 25% her weight 70 how to manage


Normal salin 2l/k now and 2L after 6 hr
Ringer lactate 5 L / k after 6 hr and 5 L after 16 hr
Normal slain 5L after 6 hr and 5L after 8 hr
Ringer lactate 2.5 now and 2.5 after 8 hr
4-CTG early decelerations causes
Head compression
Placenta insufficient
Cord prolapse

5- child activity in day:


60 min
90
100
120 min

6- LMC 17may 2019 what is the expected day


24 February2020

7- celiac disease what’s the late prognosis?

8- GBS the expected prognosis:


Permanent paralysis
Full recovery
9- baby born for hb+antigen mother
and received vaccine and antigen and mother worry about breastfeeding for him
because she have history of baby death after breastfeeding what will do :
Wait 12 hr and then breastfeeding
Don’t lactated until do antigen test

10 child 4 months c/o hypospadias in midshift come by here mother for


cercumsetion what will do :
Wait at 6 months an do it
No relationship between cercumsetion and hypospadias
Wait after done from surgery
This type of hypospadias not need surgery so do cercumsetion

11
‫صورةالطفل جاي باحمرار بدون حكة و طالبين التشخيص‬
Atopic Dermatiti
Erythema .......
candidiasis
12- scenario of hemolytic uremic syndrome

13- scenario of B12 deficiency anemia

14 - first line treatment of malaria in KSA


Quinine
Sulfa dioxin
Chloroquine
Milestones
35- Child that throws a ball at you and draws a straight line and stacks “few” cubes on
each other (they didn’t mention the number of cubes). What is the age?
A. 12 months
B. 14 months
C. 18 months
✨D. 24 months
36- A child that can raise his head slightly when prone and smiles. He turns his head
180 degrees and has head lag when you pull him to sit. How old is he?
A. 4 weeks
✨B. 8 weeks
C. 12 weeks
D. 16 weeks
38- Child can set without support, cruises around furniture, uses chair to stand, say
dada, crawl stairs. What is the age of this child?
A. 8 months.
✨B. 10 months.
C. 12 months.
D. 15 months
40 -baby can smile at which age:
✨a. 2 months
b. 4 months
c. 6 months
150-child ride tricycle can't copy square what is the age
3y
151- child can know color but with difficulty in making square
A.2 y
B.3 y
✨C.4 y
D.5 y
152- Baby wave his hand bye bye which developmental millstone reflect?
9m
153- what is the age of child should be know few word ?
A.6 month
B.8 month
C.12 month
✨D.24 month
154-child can support his head when sit and loving when stare to him or cooing
A. 4wk
✨B. 8wk
C.12month
155- Developmental milestones 5 words, hop on one leg ?
✨A.48 m = 4Y
B.36 m
C.24 m
156-Milestone, baby pull himself to stand crawl without difficulty, which age ?
A. 8 months
✨B. 10 months
C.12 months
D.14months
157- ER Child can roll over, sit triploid, attempt to take object
Which month?
✨A. 6
B. 9
C. 2
158- Child tells stories runs and plays father -role.. What's the age?
4y
159- Baby setting in mother’s lap unsupported, when the doctor spoke the baby turned
around and laughed and babbled to the doctor, baby’s age?
A:2m
B:4m
✨C:6m
160- 18 months baby says baba mama what you will do for him
✨A. Developmental assessment
B. Bone age
161- Baby can walk when he held by one hand and good pincer grasp but he cannot put
things in the bottle What is his age:
A . 9 m
✨B. 12 m
162- milestone baby can hold his head and when he looks at his flying hand he laughs
and coos ?
4m
163- milestone said baba and walk holding furniture and a lot of other features
✨A. 12M
B. 10M
280- infant sit in tripod position , role from prone to supine , reach object
A. 4 mon
✨B. 6mon
C. 12mon
D. 24 mon
291- Can't sit without support , coos and laughs
A. 6 months
✨B. 3 months
C. 1 month
D. 9 months
298- child can Know the color when you point at them, ride tricycle but cannot copy
square what is his age ?
A- 2 y
✨B- 3
C- 4
D- 5
302- 4 month child which developmental milestone ?
A- try to crawling
B- roll over from side to side
C- sit without support
✨D- complete fixation of the head
324- development child walk alone & build three cup ?
18m
326- Child say few words what age
✨a) 12m
b) 24m
329- child etc. Can Not take the object in 2 fingers.. How old is he?
a) 4 months
✨b) 6 months = ulnar grasp
c) 9 months
4 year old child , language development
a) Can say 50-100
b) 2 phrase / words sentences
c) Use past tense
✨d) Tell a story
06- child in the hospital play and come to his parents say stories , draw head and hands
and leg by stick figures what is the age of this child:
A. 3yrs
B. 4yrs
✨C. 5yrs
418- Child can walk without support ,crawling ,build 3 cubes point to something he interested
in , so what is the age of the child ?
A. 9month
✨B. 15 month*
C. 2 years
441- Say few words at the age of which ?
A. 24
✨B. 12m
502- milestone > baby healthy run to the doctor play a role model as his father (at 4 Y) can’t
complete a sentence can't eat with spoon (use of spoon 18 m)
12m
503- child sit and support his head , laughing and cooing :
A- 4
✨B- 6
C- 8
D- 16
583،baby sit briefly, crawl , move object from hand to hand , but no pencil grasp.
A- 4 month
B- 6 month
✨C- 7 month
B - 9 month
586- baby said hi when he entered the clinic, imitates his mother, feeds his doll, refers to
himself “ME” and say “eye”:
A.12 months
B.15 months
C. 18 months
✨D. 24 months
590- A child that can raise his head slightly when prone and smiles. He turns his head
180 degrees and has head lag when you pull him to sit. How many old is he?
a. 4 weeks
✨b. 8 weeks
c. 12 weeks
d. 16 weeks
592- Pencil grasp develops at ?
9 months
596- 4 month child with developmental milestone ?
A- try to crawling
✨B- roll over from side to side (roll from supine to prone)
C- sit without support
D- complete fixation of the head
470- baby can sit roll from prone to supine and back play handle object but can't pick things
by 2 fingers age
A-4months
✨B-6months
C-8months
495- Child can roll over ،sit tripod ،attempt to take object Which month?
✨A. 6
B. 9
494- Milestone at 6 months
A-Rolls over supine to prone
✨B-Rolls prone to supine (from P to S = 6M)
MOST REPEATED ETHICS QUESTIONS IN SMLE 2019

1. dr ask you to prepare the RT kidney for op but u are sure it was the LT one what should u do:
# tell the surgeon ✔
2. dr prescripe medication to the pt but the pharmacist thinks that the pt is allergic to it
what
should u do :
Ask him to sign it ✔
3. gyn explain to the pt that HRT will not help her menpousal symptom at this stage but the
pt insist to have it what should he do:
Dont give it to her.✔
4. pt were inappropriate clothes and speeks inappropritaly to the dr what should he do:
Get nurse speak same language .✔
5.6 months old infant his parent brought him for one complaints and through PEx u notice
some
signs of child abuse:
Call social service ✔
6.Mother with appeared with some mental retardation Parents wnt to take hime back
home:
Called child protection✔
7. coupls with infertility ask about serogate pregnancy u are not sure wither allowed in saudi
or not:
Warn them might not be allowed✔
8.elderly with endstage cancer in unbearable pain his family ask u to give him stronger pain
killer :
Give him stronger analgesic✔
9. pt doesn’t want to know the details, u r the surgeon appreciated that , but the anesthsi dr
insist:
Tell the pt the decision of th dr✔
10.pt result came back HIV +ve who came prior for infertility assessment:
Tell the patient first ✔
11.dr after explain to pt what is next mx and explor there emotion went through the main
points again.What did he do is:
Summarisation✔
12.pt with complete abortion while waiting for admission were tearfulness and husband was
emoitinally distress what to do :
Explore and reflect to thire emotions✔
13.DNR pt in respiratory distress surgeon thinks depulking surgery might be helpful butmight
be
fatal:
Dont do it respect patient wish .✔
14.u were late pt start to shout and angry:
Explore his anger ✔
15. female pt wants female to examine her
Respect
16.pt don’t belive this is the correct dx and wants another dr opinion:
Give her report as this is her right✔
17.not immunised child:
Explain the binfit to the family✔
18.consent in case of IUD to be taken from
wife✔
20. a pt. Wants u to do his surgery forcefully and threatens u :
refuse for surgery✔
21.a doc on round prescribed some medicine for pt. That is not good according to the
pharmacist but doctor insisted to prescribe that and tells
the intern to write down orders:
write it down and get it signed from the consultant✔
22. a doc preparing to operate ryt kidney but intern has some doubts that left kidney needs
to be operated :
inform about his concern to the consultant✔
23.a surgeon mistakenly left sth inside the abdomen of the pt during operation
explain to the pt ,apologize and remove that thing✔
24. Case of child newborn abanded by his mother as he result from out of marriage
relatioship .Child is in the hospital he has hernia and pediatric surgeon said he need repair
for his hernia now How u will act :
A. Reveiw ethics section in hospital ✔

B. Do the surgery without consent

C. Refuse to do it as no one to sign

25. You was talking to patient to get details on the history but the patient interrupted u and
start
to talk about his point of veiw regarding his diagnosis and the reason of his disease . How u
would act :

A. Refer him to another doctor


B. Let him finish his point of veiw ✔

c. Apologize to him and start with close ended questions

26.Patient need operation dr start to tell the patient about the positive and possible
complications but patient refuse to hear that but the anesthesia doctor insiseted that the
patient
should know the complications . How u will act :

A cancel the operation


B .Get another anaesthesia doctor
C tell the partient what anaesthesia doctor want ✔
D.let the anesthesia doctor deal woth the patient

27.Male smoker go to clinic asking that he want to quiet how to start this step is :

a. Preparation✔

28. Best way to stop smoking in pregnant is :

a. Behavioral way ✔
b. bupropion

c. Electronic smoking
d. Alternative to smoke

29. 67y old man felt dawn complain of pain on hip and knee assessment releved no fractures
foly catheter inserted on first day as he was unable to move with analgesic after 5 days he
discharged to nursing home with unchanged foly catheter now complain of fever urine
analysis show full wbc in urine with bacteria , this could be prevented by :

a. Prophylactic antibiotics
b. Increase iv fluid intake

c. Daily reveiw for the need of foly catheter ✔


d. Change foly catheter before discharge .

30. Patient admitted with stroke hemiplegia on 2nd day start to having 1st degree bed sores
the
physicin verbally tell the nurse to change patient position every 1 hr to avoid bed sores .
Nurse
forget to write this . 3 days later patient had fever on examination he had infected grade 3
bed sores . Where is the defect :

a. Communication between the doctor and the nurse inadequate way .✔

31. For newly hired female employee the sexual harrasment mostly include :

a. Client

b. Supervisore ✔
c. Colleges

D. Co worker.

32.Parents refuse to vaccinate their child what will u do :

a. Scocial service
b. Ethics department

C. Vaccinate him
d. Tell them about advantages of vaccines✔

33. 12y old attitude toward parent illness :

a. Cant comprehensive the concept of disease treatment prognosis ✔i chose this option

b. Depression withdrawal response

c. Forget

34. Post truma patient hah right side fractures in ribs fron rib 2 to 5 Patient had respiratory
distress cxr show wide mediastinum . U want to transfer the patient to other advanced
center most important step :

a. intubation

b. call the other center and talk to the senior surgeon there tell him details about case ✔

35. Forget guze in abdomen of post operative patient . What to do :

a. Dont tell the patient


B. Tell the patient apologize and get it out✔

36. Patient admitted to icu asked to not be resusitated before and sign form for this he had
arrested no resuscitstion done And die his son came complain why not resusciting hus father
:

a. Tell him that you respect his father wish ✔

b. Father has terminal illness


c. No resuscitation in brain death cases

37. Parents had preterm 24 weeks baby asked not to intubated him what to do :

a. Respect their wish ✔

b. Tell social services


c. Intubate

38.case u admitted dx with ebola want to leave the hospital what to do :

a. Call security ✔

b. Refer to infectiouse department


c. Sign form of leave against medical advice

39. Copd elderly admitted exerberation at night became aggresive disoreinted what the
nurse should do first:

a. Call the duty dr for lorazepsm iv

b. Restrain her

c. Call family sit beside her


d. Elevate head of bed put nasal oxygen try to oreinted her about place and time✔

40. Couple want to do surogacy came to you and you dont know if allowed what to do :

Warn them not allowed in KSA ✔

41. You entered operation for resection of kideny they want to do left but u think its the
right
one what to do :

Tell the surgeon✔


You admitted a pt for surgery but there was a delay n the man is angry n say pls doctor if you
don't discharge me now I will ascond.

A.inform your colleague.

B.call security agents in d hospital.

C.patient should sign dama.

D.refer pt to another hospital✅

) An intern is doing a ward round with nephrology consultant n he as the intern to prescribe
a drug n shortly the pharmacy dept informed the intern that the patients case is
contraindicated in such drugs n he informed the consultant but he insisted of the drugs.

A.prescibe the drugs still.

B.precribe d drugs n give the consultant to sign.

C.humbly decline without something.✅

D.inform the pt.

5) A child without immunization history

A.Council the parents about benefits of immunization✅

B.inform ethics deptmnt in d hospital

C.inform security deptmnt.


6) You admitted a patient for surgery n are aware that if you do the surgery pt will die , what
will u do?.

A.ask pt to sign high risk consent.✅

B.sign normal consent.

11) Couple came for test n husband hiv +ve.

What will you do. Tell husband and ask him to tell his wife if refuse ask health agency

A.tell the couples.

B.tell the husband.✅

C.tell the wife.

D.tell the medical ethics dept.

3) A case of Surrogacy in infertility.What will you do since your hospital doesn't offer such
service.

A.refer to another hospital.

B.refer to you colleague that has surrogacy unit contact.

C.tell the couple is no allowed in saudi Arabia.✅

D.book another appointment with them after 5dsys

16) A nurse came to work with transparent uniform,and was talking to you in a seductive
way.

A.walk her out that minute n bring another.

B.report her to the Md.✅

C.leave the office.

17) You want to examine a female patient.what do u do.

A.bring a nurse.✅

B.bring in the relative.

C.continue with you examination alone.


18) A case of infertility for 10years,with irregular menses n the woman came with bleeding n
after ultrasound is incomplete miscarriage.

What will do after evaluation.

A.show empathy n sympathy.

B.empathy should be enough.

C.show empathy n console the couples emotions..✅

‫س سوال واحد زعالن انك تأخرت عليه بسبب انشغالك وشلون تتعامل معه * واحد رفض العالج مع انه يحناجه تلقى‬
‫الجواب انك تصنحه وتحاول تقنعه * وحد حامل ووصار معها ورم والزم تشيله ورفضت تشيله وش تسوي معها * اب‬
‫ الجواب انك تصحح لهم‬.. ‫ يقولون ان التطعيم يضر المخ‬.. ‫وام جايبين ولدهم عمره كم سنة بس ما طعموه وال مرة‬
‫مفاهيمهم * وتشرح لهم‬

Back ground

fully competent.

According to existing regulations in Saudi Arabia, this age is 18 years for


both males and females.

The Age Of Discrimination,

The age of 7 is considered sinn al tamyiiz (‫) ٍ س زييًتنا‬, after which a child
can make some decisions.

Competent

A normal adult is judged legally competent unless there is a reason to


suspect otherwise.

If there is such a suspicion, or if the nature of the disease affects mental


ability.

Specific tests of competence should be carried out. In simple cases, a


physician caring for the patient can test for competence by asking
simple questions about, for instance, the patient‟s name and
address, orientation in time and place, the patient‟s ability to
understand and retain information, and making judgments.
complicated cases, a clinical psychologist may be invited to test for
competence in a formal way.
Who take the decision when patient is judged legally incompetent?

substitute is usually a member of the family

If the patient indicates at the time of admission which family


member should represent him, then we follow the patient‟s
wishes.
If the patient does not nominate anyone, according to Saudi
customs the father has the right to decide if not present the most
senior one
In some cases, the father acts as the decision maker even if the
patient has designated someone else.

Consent for the unconscious

Patients with complete loss of consciousness are considered incompetent,


and decisions about treatment are made by their next of family.

If the next of family is not available, the physician will undertake urgent
treatment that is considered in the best interest of the patient.

If the patient left an advance directive, ‫ وصيه‬it should be respected


unless it is irrelevant to the actual clinical conditions that the patient had
not anticipated at the time of making the directive.

Any decisions made by a proxy decision maker or the physician, have to


be confirmed or even reversed when the patient recovers consciousness.

If the treatment considered is not urgently needed to save life, the matter
should be referred to a court of law for a decision if the procedure is a
major one, like amputation or transplantation.

If the procedure is minor, a designated official in the hospital can be


the decision maker.

Who has the authority to make decisions for children?

Parents have the responsibility and authority to make medical decisions on


behalf of their children.

This includes the right to refuse or discontinue treatments, even those that
may be life-sustaining.

When can parental authority to make medical decisions for their


children be challenged?
emergency life-saving treatment

If both parents refuse, the physician can go ahead and give emergency life-
saving treatment with no consent in the interests of saving life.

Non-Urgent Treatment

Refusal by one or both parents of non-urgent treatment that the physician


considers necessary for saving life can be resolved by reference to the law
courts

when parental decisions decisions place the child at significant risk of


serious harm.

1- respectful discussion with parent if still refuse


2- ethics consultation. if still refuse
3- involvement of a State child protection agency or a court order might
be necessary.

What if parents are unavailable and a child needs medical treatment?

In general, a child can be treated or transported without parental permission


if the child has an emergency condition that places his or her life or health in
danger until parental permission can be obtained.

Examining a child who presents to medical attention is always appropriate


in order to establish whether a threat of life or health exists.

Should children be involved in medical decisions even though their


parents have final authority to make those decisions?

As children develop the capacity to make decisions for themselves, they


should be given a voice in medical decisions.

final authority to make medical decisions will usually remain with their
parents.

What happens when an older child disagrees with her parents about a
medical treatment?

1-first attempt to resolve the issue through further discussion.

2- ethics consultation or judicial hearing should be pursued.

Under what circumstances can minors make medical decisions for


themselves?
Economically self-supporting and not living at home
Married
A parent
On active duty in the armed services.
If minor is mature usually requires that the minor be older than 14
years of age and have demonstrated a level of understanding and
decision-making ability that approximates that of an adult. While
some states allow physicians to make this determination, most require
a judicial determination of mature minor status.
sexually transmitted diseases.
Pregnancy.
Contraception.
psychiatric disorders.
drug or alcohol abuse

Case scenario 1: Autonomy as the basis of informed consent

An 80-year-old, fully conscious, and competent man with advanced


incurable cancer needed palliative chemotherapy. The family
objected when the doctor wanted to obtain informed consent from
the patient because that would involve disclosing the diagnosis,
which would make the patient very sad and depressed. The family
wanted to make the decision without informing the patient. What
should the doctor do? Provide your moral reasoning.

Case 1: Autonomy as the basis of informed consent

The doctor should respect the patient‟s autonomy. He should first ask the

patient whether he personally wanted to receive information about his

condition in order to make decisions on his treatment, or whether he


would prefer that the information be disclosed to his family, and the
family authorized to make decisions on his behalf. If he insists on
making decisions for himself, he must receive full disclosure and
exercise his autonomous right to informed consent. If he chooses to leave
everything to the family, the doctor can deal with the family accordingly.
Case scenario 2: Scope and limitations of consent

A 30-year-old woman presented with classical signs of acute


appendicitis. She consented to an operation to open the abdomen and
remove the inflamed appendix. The surgeon found a previously
undiagnosed ovarian cyst and decided to remove it. The removal was
a simple and safe procedure that would not have increased the
duration of the operation. The head nurse refused because the
patient had not given consent. What should the surgeon do? Provide
your moral reasoning.

The doctor should not go ahead with the removal of the cyst because that
would be outside the scope of the informed consent obtained. In this
case, there is no emergency life-saving need to operate without consent.

Case scenario 3: Consent and protection of the patient

An 80-year-old diabetic man, whose son had died last year from a
transfusion of mismatched blood, was admitted to the same hospital for
observation after falling at home. He insisted that no procedure be
carried out without written approval by his physician son, whom he
wanted to sit by his bedside all the time. Nurses were inconvenienced by
having to get written permission for routine monitoring of vital signs and
insulin injections. The nurses refused to comply with his wishes and he
refused to cooperate, leading to a standoff. What should the doctor in
charge do? Provide your moral reasoning.

The doctors should respect the patient‟s autonomy and accommodate his
needs as much as possible. If, however, they find that complying with the
patient‟s wishes is not possible without disrupting the work of the ward,
they can follow the procedures for refusal of treatment by the patient,
which may later lead to discharge to another institution with the capacity
to handle the patient‟s needs.

Case scenario 4: Consent and the protection of the physician


A young neurosurgeon planned to operate on a patient with lumbar
spinal injury that had a 5 - 10% chance of success. He felt uncertain
about taking informed consent. If he informed the patient that the
operation could go wrong and result in paraplegia, there was a 90%
chance the patient would refuse the operation. If the operation was
not carried out, there was a 95% chance of further deterioration,
leading to paraplegia after a few months. What should the
neurosurgeon do? Provide your moral reasoning.

The patient has a right to full disclosure even if that will result in refusal
of treatment. Fear of refusal of necessary treatment is not a justification
for violating the patient‟s autonomy

Case scenario 5: The process of informed consent

A complex brain operation had a 3-page risk disclosure sheet. The


surgeon determined that his poorly educated patient could not
understand the information even with the best of translations, and
might even refuse the life-saving operation. The operation was
necessary to release a hematoma and a fractured bone fragment
putting pressure on the cerebrum, which would soon lead to loss of
consciousness due to increased intracranial pressure. He gave the
patient simple information that the operation would help him recover
from the effects of trauma and that it had some risks, which he did
not mention. What should the surgeon do? Provide your moral
reasoning.
The patient is entitled to full disclosure, but a summary will suffice if it
excludes technical details but covers the major benefits, and especially
the risks, of the operation in simple language. This is justified because it
maintains respect for the right of the patient to know.

Case scenario 6: Capacity/competence to consent

A university professor admitted for stroke refused life-saving


treatment even after a thorough explanation by his son, who was a
neurosurgeon. While in the hospital, he seemed to forget essential
information about his illness, forgot his age and his wife‟s name, and
was confused about the day of the week. However, he was in
continuous telephone contact with his laboratory at the university,
guiding the young researchers. What should the doctor do? Provide
your moral reasoning.

A formal testing of competence by a physician or psychologist is


necessary in this case. If the professor is found competent, his refusal of
treatment should be upheld

Case scenario 7: Proxy consent/substitute decision maker

A 30-year-old victim of a road traffic accident was in a deep coma,


with some signs of brain stem function, and was put on life support in
the ICU. He had told his wife before the accident that he would like
to be left to die in dignity rather than live with the aid of machines.
He had also authorized his wife, in writing, to make decisions
about his treatment if he fell unconscious.Led by his father, his
family refused this and insisted that life support continue until
recovery. What should the doctor do? Provide your moral reasoning.
The decision of the wife based on the desires of the patient is respected.
However, it is possible for the father to override her based on
considerations of the Sharia.

Case scenario 8: Prospective consent/advance directives

A 40-year-old victim of multiple sclerosis, aware of the final stages of


his illness, signed an advance directive authorizing doctors not to
initiate life support if he stopped breathing on his own. He developed
acute pneumonia a short while after writing the directive, and
experienced severe respiratory distress. The doctors were not sure
what to do. Members of the family were divided in their views. What
should the doctors do? Provide your moral reasoning.

The advance directive was related to respiratory failure due to multiple


sclerosis and cannot be applied to respiratory failure due to acute
pneumonia

Case scenario 9: Consent for children

A 14-year-old boy with bone cancer confined to the tibia refused


amputation that would prevent spread of the cancer to other parts of
the body. He understood the adverse consequences of his decision.
His father and mother opposed his decision and authorized the
surgeons to carry out the amputation. What should the doctor do?
Provide your moral reasoning.

A 14-year-old cannot make a decision to refuse treatment, so in this case


the parents‟ decision is the one upheld.

Case scenario 10: Consent for the mentally impaired


A 14-year-old mentally impaired girl used to wander from her home,
and her parents feared that she might be raped and become
pregnant. They took her to the hospital and asked the doctors to
sterilize her. The doctors talked to her and she opposed the operation
vehemently. What should the doctor do? Provide your moral
reasoning.

The 14-year-old is not competent to decide. In view of the irreversible


nature of the operation, advice of a court of law should be sought.

Case scenario 11: Consent for the unconscious

A 60-year-old diabetic was admitted to the hospital in a coma due to


diabetic keto-acidosis and a gangrenous foot. The doctors decided to
amputate the foot as soon as the general condition had stabilized
enough to withstand anesthesia. The patient‟s sons and daughters
refused the operation, even after explanations that the gangrene
would spread and result in fatal septicemia. They reasoned that it
was better for him to die and be buried with all parts of his body
than to live with an amputated limb. What should the doctor do?
Provide your moral reasoning

The decision of the family is upheld in this case because they are the
valid substitute decision makers.

ethical issues in terminal care

have no clear solutions; the right solution depends on the circumstances.

1. Terminal patients are entitled to palliative care.

2. They can make decisions about their care if competent, or by advance


directives made before losing their competence.

3. In cases of incompetence, decisions are made by proxies.


4. Futile resuscitation and artificial life support should be withheld, and
if started, should be stopped.

5. Organ harvesting with proper consent can be carried out in patients


with brain stem death

People need information to help them take appropriate health-


related decisions

2. This information should be presented in a way they can understand

3. The “right to know” has many ethical principles and duties related to it

4. The patient‟s right to know the truth does not mean forcing him/her to
know

5. The patient should be the one to delegate one of his/her family


members to be the substitute decision maker

6. If the patient is incompetent, then the doctor should follow the policy
of the hospital in which he/she works

7. Disclosing unfavorable information has more than one approach; we


presented the SPIKES Protocol for Delivering Bad New

Medical errors
common in surgical, obstetrics and gynecological, and surgically-related
specialties. Drugs also represent an important type of error; others are
fluid-related, such as blood transfusions or IV fluids.

2. Patients want a full disclosure of all the errors that result in harm, and
they need to know what happened and why, what the implications are,
how the problem occurred, and how to prevent it.

3. Disclosure should take place at the right time, when the patient is
medically stable enough to absorb the information, and in the right
setting. A physician should take the lead in disclosing error(s) to patients
and their families.
4. We should disclose our medical error(s) quickly, or as soon as
possible, with full disclosure to the patient. We should then offer
physical or psychological support, after discussion with our senior
colleagues in a calm setting.

Inexpensive gifts, limited hospitality, and travel sponsorship are


acceptable, and our professional associations set out clear criteria.

Professional codes of practice ban lavish gifts or inducements from


pharmaceutical companies, and prohibit company representatives
from offering them. Therefore, we should be aware of the Saudi
professional codes of practice that require prescribers to put the
patient‟s interest first
1-2 wks Neonate has non bilious vomiting on US there is olive what you find

A-Hypercholaremic metabolic acidosis

B-Hyopchl Met alkalosis

C-Hypochl resp alkalosis

2-Infant crawel, transfere object from hand to hand, had palmer grasp no pincer grisp, age by
month

A-5

B-7

C-9

2-Child e nephrotic synd developed abd pain wh you expect

A-Uti

B-Peritonitis

3-Women diabetic controlled developed dysuria and frequency urine analysis nitrate, creatinine
high

Which drug is contraindicated

A-Amoxicillin

B-Septrin

C-Ciprofloxacin

D-Pipracilin tazopactam
4-Pt e thoraccentesis site of needle

A-4th

B-5th

C-6 and 7

D-7 and 8

5-30 yrs old e painless jaundice no other symptoms physical exam normal, bilirubin 15 mainly
indirect liver enzyme normal wh is the d.d

A-Hypothyrodism

B-Cholidecal cyst

C-Gilbert synd

6-7yrs old had clumsy gait and unable to walk or stand, had checken box begore 3 wks, muscle
taken no abnormalities also had resistance to neck flexion wh is the d.d

A-GBS

B-Meningoencephalities

7-Women e vaginal discharge offensive fishy odour revealed clue test d. d

A-Bacterial vaginosis

B-Trichomonas vaginosis

8-11yrs old had night bed e out to go to toilet exam normal he feel shamed and asked help

A-Avoid punishment

B-Alarm bed e Reinforcement

C-Desmopressin intranasal
9-After cholecystectomy pt developed pain at angle of mandible, temp 38 wh to do

A-Paracetamol

B-Abs

C-Xray

D-CT

10-Pt developed appendicular abscess drainage was done wh is the pathophysiology

A-Redistribution of bld supply

B-Cardiac index

C-Bradycardia

D-Peripheral vasoconstriction

11-Pregnant 10 wk had symp of cholecystites wh to do

A-Immediate lab chole

B-Lab chole after delivery

C-Lab chole at 2 trimester

D-Lab chole at 3 tri

12-Elderly pt had pneumonia she became restless and agitated for the nurse what will do

A-Tell the family to stay e her

B-Give oxy and elevate the head bed

D-Call the Dr duty and to give lorazepam 5mg


13-Pt diagnosed ALL inves twbc 22 Hb 8 K5.7 phosphate low uric acid high which of the
electrolyte ass e this condition

A-HypoCa

B-HyperCa

C-HypoNa

D-HyperNs

14-Female presented e scanty vaginal bleeding she noticed that post coidal what is your action

A-Pelvic us

B-CBC

C-assess the vagina and cervix

15-Femal e mass at lt upper breast, painless, movable not related to menstrual cycles wh is the
d. d

A-Fibrocystic

B-Fibroadenoma

C-Ductal papilloma

16-Pt known congestive heart failure on Lasix spironolactone and digoxin also Asthmatic on
salbutamol invest showed K 2.2 wh is the cause

A-Digoxin

B-Salbutaml

C-Spironolactone
17-Female came to clinic e her sister which was known adult polycystic kidney disease, she
asking for screening wh you will requst

A-Abd US

B-Antibody for polycystic kidney disease

18-Infant crying e abd pain, he pass jelly and bloody stool

what is the most appropriate invest to reach the diagnosis

A-Barium enema

B-Abd x ray

C-Abd US

D-Abd CT

19-Child e abd pain ass e headache physical exam normal

A-Abd migraine

B-Other choice I didn't remember

20-Pt came e fever and sore throat on exam hyperemic tonsil wh complication can occur

A-Glomerulonephrites

B-Pneumonia

21-One yr old presented e high grade fever, has drolling of saliva, he looks ill, toxic, febrile what
is the intervention

A-Urgent refer to ENT for tonsillectomy

B-Admit to ICU and prepare if need intubation

C-Give him Oral abs for 7 days


22-2yrs old presented e fever and cough and resp distress on exam there is bilateral crackles

X ray showed both lung infiltrate e something I forget what is the problem

A-Bronchopneumonia

B-Bronchoectasis

C-Cystic fibrosis

23-Pregnant 37wks came to the clinic for follow up BP 140/90 no headache, invest on protinuria

Wh is the dd

A-Pre-eclampsia

B-Chronic HTN

C-Gestational HTN

24-When you will do screening for gestational diabetes

A-14

B-24

C-34

25-Pt 34yrs developed post-partum hemorrhage after S. V. D they give oxytocin and do massage
like that but not stopped wh you will do

A-Hysterectomy

B-Ligation of uterine artery


26-Postmenopausal came to take hormonal replacement therapy the Dr said no need according
to her condition but she is insisted for HRT What is your action

A-Refer to another gynecologist

B-Refuse

C-Respect her and give HRT

27-Pregnant 10 wks presented e vaginal bleeding on exam cervical os is closed what is the dd

A-Incomplete abortion

B-Missed abo

C-Threatened abo

28-Pt hypertensive on lisinpril 20mg, amlodipine 5mg and warfarine 2mg, he developed cough,
AF bacilli positive, anti T. B started what next

A-Decrease lisinpril

B-Increase amlodipine

C-Increase warfarine

D-Stop the rifampacine

29-Pt developed neck pain radiate to the shoulders e numbness wh is the dd

A-Rotator cuff

B-Cervical disc proplace

C-Polymyolegia rhumatica
30-Pt developed vesicle in mouth e cervical lymph node on exam spleen 2cm BCM What it can
be cause

A-EBV

B-HSV

C-HPV

D-HZV

31-Child had fever for 5 days, erythematous red in lip and peeling of the palm, ass e purulent
conjunctivitis what is the dd

A-HSV

B-Infectious mononucleosis

C-Kawasaki disease

32-Women had Rt upper quadrant pain e vomiting, jaundiced on exam tenderness at RUQ invest
US shower gallbladder stone e dilated CBD What next

A-CT abd

B-MRCP

C-ERCP

33-Women presented e skin rash and malar flash e severe joint pain, CBC, RFT normal what you
will add

A-Cyclophosphamide

B-Mesotrexate

C-Azathioprine
34-Pt presented e recurrent upper and lower resp infection ass e eczema, invest HB 12g/dl wbc
10     plt 50 what is the dd

A-Severe combined immunodeficiency

B-Wisckot Aldrich synd

C-Bruton a gammaglobinaemia

35-Man presented e severe abd pain, central, he had hx of cardiomyopathy what is the dd

A-Abd aortic aneurism

B-Mesentric vascular occlusion

36-Pt came e cervical lymph node enlargement the pt ask to take biopsy which of the following
if present for biopsy

A-If lymp node more than 1cm

B-If there is fever

C-Supraclavicular lymph node

37-Old man presented e rectal bleeding by exam they found 3rd degree homorroids what next
A first-degree internal hemorrhoid bulges into the anal canal during bowel movements. ... A
A-Colonoscopy third-degree hemorrhoid bulges from the anus during bowel movements and must be pushed
back in with a finger. A fourth-degree hemorrhoid protrudes from the anus all the time
B-Homoroidectomy
Cream to third
Ligation to fourth
With some overlapping!
38-Pt e rectal bleeding and pain during defecation diagnosed as anal fissure e skin tag what is
the management

A-Anal dilation

B-Lateral internal sphincterotomy

C-Lateral external sphincterotomy

D-Something e remove the skin tag


39-Pt after RTA became alert and agitation there is Lt side chest contusion and unable to
elevate his elbow and unable to move his lower limb, vitally hypotensive what is the cause of
this hypotension

A-Internal hemorrhage

B-Cardiac tamponad

C-Spinal cord injury

D-Tension pneumothorax

40-Women developed palpitation, pule rate 174,vitally stable what you will give

A-Amidarone

B-Adenosine

C-DC shock

41-Child had fever and dark urine on exam there is mild tenderness in the abdomen, BP is high,
urine analysis showed RBCs and 2+protein what is the treatment

A-Frusimide

B-Predinsolone

C-Ceftriaxone

42-Pt e glomerulonephrites developed heamoptysis what is the dd

A-Good posture synd

B-Other choice

43-Pt known case of ALL on chemotherapy he developed fever, invest CBC pancytopenia what
to do

A-Take blood, urine culture and oral Abs

B-Take blood, urine culture and inj Abs


44-15 yrs female had short stature and short neck she didn't get menstrual cycle, wt is the dd

A-Turner synd

45-Father bring his child because his brother died due to immunodeficiency or like that what
vaccine should be not given for this child

A-Influenza

B-Injectable polio

C-Varicella

46-Pt underwent surgical procedure for lung cancer, developed polyuria, polydipsia, invest
showed hypernatremia what next

A-Increase fluid intake

B-Desmopressin

47-Specific test for nystagmus

A-Red reflex

B-Cover test if there is dix hallpike maneuver choose it

C-Corneal light reflex

48-First sign of puberty for female

A-Adrenarche

B-Thelarche

C-Pubarache

D-Menarche
49-What malignancy ass e hashimoto thyroditis

A-Papillary

B-Nodular

C-Medullary

50-In which position can occur 4th degree pearneal tear Intrapartum risk factors
Instrumental delivery (eg forceps, vacuum)
Prolonged second stage of labour (>60 minutes)
Epidural use
Oxytocin use
3 or 4 cases for Bronchial Asthma
Midline episiotomy
Delivery in lithotomy or deep squatting position
Cases for IHD

Case of bacterial endocarditis

Case of SLE

Case of IBD

Case of DKA

Case of endometriosis

Man e HTN developed dysuria and intruptted urine stream invest PSA 1 what next

Beta blocker

Alpha blocker

Trans urethral prostectomy


DHA, 12 MARCH 2019.
THERE WERE ALMOST 40-50 REPEATED QUESTIONS FROM PREVIOUS PAPERS

ACTUAL QUESTIONS ARE WAY DIFFERENT AND MORE ELABORATED


SCENARIOSWITH LAB VALUES AND NECESSARY INFORMATION THAN RECALL
QUESTIONS.

RECALL QUESTIONS ARE JUST TO HAVE AN IDEA ABOUT THE TOPICS FROMS WHERE
U HAVE TO STUDY , MOSTLY CLINICAL.

NO ONE GIVES THE GUARANTEE OF CORRECT ANSWERS, SEARCH YOUR OWN.

1. An 8 yr old girl with weakness since birth, complaints of upper right abdominal
pain, hepatomegaly, jaundice, and ascites…diagnosis..pic of both usg and
histopathological slide given.

TYPE TO ENTER A CAPTION.


a. liver cirrhosis

b. fulminant liver failure

c. budd chiarri syndrome

d. hepatic cholangipathy

2. chlamydial infection…what measures taken along with intervention to public


health education.

a. water and sanitation

b. sanitation and kill reservoirs

c. kill reservoirs and improve sanitation

d. eradicate reservoirs and improve sanitation.

3. 35 year old smoker , on examination shown white patch on the tongue,


management:

a) Antibiotics

b) No ttt

c) Close observation

d) d) biopsy and excision

4. A television actress is suffering from rosacea. Since she states that the
appearance will affect her career, what is your choice of treatment?

a. Oral antibiotics

b. Antihistamines

c. Topical antibiotics

d. laser

5 initial treatment for major depression: ssri

6. A case of schistosomia hematobium infection(liver fluke),what hygiene measure


needs to be taken :

A. Build latrines

7. developmental age of a child… can sit without support, friendly with strangers but cannot
play peek a boo….

a. 6-7 month
b. 7-8 months
c. 8-9 months
d. 9-10 months

8. 40yr.old woman asking about ca cx screening, pap smear when to do and asking for ur
advice…

a. no need now
b. can be done 1 year starting from now.
c. can be done every 5 year from now
d. if 3 negative , no need to further test for screening.

9. mechanical irritation in eye for a worker doing job in an industry..what to apply..

a. terbinafine drops
b. ciprofloxacin drops
c. olopatadine drops
d. penicilline drops

10. patient with severe headache and vertigo and pain during lifting head, cannot maintain
a steady gait while walking. Vestibular neuritis is a disorder that affects the nerve of the inner ear called the vestibulocochlear
nerve. The disorder may cause a person to experience such symptoms as sudden, severe vertigo
a. bppv (spinning/swaying sensation), dizziness, balance difficulties, nausea, vomiting, and concentration
difficulties.
b. vestibular neuritis
c. optic neuritis Vestibular neuritis and labyrinthitis are closely related disorders. Vestibular neuritis involves swelling
d. snhl of a branch of the vestibulocochlear nerve (the vestibular portion) that affects balance. Labyrinthitis
involves the swelling of both branches of the vestibulocochlear nerve (the vestibular portion and the
cochlear portion) that affects balance and hearing. The symptoms of labyrinthitis are the same as
vestibular neuritis plus the additional symptoms of tinnitus (ringing in the ears) and/or hearing loss.

11. known patient of alzheimer’s family asking for treatment?

New methods of daily activity

12. Health worker exposed to hep b . took all vaccines earlier.


what to give now.

a. hbig + vaccine
b. hbig + lamivudine for 2 weeks
c. only lamivudine for 2 weeks
d. no risk as already immunized
13. scarlet fever treatment

a. ivig
b. erythromycin steptococus infection
c. intubation
d. paracetamol high dose

14. rheumatoid arthritis pt with swelling in hands..(not mentioned about pain) what to advice..

a. nsaids
b. methotrexate will help after 3rd stage
c. intra articular corticosteroid injections
d. symptomatic tx

15. pt diabetes type 2, with no pedal pulses in left leg, cold, raised red color swollen area over
the medial side of calf…(no pic given)

a. cellulitis
b. peripheral arterial insufficiency
c. polymyositis
d. pretibal myxoedema

16. Long scenario of a man with twitching of facial muscle upon tapping of the face.there is
elevation in creatinine level diagnosis:

a)Pseudohypoparathyroidism

b)di george s/d

c)chronic renal failure

d) hypoaldosteronism

17. long scenario given about pt having hepatomegaly

lab values given alt 40 ast 200 bil 160 asked about diagnosis

a. liver cirrhosis

b. hepatic angiopathy

c. portal hypertension

d. biliary cirrhosis.

18. daily requirement of vit D:400 IU/ml

19. case of a female came after 3 months of delivery, saying she used to suffer with low mood
, irritability during the first 2 weeks of delivery but symptoms resolved now completely….what
condition did she suffer from…?

a. postpartum psychosis

b. pp blues

c. pp depression

d. pp hallucinations

20. All deciduous teeth fully erupt at age??

a. 2-3 yrs

b. 3-4 yrs

c. 4-5 yrs

d. 5-6 yrs

21. Unmarried young lady , 20 yr old, with bilateral abdominal pain, she is a waitress at a resort,
very vague sexual history, no bleeding, no significant and examination….no fever, missed last
month periods….diagnosis.

a. ovarian torsion

b. salphingitis

c. ruptured ectopic pregnancy

d. endometritis and adnexal masses

22. indirect inguinal hernia mechanism


23. pt with pruritic folluculitis in extremitis treatment?

A. benzyl peroxide with hydrocortisone


B.topical antibiotics
C.oral antibiotics
D. systemic antibiotics.
24. overall deaths percentage due to post part haemorrhage.
a. 15%
b. 25%
c. 20%
d. 10%

25. 58 yr old woman, h/o epigastric pain, for 2 months, no rebound


tenderness, no abdominal mass, no bruit heard over abdominal area, history
of mild discomfort over chest since 6 months
ecg of st elevation in 2, 3, AvF given….

a. MI
b. peptic ulcer disease
c. abdominal aortic aneurysm
d. coarctation of aorta.
26. normal bereavement case
sadness for 2-3 month max 6month

27. pt with thyroid function test…


tsh 400
t3 normal
t4 normal, diagnosis…

a. sub clinical thyroiditis


b. primary hyperthyroidism
c. primary hypothyroidism
d. goitrogenic cancer

28. 14 yr old child diagnosed with DM type 1, when to refer for ophthalmic
examination…

a. now and repeat after every 5 years


b. after every 3 years
c. each year after the age of 40.
d. yearly, after 5 years from now

29. pt with meningitis, his sibling allergic to the primary treatment given for
prophylaxis….what to give him as prophylaxis now??
a. erythromycin
b. vancomycin
c. ceftriaxone
d. cefuroxime
30. pt with urti, since 2 weeks, red congested throat pic was given… (streptococcal
pharyngitis) treatment…

a. certriaxone
b. vancomycin
c. doxycycline
d. cephalosporin.

31. smoker with 30 yr pack h/o smoking …..asking about cancer , what cancer he is
most susceptible of
a. bladder ca.
b. colorectal ca.
c. bone ca.
d. prostate ca.
32. LDL level goal in a normal diabetic pt.
a. <70mg/dl
b. <100mg/dl
c. <110mg/dl
d. <70mcg/dl

33. pre auricular tenderness with fever and headache..


a. chlamydia inf.
b. gonorrheal inf.
c. influenza inf.
d. coxsackie virus inf.

34. why doctor ask about site of pain in backache..


a. location and duration help to know about biological origin
b. location and duration help to know about biological outcome

35. pt after colectomy due to colorectal ca follow up?


A.3month
B.6month
C.anually
D.2yrs

36. common cause of clinic visit in pt with chronic kidney disease?


a. Anually routine blood test
b. oliguria
37. 40 yr old man diagnosed with htn….advise
a. restrict salt intake to 4g or decrease weight

b. do heavy exercises
same Q in fudul
c. avoid fruits and vegetables
d. avoid organ meat..

38. corneal scarring….what to advise…


a. topical antibiotic
b. systemic antibiotic
c. topical steroid
d. systemic steroid

The medication works by binding to and inhibiting the ATP-sensitive potassium


39. Glipizide mechanism 

channels (KATP) inhibitory regulatory subunit sulfonylurea receptor 1 (SUR1)
[9] in pancreatic beta cells. This inhibition causes cell membrane

 depolarization, opening voltage-dependent calcium channels. This results in an
increase in intracellular calcium in the pancreatic beta cell and subsequent
a. Increase secretion of insulin from pancreas
 stimulation of insulin release.
b. Increase sensitivty of cells to insulin

c. Others choices After a cerebral ischemic insult, the blood–brain barrier is broken and
glibenclamide can reach the central nervous system. Glibenclamide has been
shown to bind more efficiently to the ischemic hemisphere.[10] Moreover,
under ischemic conditions SUR1, the regulatory subunit of the KATP- and the
NCCa-ATP-channels, is expressed in neurons, astrocytes, oligodendrocytes,
endothelial cells[11] and by reactive microglia.[10]
40. pt exposed to mosquitoes, have high grade fever, rash appears over the body ,
splenomegaly , platelet count 40,000. causative organism spread by
a. aedes egyptii
b. female anopheles
c. sand fly
d. tick

41. pt with diabetic nephropathy , u have to assess the severity of disease…


a. 24 hour urine protein
b. albumin creatine ratio
c. urine albumin
d. ketones in blood
42. a child presented with fatigueability,...., IX showed low HB (picture of peripheral blood film
showing target cells), the phycisian ordered blood transfusion, what else he should get:


a- methotrexate.


b- desferroxamine
c. penicillin
d. pneumococcal vaccine.

43. pt with scaling skin lesion in the extensor surface of the foream( with picture), your advice:


a- avoid trauma. 


b- avoid sunlight

c. apply sunscreen

d. green tinged make up

44. scenario about pt with previos infective endocarditis, going for urological surgery, it stated
in the question high risk for IE and penecillin allergic:


a. iv vancomycin and gentamycin 


b. oral vancomycin and tetracyclin

c. oral ampicillin and ceftriaxone

d. im gentamycin and oral erythromycin

45. 8 yr old girl with pain abdomen. she is weak since birth, h/o hepatomegaly, with jaundice…

AST 1024 ALT 600 bilirubin 370

a. biliary stricture

b. liver cirrhosis

c. hepatic cholangiopathy

d. congenital liver failure

46. pt developed neurological symptoms after taking tb medication..what to give for


decreasing these symptoms..

a. tobramycin

b. tenefovir

c. gabapentin

d. pyridoxine

47.55year old patient with dysphagia for solids with a previous history
of reflux
A. Carcinoma oesophagus
B. Stricture
C. Scleroderma
D. Achalasia
48. x ray of pneumothorax, pt after trauma developed difficulty in breathing,
asking best initial management

a. needle aspiration

b. chest tube insertion

c. intubation

d. emergent surgery referral


49. iv fluid management a 22kg child, hourly fluid to be given…
a. 25ml/hr
b. 65ml/hr
c. 110ml/hr
d. 600ml/hr

50. Patient came for assessment after colle’s fracture by falling on outstretched arm with
minimal trauma, what is the appropriate test to check for bone density?
A- VIT D
B- Calcium level
C-X RAY hip and pelvis
D-Dual energy x ray absorptiometry

51. Lady with migraine,during pregnancy it improved.trying to reduce work stress.whice is


best accomplishment of preventing migraine attack.
a. biofeedback
b. beta blocker.
c. sumatriptan
d. 100% oxygen.

52. A female patient came to the clinic complaining of a mass on a vagina she has a

history of repeated unprotected intercourse with multiple partners, upon


examination. she has a wart in the vagina , the causative agent is :

a. Herpes simples

b. Neisseria Gonorrhoea

c. Treponemma pallidum

d. Molluscum contagiosum

53. female pt with small papule on genital area with central pitting, h/o unprotected
sexual intercourse with multiple partners,

a. Herpes simples

b. Neisseria Gonorrhoea

c. Treponemma pallidum

d. Molluscum contagiosum

Lady with migraine,du ring pregnancy it improved.trying to reduce


work stress.whice is
best accomplishment of preventing migraine attack.
a. biofeedback
b. beta blocker.
c. sumatriptan
d. 100% oxygen.
54. old pt with macule and papule around nose and cheeks, one small pustule
over nasolabial fold, skin red, flushy and telangectasias..

a. actinic keratosis

b. rosacea

c. tinea versicolor

d. seborrhea dermatitis.

55. which of the statement is true about exercise

A. Increase basal metabolic rate

B. Doesnot effect the waist fat

C. Not recommented for cardiac patient

56. Younger diabetic patient came with abdominal pain, vomiting and ketones
smelled from his mouth. What is frequent cause:

a) Insulin mismanagement

b) Diet mismanagement

57. x ray..…child with no cough, toxic looking, initial treatment…

a. antibiotics

b. refer to orthopedic dept.

c. intubation

d. reassure

58. pcp in hiv

59. Patient with Celiac disease doesn`t follow diet. Came to the physician
regarding this condition. If he doesn`t follow diet which part of intestine much more
influence?

a) Proximal small bowel

b) distal small bowel

c) proximal large bowel

d) distal large bowel

60. Patient complain of light-headedness, tachycardia, diarrhea, relieve by lying down,


history of gastrointestinal surgery before 2 month, what is your provisional diagnosis

a) IDS

b) Dumping syndrome

c) Villous adenoma

d) Crohn`s disease

61. about hernia advise in asymptomatic………surgery is required

62. Patient comes with neck swelling, moving with deglutition located hyoid area. What is
diagnosis?

a) multi nodular goiter

b) Thyroglossalcyst Thyroglossal cyst usually presents as a midline neck lump (in the region of the
hyoid bone) that is usually painless, smooth and cystic, though if infected, pain
can occur.
c) Thyroid cancer

d) Cystic Hygroma
63. Child ate overdose of iron several hours ago, Iron level 700, best immediate
management:

a) Gastric lavage

b) Induce vomiting manually

c) Emetic drugs

d) IV Deferoxamine

64. sickle cell patient…mechanism behind pain and lethargy….long scenario…

a. fatiguability

b. vaso occlusive crisis

c. decreased erythropoietin

d. hemorrhage

65. A boy who was bitten by his brother and received tetanus shot 6 month ago and his
laceration was 1cm and you cleaned his wound next you will:

a) Give Augmentin.

b) Suture the wound.

c) Give tetanus shot

d) Send home with close observation and return in 48 hours.

66. Long scenario Child with Duchenne muscular Dystrophy only given symptoms of diseases,
his mother pregnant and probably fetus is a boy. How is chance this baby to born with this
diseases?

a) 12,5%
x- linked
b) 25%
recessive
c) 50%

d) no any chance
66. Long scenario Child with Duchenne muscular Dystrophy only given
symptoms of diseases,his mother pregnant and probably fetus is a boy.
How is chance this baby to born with this
diseases?
a) 12,5%
b) 25%
c) 50%
d) no any chance
67. bmi chart was given of a 14 yr old boy…asking for conclusion…bmi 32.5

a. normal wt

b. over weight

c. obese

d. morbid obese

68. Drug used for bipolar, shizophrenia and shizoefective disorder

a olanzapine

b clozapine

c quitapine

d resperidone
69. pt sees diplopia while viewing right or left side…..where is the leision…..

a. cn 2

b. cn 3

c. cn 4

d. cn 6.

70. An old man using 6 hour computer/day presented with red eye itching foriegn body
sensation reason in his eyes, came to u, ur reason for this condition….

a reduce tear film

b viral keratitis

c cataract

d uveitis

71. case of bronchiectasis…..lung fibrosed and the patient very well knows that some part of
his lung is permanently damaged…what modality of treatment r u going to offer him…

a. lung resection

b. cpap

c. steroids

d. tracheostomy.

72,.. tension headache…band like…

73. pt with enlarged parotid gland with secretions increased over time….

best initial investigation…

a. usg

b. ct

c. x ray

d. mri

I COULD ONLY RECALL THESE MANY…THANK YOU .


‫ دﻋﻮاﺗﻜﻢ ﻟﻲ ﺑﺎﻟﻨﺠﺎح واﻟﺘﻮﻓﻴﻖ‬٣ ‫ﻫﺬي ﺗﺠﻤﻴﻌﺎت ﺷﻬﺮ‬
Newborn with eye hemangioma that occludes the eye
completely and your concern to
not develop amblyopia when to do resection (surgery):
.A. one day
B. one week
C. one month
D. six month

72 years old man diagnosed with gastroenteritis and


has been vomiting for the past 72
hours. What's released in the body?
A. CRP
.B. cytokines
C. apoprotien

patient presented with Hypotension, his phosphatase


in normal level, after one day his
phosphate level decrease, what organ damage?
A- Liver
B- kidney
C- lung

New recommendation for sudden infantile


death syndrome
A. Pacifier
B. Lying in prone position
C. Position support devices
Offer a pacifier to the infant at nap time or bedtime
Avoid the use of home monitors or commercial devices - such as wedges or positioners - that claim to lower SIDS risk
Ensure infants have received all their recommended vaccinations
Supervised tummy time when a baby is awake is recommended in order to aid development
57. What is the drug of choice for eclamptic sezure?
A..Phenytoin
b.. Diazepam.
C..Magnesium Sulfate.

SSRI that mostly causes overdose:


A. Sertraline Sertraline and fluoxetine were the SSRIs most commonly taken in exces

B. Paroxetine
C. Citalopram

Pt suspected H pylori
A blood
B stool
C urea breath

Which muscle would be completely paralyzed by


obturator nerve injury?
A. Gluteus Maximus.
B.Adductor magnus
C. Adductor longus

Best treatment of trigeminal neuralgia?


A) Carbamazepine The disorder is thought to result from compression of the trigeminal nerve root at the cerebellopontine
angle, most often by an aberrant vascular loop.

B) Prednisolone
MRI is typically required to exclude other compressive lesions, but is usually normal.
Treatment Carbamazepine Surgery

C) Naloxon

: Metformin act on the muscle by:


A) Increase glucose release
B) Stimulate fat oxidation
C) Stimulate gluconeogenesis
D) Increase glucose uptake

TTT of H.pylori infection:


a. Omeprazol 2 weeks, clarithromycin and
amoxicillin 1 week
b. Ranitidine , erythromycin, metronidazole for 2week

40yr old male pt with h/o syncope when he exercises


and on rest amd chest pain.on exsmntn:there s
ejection systolic murmur grade 2 -4/6on the left lower
sternal border not radiating and increases when lying
dwn..there is left atrial emlargmnt.
1.Aortic stenosis
2.Pulmonic stenosis
3.HCM
4.constructve cardiomyopathy
Man travels to sudan 2 weeks ago , now he is presnted
with fever , maleas .... unspecific
symbtom ( from 3 days ). How you will conferm
diagnosis??
A) blood culture
B) Serology
C) Blood film ...

short stature child. No family hx


Consisionl
genetic
hermonal...
Child presented with petechiae and his platelets is 15 ,
otherwise healthy. What will you
do for him?
a.Splenectomy
b. IVIG
c. Observations....
d. steroid

Pt present after traumatic event, he has


vomitus over the
face and his clothes, on exam there is depressed
skull fracture,
what are you going to do?
A. 100 % O2 supplement
B. Orotracheal intubation...
C. IVF
D. C spine collar

Patient came to ER, complaining of hypotension,


tachycardia and hypercapnia, ECG showed arrhythmia,
what is your immediate action?

A. Needle decompression .

B. Pericardiocentesis

C. FAST ultrasound
D. Thoracotomy ‫هنا اعادة‬

57. What is the drug of choice for eclamptic sezure?


A..Phenytoin
b.. Diazepam.
C..Magnesium Sulfate..

22-year-old male drug addict, visits the hospital


several times with multiple
complaints that are exaggerated. When he is not
focused he seems asymptomatic?
A. Somatization
B. Malingering...

New recommendation for sudden infantile


death syndrome
A. Pacifier
B. Lying in prone position...
C. Position support devices

SSRI that mostly causes overdose:


A. Sertraline
B. Paroxetine...
C. Citalopram

Most common labor complication in new born?


A. clavicle fracture...
B. shoulder dislocation
C. femur fracture

In brief: Randomized control studies between 2


populations with coronary artery
disease. What's the first question you should ask?
A. Was the age standardized?...
B. Are the medical facilities in both cities equal?
C. If the price of cigarettes cheaper in any of the cities?

72 years old man diagnosed with gastroenteritis and


has been vomiting for the past 72
hours. What's released in the body?
A. CRP
B. cytokines...
C. apoprotien

Pregnant, 16 weeks, discovered to be RH –ve,


what to do next?
A. ultrasound
B. amniocentesis
C. IV RH antibodies...

Patient in 3rd trimester have high blood blood glucose


despite close observation
What is the suspect cause?
A. neonate hyperglycemia
B.neonate hypoglycemia
C.mother hyper...
D.mother hypoglycemia

what is true about Pap smear?


A) 21 till 65 yrs repeated every year
B) 21 till 65 yrs repeated every 5 yrs
C) 21 till 35 repeated every 3 years
D) 30 till 65 repeated every 5 yrs....

patient presented to the ER with cough hemoptysis


night sweats and malaise. what is most appropriate
initial step in the management?
A.isolation in negative pressureroom...
B.start anti TB
C.give OPD appointment after 2 weeks

other question was there was a patent age 60 he has


hyperthyroidism he wants to know what long term
complications he'll
have....?
A) Brain CT
B) Liver CT
C) Bone scan...

Old male present with mid clavicle mass


( smoker for 20y and i
think he drinks )
What is your initial management
-Core biopsy
-Fine needle biopsy..
-bronchoscopy..

Inhalation of
cotton leads to :
A.....sarcoidosis
B.....berilliosis ..
C....histoplasmosis

deep tranvese arrest


occurs in

A......gynecoid pelvis
B....Android pelvis ....
C.....Anthropoid

Q/ MVA victim in the ICU developed multiorgan


failure lab show high
phosphate what is the organ responsible:
A) Heart
B) Liver
C) Kidney.. ..
D) Lung.

.2 weeks neonate passed unformed stool.


What will you do?
A. Prescribe formula milk.
B. Give oral rehydration solution.
C. Prescribe Lactose-free milk...

Giarda diagnostic test


A- 3stool parasite
B-3 stool culture
C-concentration test
D-immunoassay...

Long scenario .. Pt brought to ER with multiple


injuries in head . Chest,
abdomen , proximal upper limb is amputated ,blood
profuse, hypotensive ,
on o2 face mask 6L , what todo?
A. Tournique .....
B. Clamp
vessels
C. OR
D. mantain airway

Which drug should be stoped before IVP!?


Thiazides ...
ACEI metformin

CCB..

dull aching headache increase with straining


and coughing in DM
patient
A. Viral meningitis
B. Tb meningitis
C. Brain abscess
D. Sinusit...

the most important structure control BP is :


Heart
Capillaries
Arterioles..
Veins
Baby thirsty , tachycardia , sunken eye , dehydration :
%9-4...
%0
< %2
%9>

child fell on outstreched hand


pain in distal forearm
what diagnosis...
A.bartons
B.something
Colles is the answer
C.smith....

parentral Rx of falciprum malaria with 10%


parasitemia..
artemether
quinine....
primaquine
944. What makes the blood air barrier of the lung.
A. Pneumocyte 1....
B. Pneumocyte 2
C. Reticular Interstiti

case of streptococcal pharyngitis ,treatment?


A. Aspirin
B. Penicillin...

Q/ what move molecule in and out the cell nucleous?


A. cytosolic ATP
B. nuclear ATP
C. cytosolic GTP
D. nuclear GTP.....

Mumps complication in child ,?


Meningitis...
Enephalitis

Best prognosis in lung cancer?


A SCC SCC is the worst
Then large
Then bronchial carcinoid
B adenosquamous carcinoma
C adenocarcinoma in situ...

٢٢/٣ ‫اﻣﺘﺤﺎن‬

‫دﻋﻮاﺗﻜﻢ ﻟﻲ ﺑﺎﻟﻨﺠﺎح واﻟﺘﻮﻓﻴﻖ‬


16. Behcet disease is more
common in?
IJ . young men
---------------------------------
years child was born to HBS positive mother is HBS
positive , he was only vaccinated by BCG after birth ,
what you will give him now :
IJ polip+ mmr+ dtp+ hib
----------------------------------
257- 4 years old child developed resistance or not
respond to Chloramphenicol in Salmonella
infection, what you will give:
IJ . IM Ceftriaxone> children
-----------------------------------
Cause of death in marfan -
IJ -rupture aortic root aneurysm
-----------------------------------
Depression ..seratonin defeincy
---------------------------------
Q/ Posterior vagan nerve supply
IJ : esophagus
----------------------------------
Q/ Source of infection in venipuncture ?
IJ : Site of insertiohn
----------------------------------
Q/ different type of acne ,pustule .. what type of acne ?
IJ :Inflammatory
-----------------------------------
Q/ how to prevent MERSA?
IJ . by hand washing*
----------------------------------
Q/ In a study they are selecting every 10th family
in the city, what is
the type of study
IJ . - Systematic study
----------------------------------
Q/ about steven jonson syndrome is most likely
occur in which
group?
IJ . Patient with something take anti epileptic
----------------------------------
Q/ what move molecule in and out the cell nucleous?
IJ . cytosolic GrTP
-----------------------------------
Myopia with pathology > IJ pathological myogpia
-----------------------------------
Q/ what move molecule in and out the cell nucleous?
IJ . cytosolic GrTP
--------------------------------
Q/ MVA anterior injury of the pancreas and
anterior leakage of
pancreatic fluid will collect:
Omentum bursa
-----------------------------------
Protein last destination> Endoplasmic reticulum
---------------------------------
Q/ Pediatric case scenario indicate respiratory
distress syndrome ,
ask about the deficiency:
IJ dipalmitoyl
phosphotidylcholine*
-----------------------------------
Patient trauma to obturator nerve , which
muscle will be fully
paralyzed :
IJ Adductor longus
-----------------------------------
Q/ ‫اﻃﻮل ﻣﺮﺣﻠﺔ ﻓﻲ اﻟﺴﻞ ﺳﺎﻳﻜﻞ‬
IJ interphase
---------------------------------
case of characteristic rash (side effect of sulfa
drug)?????????
IJ blister
---------------------------------
case of cough , SOB, only in work place
IJ hypersensitivity pneumatis
----------------------------------
‫ وﻗﻠﻮﻛﻮﻣﺎ ﻳﺶ ﺣﺘﻌﻄﻴﻠﻮ دوا؟‬COPD ‫واﺣﺪ ﻋﻨﺪه‬/
Oral Acetazolamide .C
----------------------------------
Clinical dementia
IJ vit b12 def
-----------------------------------
Q/ Meckls diverticulitis most
common in ?
IJ Terminal ileum
-----------------------------------
Which anesthetic agent is 100 times stronger t
han Morphine?
IJ Fentanyl
-----------------------------------
Q/ An elderly, known case of Alzheimer's
disease, developed
hallucinations, bizarre behaviors and became
aggressive, what drug
to add?
IJ : resperidone
-----------------------------------
Q/ A pt drank an ethylene glycol containing
product, what renal
damage is suspected?
IJ ATN
----------------------------------
Q/ a pregnant lady was provided a pudendal
nerve block as an
analgesia, Which structure would be fully sensitive
and not blocked
by the analgesia,
IJ Rectum
-----------------------------------
30- Patient has HbF(5%) and HbA2 (5%)are present.
What is the diagnosis?
IJ BETA THALASSEMIA MINOR
----------------------------------
132. Dm , osteoarthritis on NSAID
came for regular check up .. Bp found to
be 160/some- thing, Invest:Inccr +_ urea IncbicrbDec
k nlna What is the cause
of htn :
IJ NSAID induced
----------------------------------
Q/ man with erectile dysfunction with normal
morning erections.
Where to refer.
IJ Pshyaiatry
----------------------------------
Q/ pt after a surgery in the posterior triangle of
neck, developed loss
of sensation on the skin of lower mandible and
Lower part of the
ear, nerve injured is
IJ Great Auricular nerve
----------------------------------
Q/ A pt with depression on medications found
comatose with empty
pill bottle beside her On invx; ABG showed Metabolic
acidosis, anion gab =18
, what drug overdosed?
IJ Aspirin
----------------------------------
Q/ pt with fever, altered LOC for 5 days, his
condition deteriorated
and developed body rash, and became jaundiced,
lab tests showed
low Hb, low PLT?
IJ TTP
-----------------------------------
49-Thereis a study to the effecton intensive insulin
regimen in the reduction of
neuropathy in the patient,the results are as
following:
Event in the regular insulin regimen:0.092
Event in the intensive insulin regimen:0.022
Which of the following is correct
IJ Odd ratiois 4.2
-----------------------------------
587) Patient was presented by back pain relieved by
ambulation , what is the best initial treatment :
IJ physical therapy
-----------------------------------
Which of the following made in nucleolus ??
IJ rRNA in nucleolus
-----------------------------------
Q276-Case of RLQ pain and mass, what is
the treatment:
℺ Antibiotics.
℺ conservative
Not sure
-----------------------------------
64. Old pt, diabetic dehydrated, lab high Na low
k, high bicarbonate, ketones in urine, Dx? A.
metabolic syndrome
B. diabetic ketoacidosis
C. lactic acidosis
℺ not sure
-----------------------------------
54.Pain anterior to the heel Worse in the morning and
better along with the day? Diag-
nosis?
IJ .Plantar fasciitis
-----------------------------------
Q loss sensation in thumb + index + ring finger
What nerve injured
IJ median
----------------------------------
162- Milestone baby can hold his head and when he
looks at his flying hand he laughs and coos?
Answer: 4m
----------------------------------
Q/ Antiviral taking by inhalation?
Oseltamivir (Tamiflu) 75 mg PO q12h for 5 days or
Zanamivir
-----------------------------------
Q/ Elderly with vasomotor rhinitis :
IJ . Decongested
-----------------------------------
Q/ dull aching headache increase with straining
and coughing in DM
patient
IJ . Sinusitis
----------------------------------
Q/ Close fracture to humerus type of nerve damage
in the radius.n!?
IJ Neuroprexia
---------------------------------
Q/ Inferior MI which one of the following
arteries blocked?
IJ : Right coronary artery
--------------------------------
Q/ 7 y/o every thing is normal Exept she has
pubic hair growth , GH
is normal , height & weight normal for the age , P/
E normal clit normal
size for the age no breast enlargement or
other 2ndary sex
characters, , what to tell the mother / dx?
IJ Early puberty
---------------------------------
Q/ Teenage Pt. Got a bad grade in an exam and
feels guilty, he try to
explain to everyone he meets/ talk with that the
exam was poorly
written& its not his fault that he got a bad grade?
IJ intellectualization
----------------------------------
Q/ 15‫ وﺑﻌﺪﻳﻦ‬.. ‫ اﻟﻤﻬﻢ ﺟﺎﻟﻬﻢ ﺗﺴﻤﻢ ﻛﻠﻬﻢ‬،، ‫ﺣﺪ راﺣﻮا ﺗﻐﺪوا ﻓﻲ ﻣﻄﻌﻢ‬
10‫ و‬.. ‫ﻣﻨﻬﻢ ﺗﻌﺎﻓﻮا ﺗﻠﻘﺎﺋﻴﺎ‬
.. ‫ ﺗﻌﺒﻮا ﻣﺮرة و ﺗﻨﻮﻣﻮا‬٧
‫ وش ﻫﻮ اﻻورﻗﺎﻧﻴﺰم ؟‬: ‫اﻟﺴﻮال‬
‫ ﺳﺎﻋﺎت‬٦ ‫ﺧﻼل‬
‫اﻟﺨﻴﺎرات‬
IJ Staph
-----------------------------------

Q/ Patient with end stage liver disease, on central


venous line, septic,
blood showed ood c/s budding yeast, what
antifungal is appropriate
at this stage?
IJ . Caspofungin ( my answer$
IJ fluconazole
( not sure)
----------------------------------
47- You are reading a population study that states
that 90% of lung cancer
patient are smokers while 30% of lung cancer patient
are non-smokers. What is
the specificity of using smoking as a predictor
of lung cancer?
IJ 70
-----------------------------------
Q/ Child with 25 degree esotropia, whats
your manegment? Surgery
-----------------------------------
Q MI treatment? Mona
-----------------------------------
Q TX of otitis media Amoxicillin
----------------------------------
Q organism in honeymoon cystitis E.coli
----------------------------------
Q clear scenario about HZV
and asked about Tx
----------------------------------
Q about effect of HTN OR DM on kidney ( not sure)
-----------------------------------
Q about about investigation in syphli FTA-ABS or Dark field
--------------------------------
Q about Slipped capital femoral epiphysis
Clear scenario and X ray
-------------------------------
Q Tx of septic arthritis
--------------------------------
Q about osteoporosis
-------------------------------
Q about prevention of traveler diarrhea
--------------------------------
Q about hallucination
Ask u about Dx
(Clear scenario)
----------------------------------
Q about cervical spondylosis
----------------------------------

ch nerve carries the referred pain of the parotid to


the ear? A. Vagus

B. Facial
C. Auriculotemporal
D. Trigeminal

Inhibition of which of the following is the primary


action of oral contraceptives?

A. Decrease estrogen to prevent the ovulation

B. Decrease GTRH spur at the mid cycle

C. Increase prolactin

D. Suppressing the release of gonadotropins


E. Spermatozoa and thickening cervical mucusa
:Q. Patient came to ER, complaining of hypotension,
tachycardia and hypercapnia, ECG showed arrhythmia,
what is your immediate action?

A. Needle decompression

B. Pericardiocentesis

C. FAST ultrasound

D. Thoracotomy

Epidemiological study want to see the effectof


smoking in lungcancer:90%

of lung cancer patients are smokers while 30%


of those who don’t have the

disease are smokers. The specificity of smoking as


a risk factoris:
A- 10%
B- 40%
C- 30%
D- 70%
E- 90%

a daycare center 10 out of 50 had red eye in the


first week; another 30
developed the same condition in the next 2
weeks. What is the attack rate

(cumulative incidence)?
A- 40%
B- 60%
C- 80%
D- 20%

persons foundto have Typhoid Fever in a population


of 20,000 during

the last 4 years.Ifwemeasure it on a population of


100,000, what will be the
incidence in one year?

What is the incidence rate for10000 population,


2000 old cases, and 1000

new cases?
12.5

Cohort study – 2 groups: one of them exposed to


patient with positive

MERSAand the other groupis not exposed to these


patients.
What is the ratioof
exposed to non-
exposed?
A- 44:1
B- 55:1
C- 66:1

- Lung cancer affected 80 of 100 smokers, 6 of 500


non smokers, What is

Relative risk reduction?


A- 33:1
B- 55:1
C- 66:1

Incidence is 117 per 1000 per year. Median


disabilityis 2.5 Medianlost jobs

is 1.6, what is the prevalence of disability?

2900.5
Patient with end stage liver disease, on central venous
line, septic, blood showed ood c/s

budding yeast, what anti-fungal is appropriate


at this stage?
A. Caspofungin
B. Flucytosine
C. fluconazole
D. Itraconazole
Patient k/c of HF & schizophrenia on medication,
came with brownish discoloration in retina & brown
vision.. what's the drug?

Answer: Thioridazine

A patient with eye movement abnormalities and


ptosis.What is the nerve is involved?

a-3rd
b-4th
A patient known case of COPD and glaucoma
complaining recently of intermittent cough?

a-betoxolol

b-Timolol

Pt stik her eyes by her finger and came with pain


burning and crying what the

suspected symp- tom or complications :


a- Blindness
b- Photophbi
c- 2 other

picture of a patient with redness of the eye


and tearing ,
what is the most likely you will find?
a- Papilla
b-Follicles
c- Papilloma

Patient with vesicles in forehead and supraorbital


region for one day, wt will u do ?

a. Antiviral
b. Antiviral and refer to
ophthalmologist.
c-Reassure

increase effect of analgesia?

A- Metoclopramid

multipara pregnant. Medically free.. she is term..


effacement 90% ,4 cm,regularcon-

tractions, spontaneous rupture of membrane, suddenly


baby is stress from 140 to 80

beats.which type of anesthesia would you use:


A) General anesthesia
B) Narcotics
C) Pudendal n
D) upper uterain
[٢Bilateral glucoma.case.....rt eye.. 40 ..IOP...lt..
22..IOP...pani.....typical cas

case lacrmial gland tuomer ask wt mucle


affِ‫د‬ected.............exteaorpital
ِْ ِ muscle

Case ptosis loss of in and down.....nerv injery.....


3red+4 ..nerv

Pt with shock &Cherry red skin;

A. Bacteremia
B. Septicemia
C. Carbon monoxide toxicity

Patient ate from a restaurant. 2 days after that


started to complain ofdiar- rhea, vomiting

and urticaria.

a. Food poisoning

b. Food allergy

HorizontalFissure: 4th Intercostal Space ORinferior


border of 4thRib
2) Oblique Fissure: 6th Intercostal Space ORinferior
border of the 6thRib

Case scenario about patient who had injury in his right


knee, the doc- tor applied a

valgus stress to examine which ligament


A- Anterior cruciate ligament
B- Posterior cruciate ligament
C- Fibular collateral
D- Tibial collateral

9-Patient complaining of severe uncontrolled HTN,


“Renography” not sure”, showed Lt renal

artery stenosis, next step is to?


A- Venography
B- IVP
C- CT angiography
D- Renography

Urge incontinence principle management?

A-Medical

B-Surgical
C-Medical and surgical
D-Bladder training and physio
Pt had pyelography showing rt kidney 7 cm and left
kidney 16 or 14 cm, arteriography

shows rt renal stenosis, Next management?


A- arteriography
B- Lt kidney percutaneous biopsy
C- CT angiography
D- CT abdomen

patient presented with hemoptysis and signs of


nephropathy. Biopsy of the lung showed

pres-ence of anti-GBM antibodies. What is the


most likely diagnosis?
A. Rheumatoid arthritis
B. Systemic lupus erythematosus
C. Goodpasture syndrome
D. Acute glomerulonephritis

Which in lipid profile is most important risk for


coronaryheart disease?

A. LDL,
B. HDL,
C. Triglyceride ,
D. total cholesterol

64-the most effective nonpharmacological treatment


for hypertension??

a-Low sodium
diet
b-Decrease Wight
]

26.MI patient within 6 hours what is the most


expected complication :

A.PE

B.Arrhyth

Patient came with ECG of narrow complex tachycardia


with no P-wave and regular

rhythm what most important to test?

A) TSH
B) EPS
C) Coronary angiogram
The best way for lifestyle management for
Hypertensive patient:

A) Na restriction less than 6


B) Weight loss
C) Exercise
D) Swimming every day

coarctation of aorta with ?

A) down syndrome
B) Turner syndrome
C) Edouard syndrome
D) Patau syndrome

pt has HTN control on his medications , developed


albuminuria , what should you add

to his HTN medications :


A) ACEI
B) BB
C) Lasix
D) digoxin

MI patient has big thrombus in left coronary artery


what is drug that cause
throm- bolytic action?
A) TPA
B) Heparin
C) Clexane
D) Dabigatran

Or with MI he is on nitroglycerin and morphine .


After 20 minutes he

got worse with raised JVP and no lung


finding. BP drop.
A) Ruptured
B) Arrhythmia
C) RV infarction

A 50-year-old male diabetic and hypertensive,


unable to maintain an

erection, start- ed on Phosphodiesterase type 5


inhibitors. Which one of the
following drug classes should be avoided?
A) Steroids
B) Antibiotics
C) Narcotics
D) Nitrates

Elderly Patient presented with chest pain he is a


smokerwith positive family history of
coronaryarterydisease and he is dyslipidemic he is
also overweight and can not tolerate exercise

hisECGis normal what will do:


A. Stress echo
B. Stress test
C. Resting reperfusion scan

164. healthy woman travelled for 18 hours,


examination were completely

normal, investi- gation done and D dimer was 350, they


give the normal value
less than 250 what is the appropriate management:
A) Heparin
B ) LMWH and heparin
C) Unfractionated heparin

Which of the following is side effectof atropine:

A. Vasoconstriction
B. Decrease iop
C. Decrease urine output
D. Dry of mouth

Ecg with AFib pattern. A patient present with


irregular cardiacpalpitation some duration in

the pastforwhich he was taking a drug, lab values


given demonstrating normal upperlimit of aptt,
normal pt, normal inr,normal platelet, low hemoglobin,
what is the mechanism of action of the
drug?
A. Anti-thrombin iii
B. Decrease factorviii
C. Inhibit platelets aggregation
D. Decrease vitamink dependent factotminal

Case scenario about patient who had injury in his right


knee, the doc- tor applied a

valgus stress to examine which ligament

A- Anterior cruciate ligament


B- Posterior cruciate ligament
C- Fibular collateral
D- Tibial collateral

217- Ecg with AFib pattern. A patient present with


irregular cardiacpalpitation some duration in

the pastforwhich he was taking a drug, lab values


given demonstrating normal upperlimit of aptt,
normal pt, normal inr,normal platelet, low hemoglobin,
what is the mechanism of action of the
drug?
A. Anti-thrombin iii
B. Decrease factorviii
C. Inhibit platelets aggregation
D. Decrease vitamink dependent factor
- Pregnant, 10 weeks with HTN never went to doctor before (chronic HTN) what is the
complication? Pre-eclampsia ‫ واذا مو موجود‬، ‫ تكون هذي االجابه ادق‬IUGR ‫اذا كان في السؤال أجابة‬
Pre-eclampsia
- (Long Case)... IUGR , witch one of the following can be the cause?
Oligohydramnios
- Reflux esophagitis , which ding may aggravating this ?
Theophylline (anticholinergic)
- which true for breast screening?
Women (50-70) mammogram each 2 years (grad B recom)
- What's P0.3 mean?
No difference between 2 drugs
- what is the meaning of confidence interval 95 %?
Mean population
-Q about gastroenteritis.... Widal test + ve, which of the following cell will be effected?
payers Cell
-Q about menopause had fibroid before 4 years 4x3 cm come now with bleeding and in US
the fibroid size 7.6 and endometrial thickness 6 (normal <4) what is the cause?
Endometrial cancer
-Pt on ttt for meningitis with antibiotics, came later with lab result [all CBC decreased ] ( this
is a plastic anemia ? ‫السؤال وش السبب ؟ السبب انه استخدم‬
Chloramphenicol
ttt for meningitis, Chloramphenicol

-Pt on ttt for hyperthyroidism came with [ neutrophils, monocyt , basophil ]


agranulocytosis NL RBC +PLT what is ttt?
Methimazole
-Bleeding after cut of umbilical cord what's the cause?
Factor 13 deficiency
-30 year old female pain with period and breast mass and disappear after period
what is your next step ?
Re. evaluation next period
-In window of hepatitis B which test is (+ve) ?
Hep B c, antibody ve
-Elderly pt with pelvic pain + normal ROM pain with abdication (AVN) , your
next step ?
Radiological image
-Knee or leg pain decrease with passive stretch , which mnsd ?
Gastrocnemius
-Soldier with pain at medial scle of foot with Hx of walking for long distance,
which ligament is affected?
Spring Ligament
-Mutiorgan failure after trauma lab [ high phosphate] which organ can cause
this ?
Kidney
-Pt recurrent abnormal position or movement and no one can change it what is
Dx?
Catatonia
- 22-year-old male drug addict, visits the hospital several times with multiple
complaints that are exaggerated. When he is not focused he seems asymptomatic?
Malingering

Pt on medication for HTN that , co execration ?


Hydrochlorothiazide
-Pt with high hypertension lab [metabolic alkalons + hypokalemic +normal na] ur next step ?
Kidney us
-Pt with recurrent hemartharosis , lab [factor Vlll =0.2 normal range (50 -200) what is ttt?
Factor Vlll , this sever hemophilia A (1% -5%)
Mild hemophilia, > 5% ttt davab
-Urge incontinence ttt?
Kegel exercise ??? antibiotic incide, ?? surgery
-Pt with headache lab [ESR] Dx?
Temporal arteritis
-Pt pain in eye + redness + lymphocyte , Dx ?
Orbital cellulitis
-Elderly eat fibers + change in bowl habit , came with anemia microcytic hypochromic , what
is the cause ?
Change in dite [ high fibers cause iron defainanemia ]
-60 yr pt come with anemia normocytic normochromic anemia [MCV ,NL (80-100) , LDH ],
what is the type of anemia?
Hemolytic anemia
-10 weeks pregnant come with weight loss 2 kg +fatigue + appetite + vomiting after each
meal what is the. cause of these symptoms?
H. Pylori
-Elderly with symptoms(renal cell carcinoma ) Fremantle cell carcinoma ( hematuria ,
abdominal mess , pelvic or lumber pain) what is the best test ?
CT
-Witch of the following cause newborn meningitis or sepsis ?
GBS or E-Cali
-Pt on antilipid drug (hyperlipemia drug) come with flushing face , with aspirin , lab [HDL ,
LDL - CK] Witch drug cause this ?
Niacin
-Pt with symptom + Alzheimer + sometimes be agitation, what is the cause?
Alzheimer
-Ig can cross placenta?
Ig G
-Can't elevate hand drop wrist?
Radial nerve injury (in spiral grove of humans)
-Fixed belief against culture and intelligence, definition of what?
Delusion
-Newborn baby has SOB + no heart sound + usual site ... ?
Diaphragmatic
-Abdominal pop smear than in colposcopy can't found the lesiar for biobsy not clear , what is
the next step ?
Conebiopsy
-Child with gingival bleeding + hypertrophy in gingiva ,Dx ?
Vit-c deficiency
-Children BMI is high what the next step ?
Full skin folded thickness
-Man want to do study on group of things about lung cancer what is the type of t he study ?
Meta analysis
-Case control study what is its bias ?
Recall bias

-Relative Risk ?
A/(A+B) / C / (C+D)
-Pt sudden fell down no breathing his pulse is raped and weak , next step?
Intapation
-Pt getting better after naloxone , toxicity from ?
Morphine
-Pt with head fracture what is the first step ?
Airway
-Q, redness color in the shoulders since breath ,Dx ?
Strawberry nevus
-34 weeks pregnant came with breech presentation what is your next step?
Flu 36 weeks -ECV
- Pregnant in her 38 week and 2 weeks ago she was admitted and ECV was done
for her. Currently the fetus position is in linear (or lateral not sure) with amniotic
fluid index 12.
What’s the contraindication for ECV in her condition?
fetal position
-absolute eosinophilia, what is the cause?
Parasite
-Child drink juice then diarrhea?
Fructosmia
-17 year old runner (Athlete) when preue of knee cab kid pain Dx ?
Osgaud
-Case of TTP what is the response for it , 27 girl came with less of consonants of seizure +
hematuria + petechial rush + lab [throbocytenia] ?
ADAMT
-14 yrs before one week has URTI now come with non-blanchable rash , what is the cause ?
Autoimmune thrombocytopenia purpura ATP
- DM patient with bilateral lower limbs pain with crusts?
stasis dermatitis

-Elderly has swelling in metaphalogeal and proximal interphalngal at morning and


with walking. Dx?
Rheumatoid arthritis
-Sulfa drug come with blaster
-Bilatral inguinal swelling + painful in genitalia +cervical lymphadenopathy Dx?
lymphogranuloma venereum
-Baby drowning his brother undressing clothes witch year?
5 years
-Baby retate 180 and can't set , coming smile for doctor?
4-5 months

-Pt on short acting B2 against (salbutamol) + in healed steroid also have recurrent
attacks asthma on clinical give broncho inhaler (bronchodilator) penk expiratory flow rate
(PEF1) increase from 60% to 90% , what is ur next step ?
Observe how to use inhaler
-Neck pain radiated to shoulder
disk prolapse
-Pt with back pain CT show spondylitis change (spondylitis) ttt?
physical treatment
-Pt with DM on metformin controlled everything is ok , his BP measured 3 time was 138/89
what's RX ?
Add ACEI
-40. year female primigravida with Hx & DVT . RX ?
Heparin
-Female with vaginal discharge + nucleic acid amplification test (+ve) , Dx?
Gonorrhea
-Female come with grayish vaginal discharge + PH Dx, bacterial vaginosis what is ttt ?
Oral metronidazole
-Apoptosis witch gene ?
P53
-Pt work on cotton comeback with shortness of breath + lymphadenopathy :
Byssinosis ‫مو متاكده من السبلينق‬
-When to repeat -ve HIV test ?
After 3 months
-Best image for Dx intussusception :
Us (test of choice)
-Pt with dehydration +BP 80/80 what is the first step?
Fluid
-Baby with recurrent diarrhea + his capillary refill more than 3 sec + his BP 80/40 (sever
dehydration) next step?
20ml/hr/kg bullous
-Baby brought by his mother after 7 days from delivery his weight was 3.5 and now 3.1 , why?
Normal
-Mom just eat polished rice , what is the deficiency?
Vit . B1
-10 year baby came with diarrhea sometime bloody + joint pain + IDA , what is ttt ?
Dx : crohn's disease , RX : Aminosulicylates
-Bacterial meningitis lab ?
high Neutrophils, high protein , low glucose
-Pregnant female 10 weeks came with vaginal blew + abdominal pain a examination
gartational age 11 -12 weeks , what is the cause ?
Molar pregnant or incomplete abortion
-Pt with migraine she doesn't want daily drug , RX ?
Triptans
-Female has painful period she use NSAID she want stronger medication what to advice her ?
Exercise and relaxation.
-Pt with headache same as tension headache band like + stress for one month and use
paracetamol day after day . What is type of headache?
over use drug headache
-Pt with dysphagia pain + enlargement in one tonsils, Dx?
Quinsy (peritonsillar abscess)
-Which of the following potent antipsychotic drug?
Haloperidol
-Witch of the following cause metabolic syndrome +weigh gain ?
Olanzapine
-Doctor diagnosis pt with schizophrenia to Dx ( it must duration be 6 months )
-Pt with symptom of schizophrenia (hallosenation) what to give ?
Risperidone (antipsychotic)
-70 year old pt with vasomotorrihinitis and chinorrhea, ttt?
Ipratropium
-Pt with symptom of otitis media redness bulging tympanic membrain, ttt?
Amoxicillin + calvulanic acid
-Complication obstructive sleep apnea ?
DM
-Case about adjustment disorder what is ttt?
Supportive therapy
- OCD , what ttt?
Exposure and prevention therapy
-Young pt with discharge from unilateral nose + foul odor , ur next step ?
X-Ray for head and chest
-Pt with periorbital and on nose rash and painful (Dx, herpes zoster ophthalmicus , ttt?
Acyclovir and refer ophtha
-Pt with RA treated by INFH witch disense can recurrent to him when use this drug ?
TB
-Osteoporosis, best exercise?
Weight beaning exercise
-MG come to ER , he is on pyriodostigmine what to give him in ER ?
Plasmapheresis
-Female over thinking with tachycardia when she relaxed , tachycardia and anexiaty, what
cause ?
Norepinephrine
-Old pt asymptomat + fatigue lab [CLL] what is ttt?
No treatment
-Pain and discharge from eye when pt cough , why?
Mechanical pressure in retina
-Pt DM came with hemoptysis and chest pain (infection) found to have non septette hyphae
fungal, witch fungal non septite ?
Zygomycosis septite high fungel : pasiodiomycets
-Pic of swelling redness painful in upper eyelid .Dx?

Styes

-How to effect of intathecal anesthesia?


By tremblenburg position
-Atopic dermatitis best thing to Dx?
Clinical
-21 year with asthma with ulcers on knee or other site Dx?
Atopic dermatitis
-Parasite and TB witch cytokines response?
IFY
-Reservar for HIv ?
Macrophage
-Pt on NSAID for 2 weeks come with medium and epigastric pain or upper GI bleeding what is
the cause?
Peptic ulcer
-Witch of the following characteristics for cholecystitis ?
RVQ pain
-Hemeplagia and straipsnis or esotrapia , cause?
Pons
-Pt with DM witch medication to give before sleeping?
MPH ( milk protein hydrolysate )
-Female with problem in chest go for mastectomy we will take from abdominal muscle for
graft witch artery is response for it ?
superior epigastric
-End stay liver disease + fungal infection

capsofungin
-Energy in and out of cell

GTP cytoplasmic
-Poor healing in DM due to

in decreased phagocytosis
-Pt on ACEI not control what to add ?
CCB
-Save in pregnancy with DM ?

metformin
-Mechanism of action metformin?
Phosphatase kinase
-Pt on metformin and sulfonylureas but not control, what to add?
acarbose ‫ سوال بعد لو نلقاهم احسن‬15 ‫قد جاء سؤال في الهيئة كيذا من‬
-Pt allergy to sulfa what to add ?
DPP 4 inhpotor (-tide)
-Pregnant BP 160/110 at the end of pregnancy, what is the next step ?
Mg so
-Pregnant with pre-eclampsia mild 140/40 with abdominal pain .platelet and , uricaerd what
indicate severity?
Abdominal pain

-Pic of corneal abrasion , asking about the dye

Fluorescent

-Complication. of strabismus?
Amblyopia
-Pt with problem in mandible?
great auricular nerve
-Pt after surgery loss of sensation of upper lip

infraorbital nerve
- loss of sensation in out 2/3 of tongue

Trigeminal nerve
-Asthmatic. pt present in the ER witch investigation indicate security of asthma?
O2 pressure
-High myopic ttt?
Keratotomy
- Pic of baby with umbilical mass?
Umbilical granuloma
-Case give lab and ask about acid base balance ?
Normal aniongab metabolic acidosis
-2 Weeks old baby with violent vomiting + epigastric mass , next step ?
Us (piloric stenosis)
-Pt with hyperaldosteronism [normal renin -BP -aldosterone] what's the cause in the adrenal
cortex for this ?
Glomerulosa
-MG mechanism?
Antiacetylcholine receptor
-Pt on medication came with abdominal moment ( head and eye) Dx . dystonia what's the
medication?
Metoclopramide
-Elderly female came with weight loss + constipation + pelvic mass , best initial screening?
US

old.lady.known.case.of.hypothyroidism.present.with.painful.movement.of.the.
right .shoulder .and. can't. raise. the. shoulder. due. to. This .pain
,what's .the. most. likely. diagnosis?.
Adhesive .capsulitis.

- Which of the following primary cause of osteoporosis ?

Aging

Case of osteoporosis …… ttt ?


Alendronate (Bisphosphonate )
- Case of cardiac tamponade , hypotensive …..- voice not clear – trauma, distend
venous neck , next step ?
Pericardiocentesis
- Type of cancer in keratin ?
SCC
- pt with hypothyroidism and vitiligo, lab (hypocalcemia , normal po2) will cause ?
panhypopituitarism
1.pte with copd and co2 9 what u wll do

mechanical ventilation

reduce o2

antibiotixs

2.55 years old wit AF what u wll do to reduce the complica=ons

warfarin

pt with atrial fibrilation on warfarin develop inracranial heamorrage what u wll do

vitamin k

FFP

HEPARIN

PT develop orthopnia PND chest auscultation basal crips she deliverd one week ago what is ur diagnosis

perpartum heart failur

IHD

ashma

ashma=c pte have no exacerepa=on in last mounth his dialy PEF is 600 what is indicator of sevirity in this
pt

RR 25

PFR 240

ashmatic child his family stat that thy can not go out becouse the nebuluzer machin need electicity what
u wll do to help them

tell them they are right

provid them with nebulizer with battery

clearly instruct the inhaler and tell them to stop the nebulizer

old pte admited becouse of pneumonea at night she develop restlessness she is disorinted what u wll do

physicaly but her in her bed

call her family to set besid her


lorazepam 4 mg stat

pte with cholecystectomy develop pain in the angle of the mouth what u wll do????????

u did appendictomy and you found it not inflamed you remove it and closed what the next

tell the pte

tell the ethics

tell the surgery head

you r doing appendictomy u found bus and fluid in the RIFwhat u wll do

remove the fluid and appebix

remove the apendix

pt presented with some coplaint potasium wass 7.9 what u wll do

dialysis

insulin

Ca gluconat

pte on dialysis develop shivering he men=on history of fever 3 day ag

you sow pus drainag from the catheter

what u do

.take blood for culur and give AB

blood culure ABand stop dialysis 3 days

change the casheter

pet wit URT 3 days develop shortness of breath PND LLedema troponin is high what is apropriat step to
rech the doagnosis

echo

ecf cxr

mri

pte with AAA 4.5 CM have cholecystectomy vascular surgery councled what he wll say

proceed the surgery and folw up with US


PROCEED THE SURGRY FOLOW WITH CT

CT THEN DO THE SURGERY

PTE WITH recurent abdominal pain us show dilated intrahepa=c duct tembretue 38 pain increas with
morphin LFT Normal ALp high what is diagnosis

cholelethiasis

choleangitis

cholecystitis

pte have epogastric pain after vomiting cx show some pleural effusion what is diagnosis

rubture eosaphtes

peadiatric pte with testicar swlling translumination negative what u wll do

mish repair

herniotomy

hernigraphy

peadiatric pte with bilatral testecular swlling increas when he is crrying what to do

herniotomy

mish repair

laproscopic mesh repair

ptw with abdo mass after lefting heavy object mass not change with cugh

what 9s diagnosis

rectus muscl heamatoma

hernia

pte with testicular pain and vomiting scrotum is red what u wll do

US

urgent surgry
pregnant in her 37 week faint and complain of sever abd pain no pleedin pb 80/50 HR120 WHat os
diagnosis

pulmonary empolism

abruptio placente

couple seeking fwrrility clinic a=er 3 mounth what is ur advise

try more

pte with breast cancer and fibroid on tamoxfin develop vaginal bleeding USshow fibroid 8cm and
hypowchogenic area what is diagnosis

endometrial cancer

lyomyosarcoma

overian cancer

65 years old femal presented with v bleeding and abd pain Ca125 veey high what u wll do

biobsy.

antibiotic

admit

pregnant lady with recuren pain known case of gallstone what wll do

surgery in the second trimester -! belter laparoscopy in 2nd trimester

surgery after delivery

pregnant lady complain foul smelling vaginal diacharg what u wll give

amoxill

cephalexin

metronidazole

hydit cyst reatment !pendazol????

ameobic liver abcess treatment ! metronidazole

bacterial vaginosis treatment ! metronidazole

pte withe placental abrub=on BP 8/40

what u wll do

us
admission foe sciu under

maximum response team and multidisiplinary

pte with ectopic pregnancy what is pest quistion befor managment of the pte

accessability to hospital

previous surgery

->Complete mole direct quistion

[ Patien has cafe alue spot in his back what you will.do next

1. Council.the pa=ent about neurofibrmatosis

2.send the pa=ent to genteics to do gene=c

[Patien has subcutanuoes nodule and arthrlagia what one of follwoong confirm diagnosis of Rhematic
fever

1. High crp

2.high esr

3.postive blood culture

[Pa=en has st eleva=on in v1 v 2 v 3 what is your diagnosis

1.mi

2.pericardi=es

3.cardiac tamponade

Pa=ent age of 4 years convulsing at home for 3 minute he has tonsili=es temp is 38 what you will do
next

1. Diazepam

2.lorazepam

3.amoxicillin

4.paracetamol

Patien fall in out stretched hand ...pic of open colles fracture what you will.do next
1. Open reduc=on

2.close reduc=on

3.depridment with open reduc=on

[ Pregnant complain.of painful vaginal bleeding is it

1. Placenta abrtpio

2.placenta previa

[Pregnant at 20 weak complain of vaginal bleeding os closed ultra sound no fetus.....complete abortion

[Pregnant with bleeding os open ...invitable

[ Bleeding with os closed threatned ...bed rest ttt

[Magnisium sulfate givun to htn pregnant to.

...prevent the attack eclamsia????

Copd excareba=on and co2 reten=on and consious.....NIMV

Un consious copd excacerbation ......mechanical ventilation

[ Perecardities first drug to give....non steroidall antiinflamatory drug like ibuprofen

Celiac disese should avoid ...gluten conatin diet like u should knw the type of food contain gluten

Meickle divertixlum.....end of ileum 6cm

[Pt with right ilac fossa pain with bcg negetive.....acute appendicites

.....

[ Fishy smell....bacterial vaginsois

Gray white discharge....trichomanis

Secer headache ever...subarachnoid hemorrhage

Hypernatremia with normal potasium .....siadah XXXXXDM Water retention

Olive.....pyloric stenosis Target sign....intusseption US

1-a myocrial biopsy was taken from woman because of unexplained congestive HF: biopsy showed
Antischkow cells:

-rheumatic fever
-SLE

-Sarcoidosis

2-Patients with sx of BV:

What’s the test: gram stain

3-Case of Trichmonisis what’s the dx

4-and what’s the treatment (metro)

5-You are seeing a 78 yrs old lady in clinic. She looks sick had significa=on weight loss and feels =red all
the time. She has pelvic mass. What the the most appropriate screening test to confirm the diagnosis:

US

Colposcopy

Ca-125

6-Patient with DLP on statin, lab results for lipid profile all within normal range except high alanine
aminotransferee (48) ‘normal up to 40’

What you’re going to give this patient:

Fibrate

Cholestyramine

Ezetimibe

Omega 3

7-Pregnant lady in her first trimester, she was exposed to rubella 3 days ago, what you’re going to do:

Nothing

MMR
Terminate the pregnancy

Rubella Ig

8-Patient with MG crisis: plasmapheresis

Patient with fatiguable weakness( sx of MG), whats the pathophysiology of this condition:

Antibodies against Ach receptor

9-Patient with unilateral knee pain and urethral discharge of pus. Negative culture of the pus:

Riter syndrome

10-Patient with back and buttocks pain that improves with exercise:

Riter syndrome

Reactive arthritis

Ankylosing spondylitis

11-Patient with sx of gastroenteritis:

Rota,

Norovirus

Adenovirus

12-2 Qs about staph aureus food poisoning

13-Clostridium perfengis after surgery- Infection- gases Gangrene

14-End distention of protien (ER, cytosome, golgi, lysosome)


15-Hydrogen bond btw mRNA and rRNA

16-worse prognostic gene with HCM

Myocin binding protein C

Beta myocin

17-Parklond formula, how much fluid are given in first 8hrs for anterior trunk and circumferen=al le=
upper and le= lower limbs: 6.3

18-patient with hight parathyoid hormone and hypercalcemia, what u will do next:

Look for parathyroid adenoma

TSH, T3 and T4

19-Patient with breat Ca, polyuria and increase thirst, she has also Hypercalcemia: what will u give her

Normal saline

Furosemide

Amlodapine

20-Case presentation of Addison, what u will find:

Hirsutism

Hyper pigmentation

Buffalo hump

21-Case presentation of hyper aldosteronism, what u will find:

Polyuria
23-42 yrs old male came for general check up, BP and Fast blood glucose are within normal ranges his
BMI is 28 and he is asking about ur advice:

Wt reduction and exercise

Exercise

Low Salt diet

24-Patient coming for general check-up his BP is normal and has family Hx of cardiac diseases, what u
will do for him

Lipid profile

25- patient coming to see the results of INR, what the drug he is taking:

Warfrain

26- best prophylaxis for thrombosis post surgery is

LMWH

27-75 yrs old male Pa=ent with DM and HTN and hx of 2 stroke events in the past. Coming with Afib.
What u will do for him:

Warfarin to prevent any stroke event

28-Patient with bph and HTN, what u will do for him:

Alpha bloker

TURP

Open prostatectomy

29- patient on methtroxate, what would be deficient in him:

Folate
Niacin

Vit E

30-Case of rheuma=c fever (A 7yrs old boy presented with knee pain. On examina=on, he has heart
murmur. His parents told you that he had tonsilli=s 3 wks ago and treated with paracetamol. His ASO
titer is high. How u will treat:

-Penicillin

-steroid->-> sever cardites

-aspirin

31- patient presented with SOB on exertion and pallor. His CBC showed low Hb, normal MCV and high
LDH

Hemolytic anemia

Microcytic

32- patient with jaundice after quinine treatment. What the deficient enzyme:

Glucose-6 phosphate dehydrogenase

Glucose dehydrogenase

Glucose 6 kinase

33- pa=ent his bone marrow bx shwoed 80% blast cellls and Auro rods:

AML

ALL

CML

CLL

34-most oxygenated zone of the liver is ->zone1


Zone 1 - periportal (centroacinar), around the portal areas

Zone 2 - midzonal

Zone 3 - centrilobular (periacinar), bordering the hepatic venules

35- patient with presentation of generalized anxiety disorder, what is the best drug for her:

Lorazepam

Bupropion

Buspirone

Citalopram

36- patient with presentation of grvis disease, what’s the pathophysiology:

Antibodies binding to TS receptor

37- case of otitis media and asking about the diagnosis->pain

38- patient with ase of otitis media and erytham on mastoid bone, what’s the treatment:

Amoxicillin

Cefoxaim

39-Case 7 yrs has pupic hair no breast devolpment : normal 8

premature adrenarche

40-First trimester miscarriage at 5 week with history of 2nd trimester abor=on twice duo to cervical
incompetence , cause now :

Cervical incompetence->2nd

Chromosomal abnormality ->1st ->11week

41-On antidepressant succide bottle beside her has dilated pupil:

amitriptyllain

Baro recpter activation will

decrease venous capictance


43-19 yrs old lady with Diarrhea for 10 months, wt loss, postprandial periumbilical pain. Diarrhea
sometimes bloody

Crohn’s

Peptic ulcer

Chronic Pancreatic

44-Fibroid african race % ?????

45-Increase total cholesterol + high LDL + high triglyceride + low HDL ... ask about the follow up a=er 1
year

Blood pressure

Lipid profile

46-Monitor of asthma in ER

FEV1

Heart rate

partial pressure of oxygen

47- A=er stopping the OCP, pt s=ll have amenorrhea for 6 months, gain 5 kg, feel =red. labs show
elevated glucose

Hypothyroidism

Cushing syndrome

49-case female pregnant S=ll born at 38 wks every thing normal then discharge but a=er 3 days come
back with bleeding from every site injection Which best investigation ? DIC

Hemoglobin electrophoresis

Fibrin product

50-Patient with presentation of hemolytic uremic syndrom HUS. What u will do: conservative
Hemodylasis

Abx

Platelets transfusion

51-10 student with pneumonia (legionella, mycoplasma, strep ) Endamic

52-Painful axillary lump with tender and erythematous with black head papule and large pores?

Oral Abx skin infection ‫ر‬#$

53-Anterior axillary for breast cancer

54-RSV in bronchitis

55-HIV latent: CD8, b memory cell, macrophage %&'(‫ا‬

56-Stillbirth 20 wks

57-stillbirth Alpha thalassemia: 2 normal b and abnormal 4alpha

58-Bisphosphonate in osteoporosis

59-4 days boy passed meconium in first hour, poor feeding since birth. Today has vomiting and Abd
distention: Midgut valvolus, hirshpurng supportive ttt

60- pt in ER starring at man and says u can’t kill me: Hallucinating

Patient with erectile dysfunction, has depression. What drug should be avoided? Sertraline

61-Patient with MI. What’s the mechanism of pain reliever:

Stimulate CNS Mu receptor Morfen

62-Glaucoma treatment 2Qs????? Biocarbenttt

63- Achalasia case

64-cricopharngeal Dysfunction ??? #*+ ,-'.‫ ا‬upper esophageal sphentar

65- pt with Croup. What u will hear in exam:

Inspiratory sound due to subglotic narrowing

Inspiratory sound due to secretion


66-TOF components

67-middle aged with Cyanotic disease. His x-ray showed enlarged pulmonary artery and increased
vascular markings ;

Pulmonary stenosis

VSD

Truncus arterious

68-Clear case of Nephrotic syndrome

69-steroid sensitive age in minimal change disease:

6-12 yers

12-48

12-96 month

70-Caspofungin->>>> anti fugal liver F

71-SSRI in pMS paronstinttt????

72-Most common affect (behavioral) symptom in premenstrual dysphoric disorder?

Irritability ???????????????????????

Mood swings??????????????????pre

Depression

Anxiety

73-Patient who had hysterectomy and hot flush after the surgery

Transdermal Estrogen only

74-Sallmonella case. What the Moa of the treatment

DNA gyrase cipro,,,,,,,/0-

75-Limited phagocytosis in DM
76- known case of MS. Presented with relapse. What the mechanism:

Focal Demylanation

78-after dental procedure, he had loss of sensation over the mandible: Infra orbital

79- pain on parotid area increase while eating. What the nerve responsible for that:

facial

Auricotemporal

Triagmenal

80-Lower lip lymph nodes: Submental and sub /0-

81-What indicate Diabetic nepheropathy in urine analysis:

RBC case

Hyaline cast

Proteinuria

Hematuria

82-Oxybutin MOA: block muscarinic receptor

83- Osgood case

84: 2 cases of PaQelofemoral syndrome ،،،،،/0-

85: Perthes case

86: elderly with right hip pain and decrease abduc=on:

Radiography of right hip

87- Incarcerated Hernia: refer ro surgery

88-Gonerrea case. And another Q about test /0-

89- Leprosy case with ulnar thickness

90-meckles in lower ilium

91-painful vesicles in labia and cervix: HSV


92-couple presented to infertility clinic. The man is working in battery manufacturing.

Lead

93-post-mastectomy while having bath nurse noticed winging scapula ?

long thoracic nerve

94-something dureing surgery injured vagus n what organ will be effected ?

esophagus

Jejunum

Ascending colon

Cecum

95-enzymes where pancreas leaks ?

omentum bursa

96-which cells secrets melatonin ?

corticotroph

!"‫!ا‬$% '(()* '+,-.‫ ا‬/01


vaccine cI in pregnancy

HBV

MMR

Rubella

Haemophilus influenza

2-Patient take Thyroxin for hypothyroidism

High tsh

T3 normal ?

Subclinical hypothyroidism✔

3-Which group contr indicate in diabetic patient with heart faliure?

Biguanides
Dipeptidyl peptidase iv inhibitors

Glp-1 analgies

Sglt 2 inhibitors s

Thiazolidindiones✔

4- Low hb and murmur, how to treat this murmer?

Ttt of anemia✔
bi transfogen

5- Child with eczema in knee, flexure elbow, with maximum cortisone dose and eczema extending,
what to do?

Tacrolimus✔

6- Symptomatic AS, ttt?

Surgery✔

7- Prevent dvt in presurgical patient?

8- 56 y o with prolapse uterus, how to dx?‫ﺷﺮح‬

Severe hysterectomy

Not severe uterine suspension ,kegel ex. ,vaginal pessary ,estrogen

9- Cervical cancer screening every?

10- Pt with systolic murmer, Ecg show, non specific t change and St, dx?

As

Obstructive cardiomyopathy

11- First trimester with Vaginal bleeding . Fetal parts felt in the internal os . Open. Diagnosis?

12-First trimester vaginal bleeding. Internal os closed. Diagnosis?

threatened abortion ✔

13- Patient taken throxin for hypothyroidism.

TSH high

T3 normal

T4 normal.explain?
14- Tumor lysis electrolyte

A. Hypocalcemia ✔

B. hypercalcemia

C. Hyponatremia

15- Cause of meningi=s in 50 years old?


- at risk for Listeria (<1 month or >50 years old) or in immunocompromised
Listeria✔ - Post-neurosurgical à Pseudomonas aeruginosa

16- A=er 12 hours of gastric slap surgury, pt Came with upper abdominal pain, every thing normal?

ncrease analgesia ✔

Laporotomy

17- Cholecystitis with dilated duct and stone, inv? Ercp✔

ENDOSCOPIC RETROGRADE CHOLANGIO-PANCREATOGRAPHY

18- Massive pneumothorax after long air travel, ttt?

Decompression then chest tube✔

19- Short female, with htn, wide neck, dx?

Turner's Syndrome✔

20- Preterm baby deliver with dificulty breathing, dx?

21- Surgeon order you to prepare rt kidney but you sure is lt kidney?

1.go to intern supervisor

2.just leave it

3.prepare the rt kidney

4.talk with sourgeon✔

22- High cholesterol, high ldl, ttt?

Statins✔

23- Long term treatment of sickle cell anemia 1.penicilin


2.hydroxyurea✔

24- Phentolamine in ttt of pheochromocytoma act by?

Alpha adrenargic receptor blocker✔

25- Cystic fibrosis symptomatic, dx with chlorid sweet test, father and subling not symptomatic, what
investigation?

Chlorid to father

Chlorid to subling

Gen father✔

Gen subling

26- Type 2 dm, what medica=on decrese cvs disease?

Atrovastatin ✔

Fibrate

27- Elderly with hemo urea, acystoscop done and show upper most bladder mass, dx?

Transitional cell carcinoma✔

Sequamous cell carcinoma

28- Heart faliure, Best medication?

BB and hydralazine

Acei and diuretic ✔

BBand diureti

29- Lower limb flaccid and areflexia, after urti, dx?

Gbs✔

30- Feature of acute sever asthma?

Low o2 satura=on✔

31- Vircella child and his brother immunocompromize?

Vircella ig ✔

32- Patient with eye symptoms lower limb spastisity, what bes inv to reach dx?
MS (MRI brain) or Mysthenia Gravis (send for AChR anbtibody✔
‫ﺷﺮح‬

33-

٢٤ ‫م‬#34‫'ري ا‬678‫ا‬

';67&'- '‫ ? ا(>&=ھ‬4‫ه ا‬%"%#4‫=ره ا‬$% & '()‫ * ا‬+‫ا‬

:ershspring dis affect which layerChag's dis & h*

Serosa

Musclurais

Mucosa

Submucosa

*Girle with chronic constipation ‫ﻗـــﺎل ﺑﻌــــدﻳن‬

During Defecation muscle should relax ?

coccygeus

Pibococcygus

Illococcygus

Rectococcygus

*Flat topped lesion other detalis i don't remember with "pruritis" pic attached with multiple red lesions?

Lichen planus

Mollescum contigusum

Granuloma Anular

*Rash all over the body including palms and soles for 5 days no hx of drug taking "clearly men=on like
this in the qs no more detalis"?

Fixed drug eruption

Erythma nodusum

Erythma multiform

Pitryasis rosaea
*Lesion in jagular foramen which will be "preserved"?

Shoukder elevation

Sensation in the throat

Sensa=on Ant 2/3 of the toung

(Because jagular contain 9,10,11crainl nervs)

*Child run, 3 words sentence , open the door, climbs the furnature but not able to rise the starines with
alternate legs?

1y

2y

3y !!

4y

*Eye going inward which rectus muscle is affected "no further detalis" ?

Lateral

Medial

*Female DM1 has numbness and burning sensation in feet she wish if there's away to help her loss
whight?

Step arobic

Swimming

Jogging

Traidmil

*Femal she afraid from relevator she prefer to goo through staris eventhough its flair her OA however
she think that's better how to manage?
Psychoanalysis

Impitnip

Fluxatine

*42y Female no menstrua=on in the past 12 months what can be the cause ?

Hypoprolactin

Hypothyrodism

Congenital adrenal hyperplasia

*Pt with sudden Rt eye pain increase when he come to dark room eye has corneal edema & cilliary flush ,
fixed dilated pupil (they describe the lesion no pic) with manometry result shown below

Lt eye externally appear normal but with fundoscopy the upper temporal retina there's a notch

Manometry result :

Rt eye 69 normal 16-22

Lt eye 21 normal 16-22

What's the dignosis?

-Bilatral glucoma

-Rt eye uvitis & congenital change in the Lt

-Rt eye catract with sympathatic change in the Lt

-Rt eye conjunctivitis with something in the Lt i don't rember


Q:You are in OR with consultant and he ask you to prepare Lt. kidney for surgery but you are

sure it is the Rt. Kidney, what to do:

Inform chief intern

Run outside the OR and check the CT image

Inform the consultant about your concern

Q: Parents came to you and they don’t want to give their sun vaccines, how to deal with

them?

Q: case of appendicitis, ask about management

Q: child suffer from tonsillitis, you examine the patient and you find whites patches on it,

what is the management?

Q: man diagnosed with Hep C , the couple ask you what they do:

Abstinence

Use condom
Do intercourse with no restrict

Q: a husband diagnosed with HIV

Tell the husband

Tell the wife

Tell them both

Q: what antibiotic used as prophylaxis in TB?

Q: placenta previa and she has contractions, what to do?

Q: question about familial short stature

Q: managenet of papillary breast cancer

Q: management of ductal breast cancer

Q: 23 y old did cervical screen 3 years ago, ask about when to do it again, what to tell her?

Q: question about pap smear screening, every how much year to do it?

Q: there was 2 ques=ons about diabe=c ketoacidosis management

Q: how to diagnose sickle cell disease?

Q: management of cholelithiasis
Q: question about mammogram breast screen

Q: clinical scenario about overian tube torsion

Q: hyperprolac=nemia management ( 2 ques=ons, one of them, the wife did MRI and there

is tumor in pituitary gland, the other question ask about the drugs to treat the patient with)

Q: man with both lower limbs burn, ask about fluid replacement

Q: lady came to your clinic and wearing transparent cloths and speak with seductive tone,

and she speak language different than the nurse speak, what to do

Shout at her

Transfer her to another doctor

Call a nurse speak same language

Q: parathyroid tumor management?

Q: case about syphilis, ask about diagnosis

Q: ECG ,types of heart block

Q: questions about ectopic pregnancy, diagnosis and management

Q: clinical scenario about molar pregnancy, asking which type


Q: clinical scenarios about epigastric pain, pancreatic, gastric, esophagus

** you should know how to differentiate **

Q: question about diagnosis of trichomoniasis

Q: US of pregnant shows breech position, both legs and hip flexed, asking about breech

type
‫ﻃﻔﻞ‬١١‫ ﺳﻨ ﻪ ﻛﺎﻥ ﻋﻨﺪ ﻩ‬hemochromatosis

‫ ﻻﻧ ﻪ ﻋﻨﺪ ﻩ‬hemolytic anemia

‫ ﺍﺯﺍﻱ‬manag

A.venopuncture

B.penicellamine

C...

D......

Confusing
SMLE 18th of May 2019

1- Pt with painless ulcer + skin rash. Dx?


••Secondary syphilis!!!

2- Pt with painless ulcer + skin rash. Causative?


••Tryponema palidum!!!

3- If patient is HIV ve+, tell him only and tell him to till his wife butdoctor should tell the authorites. But If
pt say plz don't tell my wife, thenwhat you will do?
••You shouldn't tell her!!!
••Tell health authority's only!!!

4- Pt with high SVR but all other parameters (right side, PCWP, BP,
CVP). Dx?
••Hypovolemic shock!!!

5- Pt with ejection systolic murmur. Dx?


••Aortic stenosis!!!

6- Child with VSD less than 8 milli (i think it is 2 milli). Management?••Watchful waiting for spontaneous
closure!!!
••Surgical intervention!!!

7- What access for dialysis would you prefer?


••AV fistula!!!

8- Best site for AV fistula in dialysis?


••Brachiocephalic!!!

9- Pt with both kidney and lung affection. Dx?


••Burger disease!!!

10- Case of hemolysis + impaired kidney function + thrombocytopenia.Dx?


••HUS!!!

11- Case of hemolysis + impaired kidney function + thrombocytopenia +fever and confusion. Dx?
••TTP!!!

12- Pt with chronic HCV came with UGIB. You did resuscitation then what to give medication?

••Octreotide!!!

••Vasopressin!!!
13- Pt with weight loss and epigastric pain (features suggests gastrincancer i think). What
investigation you will order?
••Endoscopy!!!

14- Case about UC.

15- Case about CD.

16- Case about intussucception.

17- Case about meningitis. Prevention?


••Rifampicin!!!
••Ceftriaxone!!!

18- Vaccine contraindicated in egg allergy patients?


••Yellow fever!!!

19- Vaccines contraindicated in pregnancy?


••Varicella!!!

20- Vaccines given in pregnancy?


••Influenza!!!

21- Child can't collect 3 words in a sentence, can go upstairs. Age?••!!!

22- Case about anal fissure. Dx?

••Anal fissure!!!

23- Anal fissure ttt?


••Internal lateral sphinctrotomy!!!

24- Swelling below inguinal ligament lateral to pubic tubercle. Dx?••Femoral hernia!!!
••Inguinal hernia!!!

25- Pt underwent appendectomy then wound gets secretion and puscomes out of it. What to do?
••!!!

26- Nocturnal enuresis occurs at which age?


27- Smoking increase risk of ischemia by?
••Two times!!!
••Three times!!!
••Four times!!!
••Five times!!!

28- Pt has diarrhea and he is smoker. His brother has crohn disease. Heis concerned about having
crohn disease also. What things that mightdecrease occurrence of crohn disease in this patient?
••Smoking!!!
••Diet!!!

29- Case about Rheumatoid arthritis management.

30- Case about SLE flare management.

31- Pt can't look to medial above and below. What cranial nerveaffected?
••3rd!!!

32- Pt can look normally to right side. When asked to look to left side,right eye looks normally while left
eye stops at middle. Dx?
••Left 6th CN palsy!!!

33- Picture of child with open fracture. C-ray postponed I think. Whatto do?
••ORIF!!!

34- Child with features of Nisseria meningitis (similar to HSP). Askingabout what to do with his child
brother?
••Ciprofloxacine!!!
••Ceftriaxone!!!
••Rifampicin!!!
••Vaccination!!!

35- Case about abrupto placenta.

36- Case about type of abortion.

37- Case of Endometrial hyperplasia

38- Case of PMB and US shows endometrial thickness of 14 mm ‫مكرر‬. I


37- Case of Endometrial hyperplasia ‫مكرر‬.
38- Case of PMB and US shows endometrial thickness of 14 mm ‫مكرر‬. I
think asking what is the management?
••Hysterectomy!!!

39- Case of hyperprolactiemia.

40- Case of pituitary tumor, hemorrhage occurred there lead to visualfield affection. Management?
••Trans-sphenoidal interfere to relieve pressure on optic chiasm!!!

41- Case about measles. Dx?


••Measles!!!

42- Three cases about pyloric stenosis!!!

43- Pt with features of ischemia for long time, started now to haveischemia at rest and episodes
increased. Dx?
••Unstable angina!!!

44- What is the thing that increased the level of BNP falsely?••Obesity!!!
••!!!

45- Bacteria that causes pseudomemberaneous colitis is?••Clostridium dificile!!!

46- What (increase or decrease I don't remember) the activity ofClostridium dificile?
••Vitamins!!!
••Ca carbonates or bicarbonate!!!
••Ferrous sulphate!!!

47- ECG of pericarditis.

48- Adenosine is not given with what medication?


••Dipyredamone!!!
••Theophylline!!!

49- Clinical and ECG case of Inferior MI. Pt hypotensive. Next step?••Right chest leads!!!
••ECHO!!!

50- Case of Croup. diagnosis?


••Viral Croup!!!
51- Case of Croup. Organism?
••Parainfluenza!!!

52- Case of child with BA. Mother worried about the disease in his childin the future. What to tell her?
••Bronchial asthma will continue in future!!!
••BA will improve in future!!!
••BA in children is not related to BA in adult!!!

53- What parameter will tell you BA severity?


••FEV1/FVC ratio!!!

54- COPD case, best to tell you about prognosis and oxygen need inthe patient ‫مكرر من مونا‬.?
••PaO2 measurement once or twice and less than how much!!!

55- TTT of trichomonas vaginalis (greenish vaginal discharge,flagellated organism).

56- Case of bacterial vaginosis.

57- Pt with multiple repeated contractions and sluggished fetal heartrate response on CTG. Mother
was given epidural analgesia, givenoxytocin to induce labor, given MgSO4 due preeclampsia. What is
thecause of this CTG?
••Oxytocin!!!
••Analgesia!!!
••MgSO4!!!

58- Many repeated questions about trauma, always remember if ptvitally unstable (low BP, high HR) so
next is resuscitation and fluid.

59- Case about pregnant in 13 weeks with vaginal bleeding. US picturegiven of a uterus and a sac
contains something. Closed internal os.What is the type of abortion?
••Unembryonic sac!!!
••Missed!!!
••Threatened!!!
60- Case about cognitive impairment. Dx?
••Alzheimer!!!
••Vascular dementia!!!

61- Case of eczema, thrombocytopenia and recurrent infections. Dx?••Weschot Aldrich


syndrome!!!

62- Case of X linked agamma globulinemia.

63- Picture if Impetigo.

64- Picture of genital plaques chondylomata.

65- Picture of CXR.

66- Picture of ECG.

67- Picture of CTG.

68- Case of resistant eczema (atopic dermatitis, not responded tosteroid). TTT?

••Calcineurin inhibitors (Tacrolimus)!!!

69- Most common infection associated with IUCD is?


••Actinomyces!!!
1-Pt old man come to hospital c/o abd distension and vomiting and nausea long Sinario , O/E
there is abdominal mass ?

1- Intussption

2- Mid valvular

2-Pt c/o lower mandibular pain with loss of sensation ?

1- Great aricular

2- Ophthalmic

3-Pt main hx of asthma she is now on SABA , need to go to 2nd step but she refuse that , what
you do ?

1- Increase dose SABA

2- TAKE LABA

3- TELL pt how to take steroid and benefit

4-Pt c/o with thyroid problem

5-Mother came to hospital with baby 15 months she says he talk only 3 words , o/e good
development ?

1- Reassurance the normal hear

2- Hearing test and refeed to ENT

6-Pt c/o something discharge from him ear purulent ?

7-2 couple came to obs clinic need to check the infertility ( not write any duration of that ) ?

1- Start abdominal ex

2- Vaginal inspection

3- General apparent

4- PV

8-Pt 14 year p 4 G 3 GA 32 she isn’t came to ANC visit never what the cause ?

1- Visit is expensive

2- 2nd aggiornt

3- Scaring from the visit


9-Pt coming from sudan to KSA need to prophylaxis ?

1- Choloquine

2- Fansidar

3- Quinie

10-The first line of P.Falcipram ?

1-quinie

2-fansidar

11-Pt pediatric need to exercise?

1-120 min

2-60

3-30

4-15

12-Pt c/o lower abdominal pain in the left side colicky in nature , radiate to back , in CT found
stones ( no write the diameter ) in gall bladder , with normal Gall bladder ?

1- Cholestectomy

2- Ursodeoxycolic acid

13-Long sinario Pt c/o with long time of vomiting ,development of blood , no chest pain ?

1-esophageal rupture

2-mallary wise

3-peptic ulcer

14-Pt diagnosis with lekumia on chemotherapy , what about electrolyte ?

1-hypoca

2-hyperca

3-hypona

4-hyperna
15-Pt old in AL HAJ ,c/o body rash , with neck pain and fever , wbc 12 hg 11 lymphocyte low ,
how can transport this disease ?

1- Air bourn

2- Droplet

16-Pregnant woman c/o with hepatis ?

1-primary ttt

2-secondary

17- Pt c/o of fever and cough lymph node enlargement ?

1-RSV

2-influenza

18-Pt asthmatic for long time , IUGR , and do induce of labor , what the drug contraindication
with her ?

1-sintocynone

2-methocholine

19-Woman with main cause of nephrotic syndrome ?

1-minmal change disease

20-Mother came to clinic with her infant need to council about colicky pain :

1- reassure the mother 80 % of infant with same complain

2-colicky pain with flatus in abdomen

3-reassure pain relief with the 6 weeks

21-Smoker risk to pregnant

1-low birth wt for fetus

2-respiratiry problem of the baby

3-preterm baby

22-Long senariowith symptoms of Cushing syndrome, in end of the senario say they do renin 24
ratio it's double result, what the diagnostic test?

1-ct of aderinal

2-brain MRI
23-Female during examination found swelling in abdomen :

1-letual cyst

2-ectopic pregnancy

3-corpus leutal cyst

24-Most common cause of nephrotic syndromewhat the cause

1-minimal change nephropathy

2-focal segmental glomerular sclerosis

3-membranous glomerular nepheritits

25-Cause of oedma in renal failure?

26-Woman came to ER with severe abdomen pain she ergancy ovarian torsion ned to enter the
surgery, but she refuse operation, what you do?

1-respect her wish

2-ignor and enter the operation

27-Mother came to ER with her baby, his ingest something but she isn't know what it's that, and
complain black vomiting and diarrhea, what he ingest?

1-Multivitamine

2-iron supplement

28-Mother came to ER with her baby to general check up and found in cbc all in border line
( except hg is low 9), what the diagnosis?

1-thalassemia traid

2-idiopathic anemia

3-SCA

29-In the last stage of labor Pregnant woman obese and dilated 3 cm and full efacement all
investigation are normal

1-delivary by forceps

2-by ventous

3-by C/S
30-Bishop score (2 question)

31-Necrtoizing fasculitis 3 questions

32-Post orthopedic problem develop DIC on warrfrindue to A fib?

1-change to warrfrin and unfractionated heparin

2-enxoparin

33-Defentive treatment of lymphngitis?

1-antibiotic

2-infilterated of lymph chanal

34-Different between exudate and transudate in Pleural effusion

35-Excersice per week in child

1-120 min

2-60

3-30

4-15

36-Pregnant complain symptoms of hepatitis take vaccine what the type of prevention?

1-primary

2-secondary

3-tertiary

37-Picture of labia majora?

1-warts

38-Surgeon confused with which kidney will by affected to replacement

39-Allergy drug

40-Pt DM with HTN and limb ischaemia, decrease in pulse? ( cloudication)

1-angio

2-MRIA

3-duplex U/S
41-Best investigation of limb ischaemia

42-Thrmbolysis risk to

1-stroke

2-embolism

3-hemorrhage

43-Pt retro partum found mass 10 *9 cm

1-liboma

2-sarcoma

3-gastric overin

Kaswaki (2 question) :

44-What the most related to Kaswaki?

1-rash

2-lymph node

45-Kaswaki follow by

1-cbc

2-echo

3-angio

4-PCR

46-Treatment of Kaswaki

1-ASA

2-IV IG

47-COPD pt comatose what to do?

1-oxygen

2-reduce oxygen

3-CPAP

4-Mechaincal ventilation
48-Pt distress and more secretion of nose and mouth pH is acidic SO2 is low?

1-oxygen

2-reduce oxygen

3-CPAP

4-Mechaincal ventilation

49-Age of screening of colon CA

1-50

2-40

3-60

4-70

50-Case of Sarcodosis in x ray there is filtration and lymph node?

51-Erythema nodasum located in fabila and tibia

52-Granuloma

53-Congestive heart failure increase survival by

1-ACEI

2-

54-Long senario the conclusion is pregnant ( ectopic, life in rural area) her husband travel in
another country, she is complain severe pain and her neighbor going with him to hospital all the
vital sign is normal, the mass 2 cm B Hcg lower than 5000?

1-methotrexate

2-surgery because the distance

3-vital sign

55-Pregnant came to ER in her 18 weeks,in U/S corresponding to 20 with bleeding, cervix is


open and she discovered the product ‫ﺍﻝ‬GA ‫ﺃﻗﻞ ﻣﻦ ﻋﻤﺮ ﺍﻟﻤﻮﺟﺎﺕ ﺍﻟﺼﻮﺗﻴﺔ ﻭﻓﻲ ﺑﺮﻭﺩﻛﺖ ﻃﺎﻟﻌﺔ ﻭ ﺍﻟﺴﻴﺮﻓﻜﺲ‬
‫ﺍﻭ ﺑ ﻦ‬
1-therenated

2-missed

3-incomplete
56-Pt her age 55 when do breast screen ( mammogram)

1-after one year

2-2 yrs

3-3 yrs

57-Peadiatric Pt came with heamtouria after skin lesion

1-HSP

2-ITP

3-POSTstreptococcal

58-Pt Peadiatric go to dentist and complain with gum bleeding, dentist ask his mother any hx of
bleeding, she say is petechiae and bruise

1-hemophilia A

2-hemophilia b

3-ITP

59-thyroid nodule, what the intimation investigation

1-thyroid scan

2-FNA

60-Pt with enlargement of Thyroid, large after in thyroid scan, what the second option

61-pic of HSP ( ask the diagnosis)

62-pt came to ER with rash and blood in urine found all sign of HSP but not found it in
investigation

( hypersensitive vasculitis)

63-pt of SLE came with severe joint pain?

1-methotrexate

2-cyclophsomide

64-pt complain of erection, Thyroid hormone normal

1-hyperpitutary

2-hypopituitary

3-macrprolactinoma

4-microprolactinoma
65-pt Peadiatric, conclusion of deficit

1-500

66-pt Peadiatric, conclusion of maintance

1-1000

67-mother came with her chlid she is complain,there is brusies in labia majora

1-abuse

2-Foreign body

3-truama

68- girl came to ER with child abuse, what the prove of abuse?

1- hymine at 6 clock

69-GERD

1-endoscopy

2-ph dependened moniter

70-( long senario) esophagus with metaplasia with high grade what you give?

1-pendazole and randitadine

2-PPI

3-Sclerotherapy, resection

71-pt with heamtemsis after long vomiting

1-gastritis

2-mallery wiess

3-peptic ulcer

4-oesophageal variceses

72-bleeding per rectum, anal swelling and pain

1-thrombos pill

2-anal fissure

3-hemorrhoid
73-pt paraanal sweeling and pain

1-absces

2-fissure

3-hemorrhoids

74-eldery pt he do operation for colon, after 5 days complain swelling in rectum

1-laproscopy drainge

2-explore operation

75-neonate complain diaphragmatic hernia not cry , what do to support him

1-chest tube

76-placenta previa in ER what to do?

77-DM pt came to ER with DIC on heparin, past hx of cloudication, atrial fib

After DIC what to stop?

1-heprain

2-insulin

3-furosmide

78-pic of ECG ( heart block) what to give?

1-pace maker

2-BB

3-Warrfrain

79-pic of CTG ( late) what the diagnosis?

1-cord

2-placenta

80-eldery pt long senario of heart diseases and BPH , and new diagnosis colon cancer and mets

1-palliative care

2-need of high care

81- colon cancer what to give

1-vit k 2- Vit c 3-fibrate


82- Hajjaj with long senario, severe fever and neck pain and rash , blood film wbc 13 high
lympocyte, what transmission of this diseases?

1-airborne

2-droplet

3-contact

83-Hajjaj one of them, came with fever and blood of sputum, lymph node enlargement, how to
protect another?

1- by safety clothes

2-by face mask

3- control of air drop

4-control of contact
1.pte with copd and co2 9 what u wll do

mechanical ventilation

reduce o2

antibiotixs

2.55 years old wit AF what u wll do to reduce the complications

warfarin

pt with atrial fibrilation on warfarin develop inracranial heamorrage what u wll do

vitamin k

FFP

HEPARIN

PT develop orthopnia PND chest auscultation basal crips she deliverd one week ago what is ur
diagnosis

perpartum heart failur

IHD

ashma

ashmatic pte have no exacerepation in last mounth his dialy PEF is 600 what is indicator of
sevirity in this pt

RR 25

PFR 240

ashmatic child his family stat that thy can not go out becouse the nebuluzer machin need
electicity what u wll do to help them

tell them they are right

provid them with nebulizer with battery

clearly instruct the inhaler and tell them to stop the nebulizer

old pte admited becouse of pneumonea at night she develop restlessness she is disorinted
what u wll do

physicaly but her in her bed


call her family to set besid her

lorazepam 4 mg stat

pte with cholecystectomy develop pain in the angle of the mouth what u wll do

u did appendictomy and you found it not inflamed you remove it and closed what the next

tell the pte

tell the ethics

tell the surgery head

you r doing appendictomy u found bus and fluid in the RIFwhat u wll do

remove the fluid and appebix

remove the apendix

pt presented with some coplaint potasium wass 7.9 what u wll do

dialysis

insulin

Ca gluconat

pte on dialysis develop shivering he mention history of fever 3 day ag

you sow pus drainag from the catheter

what u do

.take blood for culur and give AB

blood culure ABand stop dialysis 3 days

change the casheter

pet wit URT 3 days develop shortness of breath PND LLedema troponin is high what is apropriat
step to rech the doagnosis

echo

ecf cxr

mri

pte with AAA 4.5 CM have cholecystectomy vascular surgery councled what he wll say
proceed the surgery and folw up with US

PROCEED THE SURGRY FOLOW WITH CT

CT THEN DO THE SURGERY

PTE WITH recurent abdominal pain us show dilated intrahepatic duct tembretue 38 pain increas
with morphin LFT Normal ALp high what is diagnosis

cholelethiasis

choleangitis

cholecystitis

pte have epogastric pain after vomiting cx show some pleural effusion what is diagnosis

rubture eosaphtes

peadiatric pte with testicar swlling translumination negative what u wll do

mish repair

herniotomy

hernigraphy

peadiatric pte with bilatral testecular swlling increas when he is crrying what to do

herniotomy

mish repair

laproscopic mesh repair

ptw with abdo mass after lefting heavy object mass not change with cugh

what 9s diagnosis

rectus muscl heamatoma

hernia

pte with testicular pain and vomiting scrotum is red what u wll do

US

urgent surgry

pregnant in her 37 week faint and complain of sever abd pain no pleedin pb 80/50 HR120 WHat
All go with PE exept abdominal pain
os diagnosis
All go with abrupto exept no bleeding
pulmonary empolism

abruptio placente

couple seeking fwrrility clinic after 3 mounth what is ur advise

try more

pte with breast cancer and fibroid on tamoxfin develop vaginal bleeding USshow fibroid 8cm
and hypowchogenic area what is diagnosis

endometrial cancer

lyomyosarcoma

overian cancer

65 years old femal presented with v bleeding and abd pain Ca125 veey high what u wll do

biobsy. Do C then A (We should admit, stabilize, then do


antibiotic
biopsy)

admit

pregnant lady with recuren pain known case of gallstone what wll do

surgery in the second trimester

surgery after delivery

pregnant lady complain foul smelling vaginal diacharg what u wll give

amoxill

cephalexin

metronidazole

hydit cyst reatment

ameobic liver abcess treatment

bacterial vaginosis treatment

pte withe placental abrubtion BP 8/40

what u wll do
us

admission foe sciu under

maximum response team and multidisiplinary

pte with ectopic pregnancy what is pest quistion befor managment of the pte

accessability to hospital

previous surgery

comlet mole direct quistion

[ Patien has cafe alue spot in his back what you will.do next

1. Council.the patient about neurofibrmatosis

2.send the patient to genteics to do genetic

[Patien has subcutanuoes nodule and arthrlagia what one of follwoong confirm diagnosis of
Rhematic fever

1. High crp

2.high esr

3.postive blood culture

[Patien has st elevation in v1 v 2 v 3 what is your diagnosis

1.mi

2.pericardities

3.cardiac tamponade

Patient age of 4 years convulsing at home for 3 minute he has tonsilities temp is 38 what you
will do next

1. Diazepam

2.lorazepam

3.amoxicillin

4.paracetamol
Patien fall in out stretched hand ...pic of open colles fracture what you will.do next

1. Open reduction

2.close reduction

3.depridment with open reduction

[ Pregnant complain.of painful vaginal bleeding is it

1. Placenta abrtpio

2.placenta previa

[Pregnant at 20 weak complain of vaginal bleeding os closed ultra sound no fetus.....complete


abortion

[Pregnant with bleeding os open ...invitable

[ Bleeding with os closed threatned ...bed rest

[Magnisium sulfate givun to htn pregnant to.

...prevent the attack

Copd excarebation and co2 retention and consious.....NIMV

Un consious copd excacerbation ......mechanical ventilation

[ Perecardities first drug to give....non steroidall antiinflamatory drug like ibuprofen

Celiac disese should avoid ...gluten conatin diet like u should knw the type of food contain
gluten

Meickle divertixlum.....end of ileum

[Pt with right ilac fossa pain with bcg negetive.....acute appendicites

.....

[ Fishy smell....bacterial vaginsois

Gray white discharge....trichomanis

Secer headache ever...subarachnoid hemorrhage

Hypernatremia with normal potasium .....siadah

Olive.....pyloric stenosis Target sign....intusseption


1-a myocrial biopsy was taken from woman because of unexplained congestive HF: biopsy
showed Antischkow cells:

-rheumatic fever

-SLE

-Sarcoidosis

2-Patients with sx of BV:

What’s the test: gram stain

3-Case of Trichmonisis what’s the dx

4-and what’s the treatment(metro)

5-You are seeing a 78 yrs old lady in clinic. She looks sick had signification weight loss and feels
tired all the time. She has pelvic mass. What the the most appropriate screening test to confirm
the diagnosis:

US

Colposcopy

Ca-125

6-Patient with DLP on statin, lab results for lipid profile all within normal range except high
alanine aminotransferee (48) ‘normal up to 40’

What you’re going to give this patient:

Fibrate

Cholestyramine

Ezetimibe

Omega 3
7-Pregnant lady in her first trimester, she was exposed to rubella 3 days ago, what you’re going
to do:

Nothing

MMR

Terminate the pregnancy

Rubella Ig

8-Patient with MG crisis: plasmapheresis

Patient with fatiguable weakness( sx of MG), whats the pathophysiology of this condition:

Antibodies against Ach receptor

9-Patient with unilateral knee pain and urethral discharge of pus. Negative culture of the pus:

Riter syndrome

10-Patient with back and buttocks pain that improves with exercise:

Riter syndrome

Reactive arthritis

Ankylosing spondylitis

11-Patient with sx of gastroenteritis:

Rota,

Norovirus

Adenovirus

12-2 Qs about staph aureus food poisoning


13-Clostridium perfengis after surgery

14-End distention of protien (ER, cytosome, golgi, lysosome)

15-Hydrogen bond btw mRNA and rRNA

16-worse prognostic gene with HCM

Myocin binding protein C

Beta myocin

17-Parklond formula, how much fluid are given in first 8hrs for anterior trunk and
circumferential left upper and left lower limbs: 6.3

18-patient with hight parathyoid hormone and hypercalcemia, what u will do next:

Look for parathyroid adenoma

TSH, T3 and T4

19-Patient with breat Ca, polyuria and increase thirst, she has also Hypercalcemia: what will u
give her

Normal saline

Furosemide

Amlodapine

20-Case presentation of Addison, what u will find:

Hirsutism

Hyper pigmentation

Buffalo hump
21-Case presentation of hyper aldosteronism, what u will find:

Polyuria

23-42 yrs old male came for general check up, BP and Fast blood glucose are within normal
ranges his BMI is 28 and he is asking about ur advice:

Wt reduction and exercise

Exercise

Low Salt diet

24-Patient coming for general check-up his BP is normal and has family Hx of cardiac diseases,
what u will do for him

Lipid profile

25- patient coming to see the results of INR, what the drug he is taking:

Warfrain

26- best prophylaxis for thrombosis post surgery is

LMWH

27-75 yrs old male Patient with DM and HTN and hx of 2 stroke events in the past. Coming with
Afib. What u will do for him:

Warfarin to prevent any stroke event

28-Patient with bph and HTN, what u will do for him:

Alpha bloker

TURP

Open prostatectomy
29- patient on methtroxate, what would be deficient in him:

Folate

Niacin

Vit E

30-Case of rheumatic fever (A 7yrs old boy presented with knee pain. On examination, he has
heart murmur. His parents told you that he had tonsillitis 3 wks ago and treated with
paracetamol. His ASO titer is high. How u will treat:

-Penicillin

-steroid

-aspirin

31- patient presented with SOB on exertion and pallor. His CBC showed low Hb, normal MCV
and high LDH

Hemolytic anemia

Microcytic

32- patient with jaundice after quinine treatment. What the deficient enzyme:

Glucose-6 phosphate dehydrogenase

Glucose dehydrogenase

Glucose 6 kinase

33- patient his bone marrow bx shwoed 80% blast cellls and Auro rods:

AML

ALL

CML
CLL

34-most oxygenated zone of the liver is

35- patient with presentation of generalized anxiety disorder, what is the best drug for her:

Lorazepam

Bupropion

Buspirone

Citalopram

36- patient with presentation of grvis disease, what’s the pathophysiology:

Antibodies binding to TS receptor

37- case of otitis media and asking about the diagnosis Otoscopy

38- patient with ase of otitis media and erytham on mastoid bone, what’s the treatment:

Amoxicillin

Cefoxaim

39-Case 7 yrs has pupic hair no breast devolpment :

premature adrenarche

40-First trimester miscarriage at 5 week with history of 2nd trimester abortion twice duo to
cervical incompetence , cause now :

Cervical incompetence

Chromosomal abnormality .
41-On antidepressant succide bottle beside her has dilated pupil: amitriptyllain

Baro recpter activation will

decrease venous capictance

43-19 yrs old lady with Diarrhea for 10 months, wt loss, postprandial periumbilical pain. Diarrhea
sometimes bloody

Crohn’s

Peptic ulcer

Chronic Pancreatic

44-Fibroid african race

45-Increase total cholesterol + high LDL + high triglyceride + low HDL ... ask about the follow up
after 1 year

Blood pressure

Lipid profile

46-Monitor of asthma in ER

FEV1

Heart rate

partial pressure of oxygen

47- After stopping the OCP, pt still have amenorrhea for 6 months, gain 5 kg, feel tired. labs
show elevated glucose

Hypothyroidism

Cushing syndrome

49-case female pregnant Still born at 38 wks every thing normal then discharge but after 3 days
come back with bleeding from every site injection Which best investigation ?

Hemoglobin electrophoresis

Fibrin product
50-Patient with presentation of HUS. What u will do:

Hemodylasis

Abx

Platelets transfusion

51-10 student with pneumonia (legionella, mycoplasma, strep )

52-Painful axillary lump with tender and erythematous with black head papule and large pores?

Oral Abx

53-Anterior axillary for breast cancer

54-RSV in bronchitis

55-HIV latent: CD8, b memory cell, macrophage

56-Stillbirth 20 wks

57-stillbirth Alpha thalassemia: 2 normal b and abnormal 4alpha

58-Bisphosphonate in osteoporosis

59-4 days boy passed meconium in first hour, poor feeding since birth. Today has vomiting and
Abd distention: Midgut valvolus, hirshpurng

60- pt in ER starring at man and says u can’t kill me: Hallucinating

Patient with erectile dysfunction, has depression. What drug should be avoided? Sertraline

61-Patient with MI. What’s the mechanism of pain reliever:

Stimulate CNS Mu receptor

62-Glaucoma treatment 2Qs

63- Achalasia case

64-cricopharngeal Dysfunction
65- pt with Croup. What u will hear in exam:

Inspiratory sound due to subglotic narrowing

Inspiratory sound due to secretion

66-TOF components

67-middle aged with Cyanotic disease. His x-ray showed enlarged pulmonary artery and
increased vascular markings ;

Pulmonary stenosis

VSD

Truncus arterious

68-Clear case of Nephrotic syndrome

69-steroid sensitive age in minimal change disease:

6-12

12-48

12-96

70-Caspofungin

71-SSRI in pMS

72-Most common affect (behavioral) symptom in premenstrual dysphoric disorder?

Irritability from fudol

Mood swings

Depression

Anxiety

73-Patient who had hysterectomy and hot flush after the surgery

Transdermal Estrogen only


74-Sallmonella case. What the Moa of the treatment

DNA gyrase

75-Limited phagocytosis in DM

76- known case of MS. Presented with relapse. What the mechanism:

Focal Demylanation

78-after dental procedure, he had loss of sensation over the mandible: Infra orbital

79- pain on parotid area increase while eating. What the nerve responsible for that: facial

Auricotemporal

Triagmenal

80-Lower lip lymph nodes: Submental and sub

81-What indicate Diabetic nepheropathy in urine analysis:

RBC case

Hyaline cast from fudol

Proteinuria

Hematuria

82-Oxybutin MOA: block muscarinic receptor

83- Osgood case

84: 2 cases of Pattelofemoral syndrome

85: Perthes case

86: elderly with right hip pain and decrease abduction:

Radiography of right hip

87- Incarcerated Hernia: refer ro surgery

88-Gonerrea case. And another Q about test

89- Leprosy case with ulnar thickness


90-meckles in lower ilium

91-painful vesicles in labia and cervix: HSV

92-couple presented to infertility clinic. The man is working in battery manufacturing.

Lead

93-post-mastectomy while having bath nurse noticed winging scapula ?

long thoracic nerve

94-something dureing surgery injured vagus n what organ will be effected ?

esophagus

Jejunum

Ascending colon

Cecum

95-enzymes where pancreas leaks ?

omentum bursa

96-which cells secrets melatonin ?

corticotroph

1-a myocrial biopsy was taken from woman because of unexplained congestive HF: biopsy
showed Antischkow cells:

-rheumatic fever

-SLE

-Sarcoidosis

2-Patients with sx of BV:

What’s the test: gram stain

3-Case of Trichmonisis what’s the dx

4-and what’s the treatment(metro)


5-You are seeing a 78 yrs old lady in clinic. She looks sick had signification weight loss and feels
tired all the time. She has pelvic mass. What the the most appropriate screening test to confirm
the diagnosis:

US

Colposcopy

Ca-125

6-Patient with DLP on statin, lab results for lipid profile all within normal range except high
alanine aminotransferee (48) ‘normal up to 40’

What you’re going to give this patient:

Fibrate

Cholestyramine no confirmed answer yet

Ezetimibe

Omega 3

7-Pregnant lady in her first trimester, she was exposed to rubella 3 days ago, what you’re going
to do:

Nothing

MMR

Terminate the pregnancy

Rubella Ig

8-Patient with MG crisis: plasmapheresis

Patient with fatiguable weakness( sx of MG), whats the pathophysiology of this condition:

Antibodies against Ach receptor


9-Patient with unilateral knee pain and urethral discharge of pus. Negative culture of the pus:

Riter syndrome

10-Patient with back and buttocks pain that improves with exercise:

Riter syndrome

Reactive arthritis

Ankylosing spondylitis

11-Patient with sx of gastroenteritis:

Rota,

Norovirus

Adenovirus

12-2 Qs about staph aureus food poisoning

13-Clostridium perfengis after surgery

14-End distention of protien (ER, cytosome, golgi, lysosome)

15-Hydrogen bond btw mRNA and rRNA

16-worse prognostic gene with HCM

Myocin binding protein C

Beta myocin

17-Parklond formula, how much fluid are given in first 8hrs for anterior trunk and
circumferential left upper and left lower limbs: 6.3
18-patient with hight parathyoid hormone and hypercalcemia, what u will do next:

Look for parathyroid adenoma

TSH, T3 and T4

19-Patient with breat Ca, polyuria and increase thirst, she has also Hypercalcemia: what will u
give her

Normal saline

Furosemide

Amlodapine

20-Case presentation of Addison, what u will find:

Hirsutism

Hyper pigmentation

Buffalo hump

21-Case presentation of hyper aldosteronism, what u will find:

Polyuria

23-42 yrs old male came for general check up, BP and Fast blood glucose are within normal
ranges his BMI is 28 and he is asking about ur advice:

Wt reduction and exercise

Exercise

Low Salt diet

24-Patient coming for general check-up his BP is normal and has family Hx of cardiac diseases,
what u will do for him

Lipid profile
25- patient coming to see the results of INR, what the drug he is taking:

Warfrain

26- best prophylaxis for thrombosis post surgery is

LMWH

27-75 yrs old male Patient with DM and HTN and hx of 2 stroke events in the past. Coming with
Afib. What u will do for him:

Warfarin to prevent any stroke event

28-Patient with bph and HTN, what u will do for him:

Alpha bloker

TURP

Open prostatectomy

29- patient on methtroxate, what would be deficient in him:

Folate

Niacin

Vit E

30-Case of rheumatic fever (A 7yrs old boy presented with knee pain. On examination, he has
heart murmur. His parents told you that he had tonsillitis 3 wks ago and treated with
paracetamol. His ASO titer is high. How u will treat:

-Penicillin

-steroid

-aspirin
31- patient presented with SOB on exertion and pallor. His CBC showed low Hb, normal MCV
and high LDH

Hemolytic anemia

Microcytic

32- patient with jaundice after quinine treatment. What the deficient enzyme:

Glucose-6 phosphate dehydrogenase

Glucose dehydrogenase

Glucose 6 kinase

33- patient his bone marrow bx shwoed 80% blast cellls and Auro rods:

AML

ALL

CML

CLL

34-most oxygenated zone of the liver is

35- patient with presentation of generalized anxiety disorder, what is the best drug for her:

Lorazepam

Bupropion
Buspirone

Citalopram

36- patient with presentation of grvis disease, what’s the pathophysiology:

Antibodies binding to TS receptor

37- case of otitis media and asking about the diagnosis

38- patient with ase of otitis media and erytham on mastoid bone, what’s the treatment:

Amoxicillin

Cefoxaim

39-Case 7 yrs has pupic hair no breast devolpment :

premature adrenarche

40-First trimester miscarriage at 5 week with history of 2nd trimester abortion twice duo to
cervical incompetence , cause now :

Cervical incompetence

Chromosomal abnormality .

41-On antidepressant succide bottle beside her has dilated pupil: amitriptyllain

Baro recpter activation will

decrease venous capictance

43-19 yrs old lady with Diarrhea for 10 months, wt loss, postprandial periumbilical pain. Diarrhea
sometimes bloody

Crohn’s

Peptic ulcer

Chronic Pancreatic

44-Fibroid african race


45-Increase total cholesterol + high LDL + high triglyceride + low HDL ... ask about the follow up
after 1 year

Blood pressure

Lipid profile

46-Monitor of asthma in ER

FEV1

Heart rate

partial pressure of oxygen

47- After stopping the OCP, pt still have amenorrhea for 6 months, gain 5 kg, feel tired. labs
show elevated glucose

Hypothyroidism

Cushing syndrome

49-case female pregnant Still born at 38 wks every thing normal then discharge but after 3 days
come back with bleeding from every site injection Which best investigation ?

Hemoglobin electrophoresis

Fibrin product

50-Patient with presentation of HUS. What u will do:

Hemodylasis

Abx

Platelets transfusion

51-10 student with pneumonia (legionella, mycoplasma, strep )

52-Painful axillary lump with tender and erythematous with black head papule and large pores?

Oral Abx
53-Anterior axillary for breast cancer

54-RSV in bronchitis

55-HIV latent: CD8, b memory cell, macrophage

56-Stillbirth 20 wks

57-stillbirth Alpha thalassemia: 2 normal b and abnormal 4alpha

58-Bisphosphonate in osteoporosis

59-4 days boy passed meconium in first hour, poor feeding since birth. Today has vomiting and
Abd distention: Midgut valvolus, hirshpurng

60- pt in ER starring at man and says u can’t kill me: Hallucinating

Patient with erectile dysfunction, has depression. What drug should be avoided? Sertraline

61-Patient with MI. What’s the mechanism of pain reliever:

Stimulate CNS Mu receptor

62-Glaucoma treatment 2Qs

63- Achalasia case

64-cricopharngeal Dysfunction

65- pt with Croup. What u will hear in exam:

Inspiratory sound due to subglotic narrowing

Inspiratory sound due to secretion

66-TOF components

67-middle aged with Cyanotic disease. His x-ray showed enlarged pulmonary artery and
increased vascular markings ;

Pulmonary stenosis

VSD

Truncus arterious

68-Clear case of Nephrotic syndrome


69-steroid sensitive age in minimal change disease:

6-12

12-48

12-96

70-Caspofungin

71-SSRI in pMS

72-Most common affect (behavioral) symptom in premenstrual dysphoric disorder?

Irritability

Mood swings

Depression

Anxiety

73-Patient who had hysterectomy and hot flush after the surgery

Transdermal Estrogen only

74-Sallmonella case. What the Moa of the treatment

DNA gyrase

75-Limited phagocytosis in DM

76- known case of MS. Presented with relapse. What the mechanism:

Focal Demylanation

78-after dental procedure, he had loss of sensation over the mandible: Infra orbital

79- pain on parotid area increase while eating. What the nerve responsible for that: facial

Auricotemporal

Triagmenal
80-Lower lip lymph nodes: Submental and sub

81-What indicate Diabetic nepheropathy in urine analysis:

RBC case

Hyaline cast

Proteinuria

Hematuria

82-Oxybutin MOA: block muscarinic receptor

83- Osgood case

84: 2 cases of Pattelofemoral syndrome

85: Perthes case

86: elderly with right hip pain and decrease abduction:

Radiography of right hip

87- Incarcerated Hernia: refer ro surgery

88-Gonerrea case. And another Q about test

89- Leprosy case with ulnar thickness

90-meckles in lower ilium

91-painful vesicles in labia and cervix: HSV

92-couple presented to infertility clinic. The man is working in battery manufacturing.

Lead

93-post-mastectomy while having bath nurse noticed winging scapula ?

long thoracic nerve

94-something dureing surgery injured vagus n what organ will be effected ?

esophagus

Jejunum

Ascending colon
Cecum

95-enzymes where pancreas leaks ?

omentum bursa

96-which cells secrets melatonin ?

corticotroph

‫ﺑﻌﺾ ﺍﻻﺳﺎﻟﻪ ﻣﺼﺤﺤﻪ ﻓﺒﺮﺍﻳﺮ‬

vaccine cI in pregnancy

HBV

MMR

Rubella

Haemophilus influenza

2-Patient take Thyroxin for hypothyroidism

High tsh

T3 normal ?

Subclinical hypothyroidism✔

3-Which group contr indicate in diabetic patient with heart faliure?

Biguanides

Dipeptidyl peptidase iv inhibitors

Glp-1 analgies

Sglt 2 inhibitors s

Thiazolidindiones✔

4- Low hb and murmur, how to treat this murmer?

Ttt of anemia✔bi transfogen

5- Child with eczema in knee, flexure elbow, with maximum cortisone dose and eczema
extending, what to do?

Tacrolimus✔
6- Symptomatic AS, ttt?

Surgery✔

7- Prevent dvt in presurgical patient?

8- 56 y o with prolapse uterus, how to dx?‫ﺷﺮﺡ‬

Severe hysterectomy

Not severe uterine suspension ,kegel ex. ,vaginal pessary ,estrogen

9- Cervical cancer screening every?

10- Pt with systolic murmer, Ecg show, non specific t change and St, dx?

As

Obstructive cardiomyopathy

11- First trimester with Vaginal bleeding . Fetal parts felt in the internal os . Open. Diagnosis?

12-First trimester vaginal bleeding. Internal os closed. Diagnosis?

threatened abortion ✔

13- Patient taken throxin for hypothyroidism.

TSH high

T3 normal

T4 normal.explain?

14- Tumor lysis electrolyte

A. Hypocalcemia ✔

B. hypercalcemia

C. Hyponatremia

15- Cause of meningitis in 50 years old?

Listeria✔

16- After 12 hours of gastric slap surgury, pt Came with upper abdominal pain, every thing
normal?

ncrease analgesia ✔
Laporotomy

17- Cholecystitis with dilated duct and stone, inv? Ercp✔

ENDOSCOPIC RETROGRADE CHOLANGIO-PANCREATOGRAPHY

18- Massive pneumothorax after long air travel, ttt?

Decompression then chest tube✔

19- Short female, with htn, wide neck, dx?

Turner's Syndrome✔

20- Preterm baby deliver with dificulty breathing, dx?

21- Surgeon order you to prepare rt kidney but you sure is lt kidney?

1.go to intern supervisor

2.just leave it

3.prepare the rt kidney

4.talk with sourgeon✔

22- High cholesterol, high ldl, ttt?

Statins✔

23- Long term treatment of sickle cell anemia 1.penicilin

2.hydroxyurea✔

24- Phentolamine in ttt of pheochromocytoma act by?

Alpha adrenargic receptor blocker✔

25- Cystic fibrosis symptomatic, dx with chlorid sweet test, father and subling not
symptomatic, what investigation?

Chlorid to father

Chlorid to subling

Gen father✔
Gen subling

26- Type 2 dm, what medication decrese cvs disease?

Atrovastatin ✔

Fibrate

27- Elderly with hemo urea, acystoscop done and show upper most bladder mass, dx?

Transitional cell carcinoma✔

Sequamous cell carcinoma

28- Heart faliure, Best medication?

BB and hydralazine

Acei and diuretic ✔

BBand diureti

29- Lower limb flaccid and areflexia, after urti, dx?

Gbs✔

30- Feature of acute sever asthma?

Low o2 saturation✔

31- Vircella child and his brother immunocompromize?

Vircella ig ✔

32- Patient with eye symptoms lower limb spastisity, what bes inv to reach dx?

MS (MRI brain) or Mysthenia Gravis (send for AChR anbtibody✔‫ﺷﺮﺡ‬

33-

‫ﺍﺧﺘﺒﺎﺭﻱ ﺍﻟﻴﻮﻡ‬24

‫ﺍﻏﻠﺐ ﺍﻻﺳﺌﻠﻪ ﻣﻜﺮﺭﻩ ﺍﻟﺠﺪﻳﺪﻩ ﺍﻟﻠﻲ ﺍﺗﺬﻛﺮﻫﺎ ﺑﺎﻛﺘﺒﻬﺎ‬

*Chag's dis & hershspring dis affect which layer:

Serosa

Musclurais
Mucosa

Submucosa

*Girle with chronic constipation ‫ﺑﻌﺪﻳﻦ ﻗﺎﻝ‬

During Defecation muscle should relax ?

coccygeus

Pibococcygus

Illococcygus

Rectococcygus

*Flat topped lesion other detalis i don't remember with "pruritis" pic attached with multiple red
lesions?

Lichen planus

Mollescum contigusum

Granuloma Anular

*Rash all over the body including palms and soles for 5 days no hx of drug taking "clearly
mention like this in the qs no more detalis"?

Fixed drug eruption

Erythma nodusum

Erythma multiform

Pitryasis rosaea

*Lesion in jagular foramen which will be "preserved"?

Shoukder elevation

Sensation in the throat

Sensation Ant 2/3 of the toung

(Because jagular contain 9,10,11crainl nervs)


*Child run, 3 words sentence , open the door, climbs the furnature but not able to rise the
starines with alternate legs?

1y

2y

3y !!

4y

*Eye going inward which rectus muscle is affected "no further detalis" ?

Lateral

Medial

*Female DM1 has numbness and burning sensation in feet she wish if there's away to help her
loss whight?

Step arobic

Swimming

Jogging

Traidmil

*Femal she afraid from relevator she prefer to goo through staris eventhough its flair her OA
however she think that's better how to manage?

Psychoanalysis

Impitnip

Fluxatine

*42y Female no menstruation in the past 12 months what can be the cause ?

Hypoprolactin
Hypothyrodism

Congenital adrenal hyperplasia

*Pt with sudden Rt eye pain increase when he come to dark room eye has corneal edema &
cilliary flush , fixed dilated pupil (they describe the lesion no pic) with manometry result shown
below

Lt eye externally appear normal but with fundoscopy the upper temporal retina there's a notch

Manometry result :

Rt eye 69 normal 16-22

Lt eye 21 normal 16-22

What's the dignosis?

-Bilatral glucoma

-Rt eye uvitis & congenital change in the Lt

-Rt eye catract with sympathatic change in the Lt

-Rt eye conjunctivitis with something in the Lt i don't rember


Q:You are in OR with consultant and he ask you to prepare Lt. kidney for surgery but you are

sure it is the Rt. Kidney, what to do:

Inform chief intern

Run outside the OR and check the CT image

Inform the consultant about your concern

Q: Parents came to you and they don’t want to give their sun vaccines, how to deal with

them?

Q: case of appendicitis, ask about management

Q: child suffer from tonsillitis, you examine the patient and you find whites patches on it,

what is the management?

Q: man diagnosed with Hep C , the couple ask you what they do:

Abstinence

Use condom
Do intercourse with no restrict

Q: a husband diagnosed with HIV

Tell the husband

Tell the wife

Tell them both

Q: what antibiotic used as prophylaxis in TB?

Q: placenta previa and she has contractions, what to do?

Q: question about familial short stature

Q: managenet of papillary breast cancer

Q: management of ductal breast cancer

Q: 23 y old did cervical screen 3 years ago, ask about when to do it again, what to tell her?

Q: question about pap smear screening, every how much year to do it?
Q: there was 2 questions about diabetic ketoacidosis management

Q: how to diagnose sickle cell disease?

Q: management of cholelithiasis

Q: question about mammogram breast screen

Q: clinical scenario about overian tube torsion

Q: hyperprolactinemia management ( 2 questions, one of them, the wife did MRI and there

is tumor in pituitary gland, the other question ask about the drugs to treat the patient with)

Q: man with both lower limbs burn, ask about fluid replacement

Q: lady came to your clinic and wearing transparent cloths and speak with seductive tone,

and she speak language different than the nurse speak, what to do

Shout at her

Transfer her to another doctor

Call a nurse speak same language


Q: parathyroid tumor management?

Q: case about syphilis, ask about diagnosis

Q: ECG ,types of heart block

Q: questions about ectopic pregnancy, diagnosis and management

Q: clinical scenario about molar pregnancy, asking which type

Q: clinical scenarios about epigastric pain, pancreatic, gastric, esophagus

** you should know how to differentiate **

Q: question about diagnosis of trichomoniasis

Q: US of pregnant shows breech position, both legs and hip flexed, asking about breech

type
‫ فبراير‬٧ ‫متحان الهيئة‬

‫االسئلة القدرت اتذكرها‬

Amebic liver abcess management

Eccinococus liver abcess management

Pyloric stenosis management

Crecendo acrecendo murmur cause

Pleural effusion analysis and management

Meningitis treatment in elderly

Tb infection control

Varicela child and has brother immunodefeceint

Breast feeding hiv and tb mother direct and

expressed milk which is safest

Pregnent has abruptio placente what is the acid base balance of fetus

Asphyxia.. Res acidosis.. Metabolic acidosis

Picture about variable decelation in ctg what is the cause.. Abruptio or cord prolapse

#acute pancreatitis complicated by pseudocyst what is the management

What is the safe for baby mother hiv and direct breast feeding 2.mother hiv with expressed breast
milk 3.mother TB direct breast feed. 4.mother hiv with expreesd breast milk

[6:37 PM, 2/9/2019] +966 59 422 9114: February... 2019

Q.. Diabetic and heavy smoker. With 100 meter . Pain when go to mosque in left leg. During
investigation. Found the right poplitibia artery insuficiant . Best to do?

Smoking cessation

‫مامتذكره الباقي‬

Q... 2 years child with WBC 21.low HB low platelet.

Diagnosis

Q... Patient with vitilligo . Low HB. MCV high. ?

Q... Antibody to intrinsk factor


.b12 level

Diagnosis ‫ كانوا سالوا عن ال‬. ‫ايوا‬

Q.... SLE case with fever and arthritis . Treatment?

Q.... RA Patient with right and left knee arthritis.

What the type of RA

Q.... Empirical treatment of invective endocarditis.

Q... Cause of meningitis in 50 years female

Q... Cynotic heart disease?

TOF ‫مكرر‬

Q... RTA. With

Low urin osmolality . High plasma osmolality.

Q.... Phentolamin in the treatment of pheochromocytoma. Act by?

Alpha

Q.... Di george syndrome.ask about the. Diagnosis

‫مكرر‬

Q... Child with cystic fibrosis symptomatic and diagnosed with chloride sweet test . The father and
sibling are not symptomatic. What investigation to do?

Chloride to father

Chloride to seblling

Gen father

Gen seblling

Q.... Patient with complex reagonal pain after surgery to carpal tunnel . Treatment?

Q... Patint fall from 4 meter. Just has pain in feet.investigation?

X ray feet ‫مامتذكره الباقي‬

Q... Tumor lusis . Electrolytes ?

Hypocalcemia

Hypercalcemia

Hyponetremia
Hypernetremia

Q... Treatment of HB. Is example of

Primary prevention

Secondary

Tertiary

Primou

Q.... U can Brest feed with HBV.

No Brest feed with HIV

Q.... Magnisum sulphate the aim in using it in pre-eclampsia.?

To prevent convulsion🤔

Q... Patient 14 week pregnant. with hypertension?

Essential hypertension

Q.... Cervical cancer screening every?

‫مكرر‬

Q... Vaccine Not to give to low immune?

Q... What vaccine giving in 1year ?

Q... Adrenal hyperplasia. Best investigation?

Q... 56 years old with prolapse uterus. How to diagnose?

Q... Stress incontinence,ttt?

‫مكرر‬

Q... Urge incontinence,give the diagnosis,

Kegal excersi

Q... Child preciously normal up to 6 year. Then he urinate .urinalysis nitrate?

Treatment

Q... IBS treatment

Q... Child with egg sensitivity. What vaccine not to give


Q... Hemoptysis with couch . X ray show rt PE massive Plural effusion. Trachea and heart not
displaced. Diagnosis?

Rupture esophagus

TB

Heart faliure

Mallory wees

Q... Child with dark urin. 2 week early has URTI.?

IgA nephropathy.

Post streptococcus gromerulone

Q... Best medication to releave menstrual pain.?

Q... Child above 95th centile and look larger than other child in same age . Investigation?

Q... K/c of cervical incompitance came for antenatal care. When to do surgery?

[6:37 PM, 2/9/2019] +966 59 422 9114: Q.. Anal figure management.

‫سؤالين‬

Q.. Patient. With eye symptoms. Lower limb spasticity.what is the best inv to reach diagnosis

Q.. Patient on anti TB medication after one month he had high Liver profile. What to do?

Stop all medication

Stop isoniazid

Stop ethambutol

Q.. Patient on anti TV medication and has prepared sensation of needle .drug?

Q.. Child with history of eczema in knee and flexure elbow. With maximum cortisone dose. And the
eczema extending. What to do?

Q.. Counselling the mother about method of doing it or to do it. ‫مامتذكره‬

.reffer to do UV

Give calcitonin

Q.. patient with abdominal pain release with defecation. diarrhea 6month. No wt loss . No
anemia.diagnosis?

Microscopic appearance of ceailiac disease

Q.. What type of communication


Q.. Child with croup. Repeat Epinephrine

Q.. Double bubble

Q.. Screening of cervical CA. To which age group

Q.. CA colon screening age?

Q.. Symptomatic AS. Treatment?

Q.. Severe Mortal stenosis. ? Valve gradient.

1‫اقل من‬

Q.. 4 year Child with fever . Then convulsions.treatment?

Q.. Status epileptics treatmentf

Q.. Aortic stenosis. Surgery if symptomatic

Q.. Partner came with infertility. And you discover the father has HIV .

Tell the father

Tell mother

Tell polic😂

Q.. Male with Gun shots. Inter lateral to left nipple .and exit under the scapula. The patient
collapsed in ER. What to do?

Iv.fluid

Pericardiocentesis

Q.. Cholysistitis with dilate duct and stone. INV?

Ercp

US

Q.. Fever Lowe eye lid swelling redness .

Low protin

Abces

Q.. case about HUS.


Q.. History of bloody diarrhea .

Q.. Progesterone only pills increase what CA?

Q.. Ulcerative colitis case. Diagnosis

Q.. Massive Pneumothorax after long air travel. Treatment?

Q.. Polyps in colon. With history of blood .diagnosis?

Hereditary polyp Isis

.‫مامتذكره باقي الخيارات‬

Q.. Counselling to TB patient. ?

Contact

Air born

Q.. Child born with large tongue ...( hypothroidism)

Treatment?

Thyroxin for 1 month

Thyroxin for life

..‫مامتذكره الباقي‬

Q.. Meningitis case with skin rash. ?

Airborne

Contact

Q.. Pulsating abdominal aorta,DIAGNOSIS?

‫مكرر‬

Q.. Chang between tow arm pulsation . Best diagnostic?

Q.. Right heart failure. What medication show to be effective?

Betablocker

Q.. Heart failure. Best medication .?

1.Betabloker and hydralizine

2.acei and diuretic

3.beta blocker and diuretic


4 betabloker and nitrate

Q.. Prevent dvt in presurgical patient?

Q.. Ductal cell carcinoma of Brest. Colour of discharge?

White

Red

Green

[6:37 PM, 2/9/2019] +966 59 422 9114: Q.. Salpingiotomy done for tubal pregnancy. After 6 weeks.
Bhcg high . What's to do?

Q.. Lower and upper limb flaccid..and arefleixa after URTI.diagnosis?

Q.. Alarming sign in stridor patient? Cynosiis

‫مكرر‬

Q.. Pregnant lady with no past history of chicken box .best to prefect her from disease?

.avoid contact with patient

.IVIG

.acyclovir

Q.. Down syndrome patient. with fixed split S2. And pansystolic murmur in left sternum. What
cardiac abnormality?

Q.. Short female. With hypertension wide neck. Diagnosis?

Q.. Diagnostic tests for eradication H.P?

Q.. Urea breath test

Q.. Elderly with hem a you're a.cystoscopy done and show in the uppermost of bladder mass?

.Transitional cell carcinoma

.squamous cell carcinoma

Q.. ‫ سوالين عن ال‬external cephalic version contraindicated in?

Low lying placenta

Poly hydrominus

Long umbilicus cord

Q.. ٢ month old child when he supin there is abnormal sound disappear when he in probate
posion.🤔(‫)مش متاكده هل االصوات بعد االكل او االصوات من التنفس‬
What to said to mother?

1.Reassurance that is normal

2.It will disappear in the first year.

Q.. Low hb and murmur.how to treat this murmur? ‫طبعا السيناريو طويل كم سطر وكالم مش مهم‬

Q.. 2question about Testicular torsion treatment.?

‫من البرتقاليه‬. ‫مكرر‬

Q.. Patient with AF has lower limb ischemia .treatment?

Q.. Preterm baby deliverd with difficult breathing .diagnosis?

Preterm appear

Respiratory distress syndrome ‫مامتذكره الباقي‬

Q.. Pregnant 36.

LAnswer

‫ماتنامي علي ظهرك‬

‫مكرر من البرتقاليه‬

Q.. Uterus Compress inferior Vena cava

Hypotension

Q.. What is the leading cause of death in hemodylisis patron?

Q.. Pulmonary symptoms and raised jvp hepatomegaly and lower limb swelling.diagnostic test?

Q.. Case with pancytopenia ANA+ve and arthritis.diagnosis?

Q.. Rt side heart cathetrization 🤔 ‫تقريبا‬

‫عشان ال‬pulmonary hpertension

Q.. Post MI bradycardia.treatment+

Q.. Smoker for long time came with couph white sputum.. EFV less than 70% what is the diagnosis?

Chronic bronchitis

Bronchial asthma

Lung fibrosis

Q.. Asthmatic pat. On short beta blocker and low dose steroid. And still have asthmatic attack.next
step ?
Add long acting beta to steroid

Q.. Anterior resection of colon . Supply from which artery?

Q.. After 12 hour Gastric slap surgary. The patient came with upper abdominal pain . Every thing in
normal.?

1.Increase analgesia

2.laprotomy ....

Q.. ln child what is the symptoms that indicat there is disease ?

1.Pain around ambalicus

2.Pain at night when patient sleep

3.Pain when patient awake

Q.. Firm mass in breast.diagnosis? Fibroadenoma

Q.. Pregnant lady with late deceleration . And bleeding?

Placenta abruptio .

‫مكرر من البرتقاليه‬

Q.. Pregnant with massive bleeding from abruptio placentea . Treatment to save her life?

1.admitted to ICU with obstetric team

Admitted with different spacilized team

[6:37 PM, 2/9/2019] +966 59 422 9114: Q.. Patient with history of chest pain since 4 week ago
.increasing . In rest and movement.diagnosis?

Prizemental

Unstable angina

Heberden

Stable angina

Q.. Patient with systolic murmur, ecg show undetermined(non spacific) T changes and St.diagnosis?

AS

Obstructive cardiomyopathy

Q.. High 125 marker.? Ovarian tumor


Q.. Type 2 DM . What medication decrease cardiovascular disease?

Atorvastatin

Fibrat

Q.. High cholesterol

High ldl. Treatment?

Q.. Child sleep with milk bottle in his mouth. He develop dental Carie's. Diagnosis?

1.Bottles induced something...

Q.. Patient com with convulsion . Lymph node . Rash .....?‫مامتذكره الباقي‬

HIV serology

Q.. Patient with constipation and pain full. Bugging .?

Fissure

Hemorrhoid thrombus

Q.. Feature of acute sever asthma.?

Q.. Vomiting after 4 or 6 hour eating?

Staph aureous

Q.. Picture of Urin dipstick test. +2 proteinurea

...And scenario about pregnant female with bp140/90.treatment?

Q.. Patient taken throxin for hypothyroidism.

TSH high

T3 normal

T4 normal.explain?

[6:42 PM, 2/9/2019] +966 53 302 1718: Which group of drugs is contraindicated in diabetic patients
with heart

failure?

A. Biguanides

B. Dipeptidyl peptidase IV inhibitors

C. GLP-1 analogues
‫‪D. SGLT 2 Inhibitors‬‬

‫‪E. Thiazolidinediones‬‬

‫‪[6:42 PM, 2/9/2019] +966 53 302 1718: What is hemosiderin deposition in macrophages in the lung‬‬
‫‪in‬‬

‫?‪indicative of‬‬

‫‪A. CMV‬‬

‫‪B. Chronic lung infection‬‬

‫‪C. Heartfailure‬‬

‫‪D. Pneumocystis jiroveci‬‬

‫األسئلة اللي جتني باالثيكس‬

‫سوال واحد زعالن انك تأخرت عليه بسبب انشغالك وشلون تتعامل معه *‬

‫واحد رفض العالج مع انه يحناجه تلقى الجواب انك تصنحه وتحاول تقنعه *‬

‫وحد حامل ووصار معها ورم والزم تشيله ورفضت تشيله وش تسوي معها *‬

‫اب وام جايبين ولدهم عمره كم سنة بس ما طعموه وال مرة ‪ ..‬يقولون ان التطعيم يضر المخ ‪ ..‬الجواب انك تصحح لهم مفاهيمهم *‬
‫وتشرح لهم‬
Q:You are in OR with consultant and he ask you to prepare Lt. kidney for surgery but you are
sure it is the Rt. Kidney, what to do:

Inform chief intern

Run outside the OR and check the CT image

Inform the consultant about your concern

Q: Parents came to you and they don’t want to give their sun vaccines, how to deal with
them?

Q: case of appendicitis, ask about management

Q: child suffer from tonsillitis, you examine the patient and you find whites patches on it,
what is the management?

Q: man diagnosed with Hep C , the couple ask you what they do:

Abstinence

Use condom

Do intercourse with no restrict

Q: a husband diagnosed with HIV

Tell the husband

Tell the wife

Tell them both

Q: what antibiotic used as prophylaxis in TB?

Q: placenta previa and she has contractions, what to do?

Q: question about familial short stature

Q: managenet of papillary breast cancer


Q: management of ductal breast cancer

Q: 23 y old did cervical screen 3 years ago, ask about when to do it again, what to tell her?

Q: question about pap smear screening, every how much year to do it?

Q: there was 2 questions about diabetic ketoacidosis management

Q: how to diagnose sickle cell disease?

Q: management of cholelithiasis

Q: question about mammogram breast screen

Q: clinical scenario about overian tube torsion

Q: hyperprolactinemia management ( 2 questions, one of them, the wife did MRI and there
is tumor in pituitary gland, the other question ask about the drugs to treat the patient with)

Q: man with both lower limbs burn, ask about fluid replacement

Q: lady came to your clinic and wearing transparent cloths and speak with seductive tone,
and she speak language different than the nurse speak, what to do

Shout at her

Transfer her to another doctor

Call a nurse speak same language

Q: parathyroid tumor management?

Q: case about syphilis, ask about diagnosis

Q: ECG ,types of heart block


Q: questions about ectopic pregnancy, diagnosis and management

Q: clinical scenario about molar pregnancy, asking which type

Q: clinical scenarios about epigastric pain, pancreatic, gastric, esophagus

** you should know how to differentiate **

Q: question about diagnosis of trichomoniasis

Q: US of pregnant shows breech position, both legs and hip flexed, asking about breech
type
26 yrs male did appendectomy then was discharged next day as he was well , 8 hours later he
came back to hospital with mild pain at site of incision ,US reaveled collection 2 cm x 2 cm , HB
11 TLC 7.9 bp 110/70 p.76 RR 16

How would you do

A. Exploration

B.observation

C. CT

D. Needle suction

31 yrs p4 8 wks of gestation , severe lower Abd pain ,moderate vag. Bleeding ,examined CX is
open ,conceptional content at CX

Ur DX

A.missed abortion

B.complete

C.threatened

D.inevitable

24 yrs primigravida. Married 6 months ago, hx of irregular menses , at ER complaining lower


Abd pain ,scanty vag. Bleeding , by us uterus empety ,rt adnexal mass 2,5 cm, bhcg 2500 ,bp
120/70

P78 RR 14

Management

A.surgical

B.medical

C.medical plus surgical


18 yrs primigravida . Married 6 months ago, hx of irregular menses , at ER complaining lower
Abd pain , bloody vag discharge , us and urinalysis normal ,bp 120/70

P78 RR 14

Next step

A.pregnancy test

B.duplex us

C.ct angio

D.laparotomy

G2 p1002

8 wks

Routine antenatal care us no pulsation

A.threatened abortion

B.missed

C.septic

‫ﻧﻔﺲ ﺍﻟﺼﻮﺭﺓ ﻗﺎﺱ ﻗﻄﺮﻫﺎ ﻭﻛﺘﺐ ﻋﻠﻴﻪ‬275 ‫ ﻣﻢ‬best management

A.conservation

B.termination of preg

C. Progesterone

56 yes postmenopausal her pap smear hx was normal except one month ago revealed ASCUs ,
then given local estrogen for one month then pap smear repeated revealed intraepithelial lesion

Next step is

A.conization

B.hysterectomy

C.colposcpy
Pap smear screening is done every

A.2 years

B.3

C.5

64 years post menopausal bleeding since 5 days HB 11

Pt ,PTT normal,tvus revealed normal except endometrial thickness 13mm

Best management

A.progesterone

B.endometrial biopsy

C.hysterectomy

D.reassure

29 yrs male did sleeve gastrectomy 6 days ago,

Come to ER complaining persistent vomiting since operation , examination done and of no


significance, Abg normal ‫ ﻛﺎﻥ ﺍﺭﻗﺎﻡ‬,us normal

Best management

A.laparoscopy

B.Admissions

C.laparotomy

D.reassure

9 years boy ‫ ﺟﻪ ﺍﻟﻌﻴﺎﺩﺓ ﻣﻊ ﺍﺑﻮﻳﻪ ﻳﺸﺘﻜﻮﻥ ﻣﻦ ﻗﺼﺮ ﻃﻮﻟﻪ‬، he appears healthy ,less than 50 % percentile
for height , his profile reaveled progressive increase in height, his labs was normal, best
indicator of his future height is

A. His weight vs height

B.parental height

C.parental weight

D. Can't be expected
Child admitted in ER due to DKA ,severely dehydrated

Managed with fluid and insulin , bad management complications can occurs

A.hyperkalemia

B.hypoglycemia

C.brain oedema

D.hyperglycemia

Child in Er DX as DKA managed by fluid and insulin ,serum electrolytes will be imbalanced

A.can

B.k

C.na

D.mg

‫ﻭﺍﺣﺪﺓ‬٤٣ ‫ﺳﻨﺔ ﻣﻌﺎﻫﺎ‬3 ‫ ﺍﻃﻔﺎﻝ ﺍﻭﻝ ﻣﺮﺓ ﻭﻟﺪﺕ ﻗﻴﺼﺮﻱ ﻭﺍﻟﺘﺎﻧﻴﻴﻦ ﻧﻮﺭﻣﺎﻝ ﺟﺎﻳﺔ ﻓﻲ ﺍﻟﻢ ﻭﻻﺩﺓ‬36 wks

Rupture of membrane , by us placenta previa, AFI 4 cm

,CX is 2 cm

50% effaced

Absolute CI for ECV is

A.previous CS

B.her age

C.us result

31 yrs 37 wks ROM in labor


Examination revealed breach presentation , ctg reactive

PV CX 2cm ,50% effaced ,mid anterior CX

Ur management

1.cs

2.oxytocin

3. Vag. Prostaglandin

4.iv antibiotics

‫ﻋﺸﺎﻥ ﺍﺩﻱ ﻣﻀﺎﺩ ﺣﻴﻮﻱ ﻻﺯﻡ ﻳﻜﻮﻥ ﻋﺪﻱ‬١٨ ‫ ﺳﺎﻋﺔ ﻋﻠﻲ‬ROM

‫ﺍﻣﻪ ﻗﺎﻟﺘﻠﻚ ﺑﻘﺎﻟﻪ ﺳﺎﻋﺔ ﺑﻴﻌﻴﻂ‬، ‫ﻋﻴﻞ ﻋﻤﺮﻩ ﺷﻬﺮﻳﻦ ﻣﺘﺴﺨﺺ ﺍﻧﻪ ﻋﻨﺪﻩ ﻋﻴﺐ ﺧﻠﻘﻲ ﻑ ﺍﻟﻘﻠﺐ ﺟﻪ ﺍﻟﻄﻮﺍﺭﺀ ﻋﻨﺪﻩ ﺯﺭﻗﺔ ﻓﻲ ﺍﻟﺠﻠﺪ‬
‫ﺗﻌﻤﻞ ﺍﻳﻪ‬

A.immediate heart surgery

B.reexamine after baby calmed

C.go home

D.refer to another doctor

‫ﻭﺍﺣﺪ ﺗﺨﻴﻦ ﻭﻋﻨﺪﻩ ﺑﻔﻠﻮ ﻫﻤﺐ ﻭ‬sretch marks in skin

Ur diagnosis

A.chushing

B.chines

C.pheochromocytoma

‫ﻭﺍﺣﺪﺓ‬
47 ‫ ﺳﻨﺔ ﻣﻦ‬3‫ ﺳﻨﻴﻦ ﺷﺎﻟﺖ ﻛﻴﺲ ﻣﻦ ﻉ ﺍﻟﻤﺒﻴﺾ‬chochlote cyst

‫ﺟﺎﻳﻪ ﺑﺘﺸﺘﻜﻲ ﻣﻦ‬

Menorrhagia, dysuria, us revealed another chocolate cyst on other ovary dysparunia,she


completed her family

Best management is

1 .laparoscopy

2.hysterectomy

3.

Cystectomy

4 .follow up

‫ﻃﻔﻞ ﻋﻤﺮﻩ ﺷﻬﺮ ﺟﺎﺗﻠﻪ ﺻﻔﺮﺍ ﺗﺎﻟﺖ ﻳﻮﻡ ﻭﺍﺗﺸﺨﺺ ﺍﻧﻪ ﻓﺴﻴﻮﻟﻮﺟﻴﻚ ﺻﻔﺮﺍ ﻧﺎﻭ ﺟﺎﻱ ﺟﺴﻤﻪ ﻭﻋﻨﻴﻪ ﺻﻔﺮﺍﺀ‬

Total bilirubin 25

‫ﺍﻟﺴﺒﺒﺎﻳﻪ‬

A. Rh incomputable

B.ABO incompetable

C.c.najjar

D.gilbert

15 yrs female not menstruating yet at clinic by her parents ,she is short stature , short neck ,
hypertensive

Most appropriate diagnosis

A.Turner

B.down

C.TFS

D.normal variant
‫ﻋﻴﻞ ﻋﻤﺮﻩ‬3‫ﻟﺴﺎﻧﻪ ﻃﺎﻟﻪ ﻣﺘﺎﺧﺮ ﻓﻲ ﺍﻟﻨﻤﻮ‬، ‫ ﺳﻨﻴﻦ ﺷﻐﺮﻩ ﻧﺎﻋﻢ‬،single palmar crease

DX

A.turner

B.down

C.marfan

D.pateu

‫ ﻃﻔﻞ ﺍﺗﻌﻤﻠﻪ ﻻﺑﺲ ﻃﻠﻌﺖ‬tsh ‫ﻋﺎﻝ‬

Total T4 ‫ﻭﺍﻃﻲ‬

Next step

A.start throxine

B.Abd.us

C.brain MRI

D.brain CT

‫ ﻭﺍﺣﺪ‬HF ‫ ﻣﺎﺷﻲ ﻋﻠﻲ‬captopril,BB,lasix

‫ ﻛﻞ ﺍﻟﻔﺤﻮﺻﺎﺕ ﻛﻮﻳﺴﻪ ﻣﺎﻋﺪﺍ ﺍﻧﻪ ﻋﻨﺪﻩ‬chronic dry cough

‫ﺍﻳﻪ ﺍﻟﺴﺒﺐ‬

A.BB

B.captoprill

C.lasix

D.drug irrelevant

‫ﻭﺍﺣﺪﺓ ﺣﺎﻣﻞ‬39 ‫ ﺍﺳﺒﻮﻉ ﺿﻐﻄﻬﺎ‬150/90

‫ﺗﺤﻠﻴﻞ ﺍﻟﺒﻮﻝ ﻃﺒﻴﻌﻲ‬،

‫ ﻣﻔﻴﺶ ﺻﺪﺍﻉ ﺯﻋﻠﻠﻪ ﻑ ﺍﻟﻌﻴﻦ‬،nor epigasric pain.her previous bp measures was below 110/70

Ur DX

Preeclampsia

Eclampsia
Gestational hypertension

Preeclampsia imposed htn

‫ ﻭﺍﺣﺪﺓ ﺣﺎﻣﻞ ﺣﺎﺳﺔ ﺏ‬hard lump at epigastrium

FHS is heard at umblicus

Fetal presentation is

A.face

Brow

Breech

OP

‫ ﻭﺍﺣﺪ ﻋﻨﺪﻩ ﺣﺼﻮﺍﺕ ﻓﻲ ﺍﻟﻤﺮﺍﺭﺓ ﺟﺎﻱ ﺑﻴﺸﺘﻜﻲ ﻣﻦ ﺻﻔﺮﺍ ﻭﺍﻟﻢ‬epigastric pain, his labs
alt,at,alp,bilirubin,amylase are high

Next step

A.us

B.ct

C.ERCP

‫ ﻭﺍﺣﺪﺓ ﺣﺎﻣﻞ ﻣﺶ ﺑﺘﺸﺘﻜﻲ ﻣﻦ ﺣﺎﺟﺔ‬her urine analysis shows nitrite , leucocytes

,then culture revealed 50.000organism

DX

A.cystitis

B.pyelonehritis

C.asymptomatic bactururia

‫ﻭﺍﺣﺪ ﻋﻨﺪﻩ ﺳﺨﻮﻧﺔ ﻭﺭﻋﺸﺔ‬

fever ,lower back pain , his urine +leucocytes ,+nitrite

How to manage

A.oral antibiotic

B.iv abx at home


C.admission

‫ ﻃﻔﻞ ﻋﻨﺪﻩ ﻛﺴﺮ ﻓﻲ‬tibia

٣٠ ‫ﺩﺭﺟﺔ‬angulation ‫ ﺑﻌﺪ‬close reduction

How to manage

A.cast

B ‫ﻣﺴﻤﺎﺭ ﻧﺨﺎﻉ ﻋﻈﻢ‬

C.internal fx with plate

‫ ﻛﺎﻥ ﺟﺎﻳﺐ ﺭﺳﻢ ﻗﻠﺐ ﻓﻴﻪ‬st segment elevation in lead 2,3,avf

Our DX

Anterior mi

Lateral mi

Inferior

‫ﺻﻮﺭﺓ ﻭﺍﺣﺪﺓ ﺣﺎﻣﻞ ﻋﻨﺪﻫﺎ ﻧﻤﺶ ﻓﻲ ﻭﺷﻬﺎ ﻇﻬﺮ ﻣﻊ ﺗﻘﺪﻡ ﺍﻟﺤﻤﻞ‬

Ur dx

Choloasma

Loloasma

Sle

Urticaria

‫ ﺻﻮﺭﺓ ﺑﻠﻮﺩ ﻓﻴﻠﻢ ﻭﻋﺎﻃﻴﻚ ﺑﻴﺎﻧﺎﺕ‬HB 9

MCV 65 ‫ﺻﻐﻴﺮﺓ ﻳﻌﻨﻲ‬

TIBC ‫ﺭﻗﻢ ﻋﺎﻟﻲ ﺍﻭﻱ‬

‫ﺍﻳﻪ ﺍﻟﺘﺸﺨﻴﺺ‬

IDA

Folate def.

Bit b12 def.

‫ﻭﺍﺣﺪ ﻋﻨﺪﻩ ﺍﻟﺘﻬﺎﺏ ﻣﻔﺎﺻﻞ ﻭﺑﻴﺎﺧﺪ ﻣﺴﻜﻨﺎﺕ ﻟﻤﺪﺓ ﻃﻮﻳﻠﺔ ﻭﻋﻨﺪﻩ ﺍﻧﻴﻤﻴﺎ‬
MCV ‫ ﺻﻐﻴﺮﺓ‬HB 9

‫ﺍﻟﻌﻼﺝ‬

Oral iron

I'm iron

FFPlasma

Packed RBcs

‫ﺻﻮﺭﺓ ﺯﻱ ﺩﻱ ﻧﻔﺲ ﺍﻟﺴﻨﺎﺭﻳﻮ ﻭﻧﻔﺲ ﺍﻟﺼﻮﺭﺓ ﺟﻪ ﺳﺆﺍﻟﻴﻦ‬

pt 13wks gestation during routine ANC us reveealed as in picture ur dx

1- missd abortion 2- anmeryonic prgnancy 3 thratned abortion

‫ ﺑﻴﻘﻮﻟﻚ ﻭﺍﺣﺪﺓ ﺍﺛﻨﺎﺀ ﺍﻟﻮﻻﺩﺓ ﺍﻟﻄﺒﻴﻌﻲ ﺍﻳﻪ ﺍﻟﻮﺿﻊ ﺍﻟﻠﻲ ﻳﺨﻠﻴﻬﺎ ﺗﺠﻴﺐ‬fourth degree perineal tear

A.unrestration ‫ ﻛﻠﻤﺔ ﻏﺮﻳﺒﺔ ﻛﺪﺓ‬in semisettiinng position

B.‫ﻧﻔﺲ ﺍﻟﻜﻠﻤﺔ ﻓﻲ ﻭﺿﻊ ﺍﻟﻘﺮﻓﺼﺎﺀ‬

C.‫ﻧﻔﺲ ﺍﻟﻜﻠﻤﺔ ﻭﻗﺎﻋﺪﺓ ﻋﻠﻲ ﻛﺮﺳﻲ‬

4 . ‫ﺣﺎﺵ ﺍﻟﻤﻘﻄﻊ‬un

‫ﻣﻦ ﺍﻟﻜﻠﻤﺔ‬
‫ ﻭﺍﺣﺪﺓ ﺍﺗﺠﻮﺯﺕ ﻭﺧﻠﻔﺖ ﻋﻴﻞ ﺍﻋﺘﻘﺪ ﻋﻨﺪﻩ‬sicle cell anemia

‫ﺍﺗﺠﻮﺯﺕ ﺗﺎﻧﻲ ﻭﺧﺎﻳﻔﺔ ﺗﺨﻠﻒ ﻧﻔﺲ ﺍﻟﻤﺮﺽ‬

‫ﺗﻌﻤﻞ ﺍﻳﻪ‬

A.karyotype ‫ﻟﻠﺰﻭﺝ‬

B.karyo ‫ﻟﻴﻬﺎ‬

C,HB electrophoresis ‫ﻟﻴﻬﺎ‬

D‫ﻟﻠﺰﻭﺝ‬

‫ ﻃﻔﻞ ﻛﺎﻥ ﻋﻤﺮﺓ ﺍﻗﻞ ﻣﻦ ﺳﻨﺔ ﺍﻋﺘﻘﺪ ﻭﻛﺎﻥ ﻋﻨﺪﻩ ﻧﺰﻟﺔ ﻣﻌﻮﺑﺔ ﺍﺳﻬﺎﻝ ﻭﺗﺮﺟﻴﻊ‬oriented,aware,irritible,dry tongue,
sunken eye,normal

pulse volume dehydration degree

A.<١٪

B.5-9%

C>10%

‫ﻭﺍﺣﺪﺓ ﺣﺎﻣﻞ‬42‫ ﺍﺳﺒﻮﻉ ﻭﺗﻼﺕ ﺍﻳﺎﻡ‬pg.bishop score 8,ctg reactive,no cpd

How to manage

A.expectant

B.cs

C.iol

D.decide after one week

‫ ﻃﻔﻞ ﻣﺶ ﻗﺎﺩﺭ ﻳﺮﻛﺰ ﻓﻲ ﺍﻟﺪﺭﺍﺳﺔ ﻋﻨﺪﻩ ﺻﺪﺍﻉ‬،hb ,7.9

MCV less normal

TIBC high

How to manage

Oral iron

Oral folate
Oral vitb12

Packed rbcs transfusion

‫ ﻭﺍﺣﺪ ﻣﺮﻳﺾ ﺳﻜﺮ ﻣﺎﺷﻲ ﻉ ﺍﻧﺴﻮﻟﻴﻦ ﻭﻋﻨﺪﻩ ﺗﻨﻤﻴﻞ ﻑ ﺍﻻﻃﺮﺍﻑ ﻭﻧﺴﻴﺎﻥ‬HB 10

MCV large how to manage

Oral iron

oral folate

Oral vit b12

I'm vit b 12

‫ ﻃﻔﻞ ﻋﻨﺪﻩ‬polyuria poly dipsia

OGTT normal

Na ,,‫ ﻋﺎﻝ‬cl ‫ﻋﺎﻝ‬

‫ﺑﺎﻗﻲ ﺍﻝ‬electrolyte normal

Ur DX

A.DM

B.di

C.siadh

Conns

‫ﺟﻪ ﺭﺳﻢ ﺗﺨﻄﻴﻂ ﺟﻨﻴﻦ ﻣﻮﺿﺢ‬3‫ ﺍﻧﻘﺒﺎﺽ ﺭﺣﻤﻲ ﻣﻦ ﻏﻴﺮ‬deceleration

Ur DX

Head compression

U cord compression

Uteroplacental insufficiency

Non of above

‫ﻭﺍﺣﺪﺓ ﺣﺎﻣﻞ ﻓﻲ‬٤٣


‫ﺍﺳﺒﻮﻉ ﻛﺎﻥ ﻋﻨﺪﻫﺎ ﺷﻔﺖ ﺑﺎﻟﻠﻴﻞ ﻓﻲ ﺍﻟﺸﻐﻞ ﻭﻣﻜﻨﺘﺶ ﺣﺎﺳﺔ ﺣﺮﻛﺔ ﺍﻟﺒﻴﺒﻲ ﺍﻟﻤﻌﺘﺎﺩﺓ ﺑﻌﺪ ﺍﻟﺸﻔﺖ ﺭﺍﺣﺖ ﻟﻠﺪﻛﺘﻮﺭ ﻋﻤﻠﻬﺎ ﺭﺳﻢ‬
‫‪ reactive‬ﺗﺨﻄﻴﻂ ﺟﻨﻴﻦ ﻛﺎﻥ‬

‫‪Ur management‬‬

‫‪Urgent cs‬‬

‫‪Bpp‬‬

‫‪Refer for admission‬‬

‫‪Reassure‬‬

‫ﺳﻨﻴﻦ ﺍﻣﻪ ﻭﻫﻴﺎ ﺑﺘﺤﻤﻴﻪ ﻻﺣﻈﺖ ﻣﺎﺱ ﻓﻲ ﺍﻟﺒﻄﻦ ﺭﺍﺣﺖ ﻟﻠﺪﻛﺘﻮﺭ ﻋﻤﻠﻬﺎ ﺳﻮﻧﺎﺭ ﻟﻘﻲ ﻓﻌﻼ ﻣﺎﺱ ﻓﻲ ﺍﻟﺒﻄﻦ‪9‬ﻃﻔﻞ‬

‫‪Ur most appropriate DX‬‬

‫‪A.wilms‬‬

‫‪B....‬‬

‫‪C....‬‬

‫ﻋﻴﻞ ﻋﻨﺪﻩ ﺳﺒﻊ ﺳﻨﻴﻦ ﺑﻴﺘﻌﺎﺑﺞ ﻣﻦ ﻣﺮﺽ ﻣﻌﻴﻦ ﻭﻣﺶ ﻣﻠﺘﺰﻡ ﺑﺎﻟﻌﻼﺝ ﺍﻭﻱ ﺍﻧﺖ ﻛﻄﺒﻴﺐ ﺗﻌﻤﻞ ﺍﻳﻪ‬

‫ﺗﺸﺮﺡ ﻟﻼﺑﻮﻳﻦ ﺍﻫﻤﻴﺔ ﺍﻻﻟﺘﺰﻡ ﺑﺎﻟﻌﻼﺝ‪A.‬‬

‫ﺗﺤﻮﻟﻬﻢ ﻟﻠﺠﻨﺔ ﺍﻻﺧﻼﻗﻴﺎﺕ‪B.‬‬

‫ﺗﺸﺎﺭﻙ ﺍﻟﻄﻔﻞ ﻭﺍﻻﺑﻮﻳﻦ ﻑ ﺧﻄﺔ ﺍﻟﻌﻼﺝ‪C.‬‬

‫ﺗﺒﻠﻎ ﺍﻟﺸﺮﻃﺔ‪C.‬‬

‫ﻣﺮﻳﻀﺔ ﺟﺎﺕ ﺍﻟﻌﻴﺎﺩﺓ ﻻﺑﺴﺔ ﻟﺒﺲ ﺷﻔﺎﻑ ﻭﺍﺛﻨﺎﺀ ﺍﻟﻜﺸﻒ ﻗﺮﺑﺖ ﻣﻦ ﺍﻟﺪﻛﺘﻮﺭ ﻭﻫﻤﺴﺖ ﺑﻜﻼﻡ ﺑﻠﻐﺔ ﻻﺗﻔﻬﻤﻬﺎ ﺍﻟﻤﻤﺮﺿﺔ ﺍﻟﻤﻮﺟﻮﺩﻩ ﻣﻊ‬
‫ﺍﻳﺤﺎﺀﺍﺕ ﻏﻴﺮ ﻻﺋﻘﺔ‬

‫ﻣﺎﺫﺍ ﻳﻨﺒﻐﻲ ﻉ ﺍﻟﻄﺒﻴﺐ ﻓﻌﻠﻪ‬

‫ﻳﺰﻋﻘﻠﻬﺎ‪A.‬‬

‫ﻳﻐﻴﺮ ﻟﺪﺍﻟﻤﻤﺮﺿﺔ ﺑﻮﺍﺣﺪﺓ ﺗﺘﻜﻠﻢ ﻧﻔﺲ ﻟﻐﺔ ﺍﻟﻤﺮﻳﻀﺔ‪B.‬‬

‫ﻳﺒﻠﻎ ﺍﻣﻦ ﺍﻟﻤﺸﻔﻲ‪C‬‬

‫‪D......‬‬
‫ﻭﺍﺣﺪﺓ ﺗﻌﺎﻧﻲ ﻣﻦ ﻋﻘﻢ ﻣﻦ‬١٠ ‫ﺳﻨﻮﺍﺕ ﺣﻤﻠﺖ ﺍﻻﻥ ﻓﻲ‬10 ‫ﺍﺳﺎﺑﻴﻊ ﻭﺟﺎﻟﻬﺎ ﻭﺟﻊ ﻓﻲ ﺍﻟﺒﻄﻦ ﻭﻧﺰﻳﻒ ﻣﻬﺒﻠﻲ ﺭﺍﺣﻮﺍ ﺍﻟﻄﻮﺍﺭﻱ ﺍﻟﻄﺒﻴﺐ‬
‫ﺍﺑﻠﻐﻬﻢ ﺍﻥ ﻣﻔﻴﺶ ﻧﺒﺾ ﻭﺩﻩ ﺍﺟﻬﺎﺽ ﻭﻗﻊ ﺍﻟﺨﺒﺮ ﻋﻠﻴﻬﻢ ﻛﺎﻟﺼﺎﻋﻘﺔ ﺍﻟﻤﻄﻠﻮﺏ ﻣﻦ ﺍﻟﻄﺒﻴﺐ‬

Sympathy

Admission after evacuation as post partum care

Search for cause of abortion

‫ ﻃﻔﻞ‬elevate his head when probated

,turn his head left and rights ,coombs ,his age

A.3 months

B.4

C.5

D.6

‫ﻭﺍﺣﺪﺓ ﻋﻨﺪﻫﺎ ﻣﺮﺍﺭﺓ ﻓﻴﻬﺎ ﺣﺼﻮ ﻣﻦ ﺧﻤﺲ ﺳﻨﻮﺍﺕ ﻣﺴﺒﺒﺎﻟﻬﺎ ﺍﻟﻢ ﺩﺍﻳﻤﺎ ﻭﺑﺘﺎﺧﺪﻟﻬﺎ ﻋﻼﺝ ﺩﻟﻮﻗﺖ ﺣﺎﻣﻞ ﻓﻲ‬

12 wks

Still complaining pain

When to do cholsystectomy

1.now

2 .2nd trimester

3.3rd trimester

4.after delivery
‫ﺷﺎﺏ‬٢١‫ ﺳﻨﻪ ﺟﺎﻟﻪ ﻭﺟﻊ‬around umbilicus then traveled to rt iliac quadrant tenderness ,rebound
tenderness,positive rovsing sign

Vomiting, constipation

DX

A.polyposis

B.appendicitis

C.intestinal obstruction

D.intussception

‫ ﻃﻔﻞ‬tell story ‫ﻛﻢ ﺳﻨﺔ‬

‫ﻣﺮﻳﺾ‬٧٥‫ ﺳﻨﺔ ﻋﻨﺪﻩ‬terminal I'll lung cancer

‫ﺍﻟﺠﺮﺍﺡ ﻋﺎﻳﺰ ﻳﻌﻤﻠﻪ ﺟﺮﺍﺣﺔ‬،‫ﻗﺪﺭﺗﻪ ﻋﻠﻲ ﺍﺗﺨﺎﺫ ﺍﻟﻘﺮﺍﺭ ﻣﺶ ﺣﻠﻮﺓ‬، ‫ ﻣﺶ ﻗﺎﺩﺭ ﻳﺘﻨﻔﺲ‬debulking of cancer to help him
breathing well ‫ﻭﻧﺠﺎﺡ ﺍﻟﻌﻤﻠﻴﺔ ﻳﻌﻨﻲ ﺍﻧﻪ ﻫﻴﻌﻴﺶ ﺑﻘﻴﺔ ﺍﻳﺎﻣﻪ ﺍﻓﻀﻞ‬، ‫ﻟﻜﻦ ﻋﺎﺭﻑ ﺍﻥ ﻣﺨﺎﻃﺮ ﺍﻟﺠﺮﺍﺣﺔ ﻭﺍﻟﺘﺨﺪﻳﺮ ﻋﻠﻴﻪ ﻛﺒﻴﺮﺓ‬
‫ﻣﺎﺫﺍ ﻳﻨﺒﻐﻲ ﻓﻌﻠﻪ ﻋﻠﻲ ﺍﺑﻄﺒﻴﺐ‬

A.go ahead surgery

B.informed consent pt

C.search alternative decision maker consent


Ectopic prgnancy case

Ecg5000

Adenxal mass4cm

Live far away from hospital

Bp110/70

40years old wamen prgnant not cometo ant natal care why?

Ignorance

She treat her self

Cost of the consltation

Typical case of Gulien bar syndrome ‫ﻋﺎﻭﺯ ﺍﻟﺘﺴﺨﻴﺺ‬

Surgery ‫ ﺍﺳﺌﻠﻪ ﻛﺘﻴﺮ ﺟﺪﺍ‬cholangitis

Cholecystecomy ‫ﺑﺲ ﻣﺶ ﻓﺎﻛﺮﻩ ﺻﻴﻐﺔ ﺍﻻﺳﺌﻠﻪ‬

‫ﻃﻔﻞ ﺟﻴﻪ ﺍﻟﻄﻮﺍﺭﺉ ﻣﺤﺘﺎﺝ‬intubation ‫ﺍﻻﻡ ﻭﺍﻻﺏ ﺭﻓﻀﻮ ﻻﻧﻪ ﺍﺧﻮﻩ ﻣﺎﺕ ﻣﻦ ﻧﻔﺲ ﺍﻟﻘﺼﻪ‬

‫ﻣﺎﺟﺎﺑﺶ ﻑ ﺍﻻﺧﺘﻴﺎﺭﺍﺕ‬respect their wish

Woman come afer2weeks after delivery by c.s with vaginal bleeding what is the cause

Retained parts of placenta

Endometritis

Utrine atony

Child on anti eplipetic therapy

Stop live vaccine

Stop all vaccines

Replace opv witn ipv

‫ ﺍﻟﻘﺎﻋﺪﺓ ﻑ ﺍﻟﻄﻮﺍﺭﺉ‬،‫ﺑﺎﻟﻔﻌﻞ ﺟﻪ ﺳﺆﺍﻝ ﺯﻱ ﺩﻩ ﺗﻘﺮﻳﺒﺎ‬if life threating. Condition‫ﻻﺗﻌﺘﺪ ﺑﺮﺃﻱ ﺍﻻﺑﻮﻳﻦ‬

‫ﻓﻌﻼ ﺷﺨﺺ ﻛﺎﻥ ﻋﻨﺪﻩ‬fever ‫ ﻭﻛﺎﻥ ﺍﻗﺮﺏ ﺗﺸﻐﻴﺺ‬cholangitis‫ﻭﺑﻴﺴﺎﻝ ﻋﻦ ﺍﻟﻌﻼﺝ‬


‫ﺻﺢ ﻛﺎﻥ ﺑﻴﺴﺄﻝ ﻋﻦ ﺍﻝ‬prognosis

A.complete recovery

B.some persistent weakness

C.parasthesia

D.complete paralysis

‫ﻃﻔﻞ ﻋﻨﺪﻩ ﺍﺯﻣﺔ ﻭﺍﻻﻡ ﺑﺘﻘﻮﻝ ﺍﻥ ﺍﻻﻋﺮﺍﺽ ﺑﺘﺎﺟﻴﻠﻪ ﻣﺮﺗﻴﻦ ﺗﻼﺗﺔ ﺍﺳﺒﻮﻋﻴﺎ ﺑﺎﻟﻨﻬﺎﺭ ﻭﺑﺎﻟﻠﻴﻞ ﻣﺶ ﺑﺘﺎﺟﻲ‬first step in
management

A.Saba

B.oral.steroid

C.inhaler steroid

D.laba

Breast mass female pt diagnosed as cystosarcomatous phylloids

Ur management

A.mastectomy

B.excision ( with safety margin ‫ﺍﻋﺘﻘﺪ ﻛﺎﻧﺖ ﻛﺪﻩ‬

C.....

D.....

‫ﻛﺎﻥ ﻓﻴﻪ ﺳﺆﺍﻝ ﻭﺍﺣﺪﺓ ﻋﻨﺪﻫﺎ ﺻﺮﻉ‬epilesy‫ﻭﻧﺎﻭﻳﺔ ﺗﺤﻤﻞ ﻗﺮﻳﺐ ﺍﻱ ﺍﻟﻌﻼﺡ ﺍﻟﻠﻲ ﺗﺎﺧﺪﻩ ﻓﻲ ﻓﺘﺮﺓ ﻣﺎﻗﺒﻞ ﺍﻟﺤﻤﻞ‬

A.folate

Gastro

1/peptic ulcer

2/GERD

3/celic

4/Ibd

5/pancaritis

6/Gastritis
‫‪duodenal ulcer‬ﺟﻪ ﺳﺆﺍﻝ ﻋﻦ‬

‫‪best way for follow up recovery‬ﺑﻴﻘﻮﻝ ﺍﻝ‬

‫‪A.endoscopy‬‬

‫‪B.urea breath test using radioactive techtenium 99‬‬

‫‪C.clinical‬‬

‫‪D.......‬‬

‫‪ .....///...////....‬ﻟﻤﺪﺓ ﺷﻬﺮ ﻭﺑﻌﺪﻳﻦ ﺭﺟﻌﺘﻠﻪ ﺍﻻﻋﺮﺍﺽ ﺗﺎﻧﻲ‪ PPI‬ﻭﺧﺪ ‪gerd‬ﺳﺆﺍﻝ ﻋﻦ ﺣﺪ ﻣﻌﺮﻭﻑ ﺍﻧﻪ‬

‫‪A...‬‬

‫‪B....‬‬

‫‪C.24 hours pH monitor‬‬

‫‪D......‬‬

‫ﺟﻪ ﺳﺆﺍﻝ ﻏﺮﻳﺐ‬

‫ﺳﻨﺔ ﻭﺩﺍﻳﻤﺎ ﺑﺘﻌﺎﻧﻲ ﻣﻦ ﺍﻟﻢ ﻓﻲ ﺍﻟﺒﻄﻦ ﺗﺤﺖ ﺍﻟﺴﺮﺓ ‪،‬ﺍﻻﻟﻢ ﺑﻴﻌﻤﻠﻬﺎ ﺻﺪﺍﻉ ﺩﺍﻳﻤﺎ ‪،‬ﻟﻬﺎ ﺍﺧﺘﻴﻦ ﻋﻨﺪﻫﻢ ‪13‬ﻃﻔﻠﻪ ﺍﻋﺘﻘﺪ ﻏﻤﺮﻫﺎ‬
‫‪migraine‬‬

‫‪Ur DX‬‬

‫‪A.abdominal migraine‬‬

‫‪B....‬‬

‫‪C....‬‬

‫‪D....‬‬

‫ﺟﻪ ﺳﺆﺍﻝ‬

‫ﻭﺍﺣﺪ ﺍﻋﺘﻘﺪ ﻓﻲ ﺍﻻﺭﺑﻌﻴﻨﺎﺕ ﻣﺶ ﻓﺎﻛﺮ ﺷﻜﻮﺗﻪ ﻛﺎﻧﺖ ﺍﻳﻪ‬

‫‪coloscopy traveled multiple polyps at .....‬ﻋﻤﻠﻮﻟﻪ ﻣﻨﻄﺎﺭ‬

‫‪Our DX‬‬

‫‪A...‬‬

‫ﺍﻋﺘﻘﺪ ﻛﺪﺓ ‪B.familial.....polyoosis‬‬

‫ﺍﻋﺘﻘﺪ ‪C.cancer‬‬
D.....

Pregnant woman have vaginal itching ,whitish vag discharge

Ur ttt

A.flagyl

B.fluconazole ‫ﻭﺍﺣﺪ ﻣﻦ ﺍﻟﻌﻴﻠﺔ ﺩﻱ‬

C...

D.....

Vaccine c.I in child with (white)egg allergy

A.influenza

B.yellow fever

C....

D....

‫ﻭﺍﺣﺪﺓ ﺍﻋﺘﻘﺪ ﻓﻲ ﺍﻟﺘﻼﺗﻴﻨﺎﺕ ﻣﻦ ﻋﻤﺮﺓ ﺟﻪ ﺍﻟﻄﻮﺍﺭﺉ ﺏ‬convulsion

‫ﺍﺧﺪ ﺩﺍﻳﺎﺯﻳﺒﺎﻡ ﻭﻣﺴﺘﺠﺒﺶ ﺍﻟﺨﻄﻮﺓ ﺍﻟﺘﺎﻟﻴﺔ‬

A.oral phenytoin

B.I.v phenytoin

C...

D....

‫ﻃﻔﻞ ﺟﻪ ﺍﻟﻄﻮﺍﺭﺉ ﺍﻻﻡ ﻗﺎﻟﺖ ﺍﻧﻪ ﺍﺗﺸﻨﺞ ﻑ ﺍﻟﺒﻴﺖ ﻓﺤﺼﺘﻪ‬T39 fever

Ur priority in management

A)o2

B.paracetamol

C .anticonvulsant

D.....

Surgical infection and antibiotics


Preop assessement

Ansthesia

Wound care

Post op complication

‫ﻭﺍﺣﺪﺓ ﻧﺎﻭﻳﺔ ﺗﺤﻤﻞ ﻗﺮﻳﺐ ﺍﻳﻪ‬vaccine should be given in preconception

A.rubella

B.varicella

C.....

D.....

‫ﻭﻛﺎﻥ ﻣﺰﻭﺩ ﺣﺎﺟﺎﺕ ﺗﺨﺼﺺ ﺍﻟﺮﻭﺑﻴﻼ‬، ‫ﻃﻔﻞ ﻋﻨﺪﻩ ﺍﻋﺮﺍﺽ ﻭﻋﻼﻣﺎﺕ ﺍﻟﺤﺼﺒﺔ‬

Ur DX

Rubella

‫ﻟﺤﺴﻦ ﺍﻟﺤﻆ ﻣﻜﻨﺶ ﺟﺎﻳﺐ‬measles

‫ﻭﺍﺣﺪ ﺷﺎﻝ ﺍﻟﺰﺍﻳﺪﺓ ﻣﻦ‬3‫ﺍﻳﺎﻡ ﺍﻋﺘﻘﺪ ﻭﺟﻲ ﺑﺎﻟﻢ ﻓﻲ ﻣﻜﺎﻥ ﺍﻟﺠﺮﺡ‬

‫ ﺑﻴﻄﻠﻊ ﺻﺪﻳﺪ‬،‫ﻓﺤﺼﺘﻪ ﻟﻘﻴﺖ ﺍﻟﺠﺮﺡ ﻣﺤﻤﺮ‬

Our next step

I.v abx

Wound drainage

C....

D......

‫ﺍﻋﺘﻘﺪ ﻛﺪﺓ‬

‫ﻭﺍﺣﺪ ﻋﻨﺪﻩ ﻣﻦ ﻛﺬﺍ ﺳﺎﻋﺔ‬fresh anal blood

Nasogastric tube revealed greenish secretion

Our next step

A.barium meal

B.coloscopy
C...

D....

Colonoscopy should be done in any other with lower GI bleeding

Full term pregnancy in labor CX 4 cm three hours ago

Ut contractions 2 in 10 minutes,ctg reactive

No cpd

Our management

A.oxytocin

B.wait and see

C.c.s

D....

Pregnant woman has cardiac ms full term ctg reactive fetus .....indication for forceps use

A.fetal distress

B.cardiac disease

C...

D....

Pg 24 wks in labor CX 6 cm Rom

50% effacement

Ur next step

.I.m dexthamesasone

B.antibiotic

C...

D..

‫ﺍﺧﺘﺮﺕ ﺣﺎﺟﺔ ﺗﺎﻟﺘﺔ‬


‫ﻛﺎﻥ ﻓﻴﻪ ﺳﺆﺍﻝ ﻋﻦ ﻛﻴﻒ ﻳﻨﺘﻘﻞ ﺍﻝ‬T.b

A.air borne

B.droblet

C....

D....

‫ﻛﻨﺖ ﺍﻋﺮﻑ ﺍﻥ ﺍﻳﻪ ﻭ‬b ‫ ﻧﻔﺲ ﺍﻟﻤﻌﻨﻲ ﻟﻜﻦ ﺍﺧﺘﺮﺕ‬b ‫ﻋﻠﻲ ﺍﺳﺎﺱ ﺍﻧﻬﺎ ﺍﻗﺮﺏ ﻟﻠﻮﺍﻗﻊ‬

HBV

HBsag negative, anti surface+anti core positive ur DX

A.immunity of vaccination

B immunity due to past infection

C.acute infection

D.carrier
Hcv lactating

Has cracked nipple

Ci of lactation

A.HCv

B.her nipple

C...

D..

‫ ﺍﻋﺘﻘﺪ ﻛﺎﻥ ﻓﻴﻪ ﺳﺆﺍﻝ ﻋﻦ‬EBv ‫ﻃﻔﻞ ﻋﻨﺪﻩ ﺍﻋﺮﺍﺽ ﺍﻟﻠﻮﺯ‬

‫ﻭﻛﺎﻧﺖ ﺍﻻﺟﺎﺑﺔ‬

A...

B...

C case deteriorate if amoxicellin given

‫ ﻭﺍﺣﺪﺓ ﻓﻲ ﺍﻟﺘﻼﺗﻴﻨﻴﺎﺕ ﻋﻨﺪﻫﺎ‬non pitting odema ‫ ﻓﻲ ﺭﺟﻠﻴﻬﺎ ﺍﻻﺗﻨﻴﻦ‬our DX

A.bilateral DVT

B.lymphangitis

C..

D....

‫ ﻭﺍﺣﺪ ﻓﻲ ﺍﻟﺨﻤﺴﻴﻨﺎﺕ ﻣﺶ ﻓﺎﻛﺮ ﺍﻟﺴﻴﻨﺎﺭﻳﻮ ﺑﺲ ﺍﻋﺘﻘﺪ ﻛﺎﻥ‬atherosclerotic

‫ ﻋﻨﺪﻩ ﺭﺟﻠﻪ ﺍﻟﺸﻤﺎﻝ ﺑﺎﺭﺩﺓ‬pallor, ‫ﺍﻟﺸﻌﺮ ﻃﺎﻳﺮ ﻣﻨﻬﺎ‬

Ur DX

A.arterial embolus

B.arterial thrombosis

C.DVT

D... .

‫ ﻭﺍﺣﺪﺓ ﻓﻲ ﺳﻦ ﺍﻟﺤﻤﻞ ﻛﺎﻥ ﻋﻨﺪﻫﺎ ﺍﻋﺮﺍﺽ‬DVT ‫ ﻗﻲ ﺭﺟﻠﻬﺎ ﺍﻟﻴﻤﻴﻦ ﻭﺍﻟﺸﻤﺎﻝ ﻛﻮﻳﺴﺔ‬ur investigation
Venogeaphy

Plethro.....

Ct angiograthy

Duplex

‫ ﻛﺎﻥ‬confusing for me

‫ ﻭﺍﺣﺪﺓ‬diabetic ulcer on lateral maleolus ‫ﺍﻋﺘﻘﺪ ﻣﻦ ﺷﻬﻮﺭ ﻣﺶ ﺭﺍﺿﻴﺔ ﺗﻠﺘﺌﻢ‬

A.antibiotic

B.chemotherapy

C.surgical depriment

D. . ..

Most common site for rodent ulcer

A.foot

B.arm

C.face

D.shoulder

Female has painful ex.genital ulcer

A.syphilis

B.genital herpes

C.chondyloma lata

D...
‫ ﻛﺎﻥ ﻓﻴﻪ ﺳﺆﺍﻝ ﻣﺮﻳﺾ‬HF ‫ ﻋﻨﺪﻩ‬dry cough

‫ﺍﺗﻜﺮﺭ ﺗﻼﺕ ﻣﺮﺍﺕ‬

1 cause of dry cough

2 to ttt dry cough

3 electrolyte error

‫ ﺍﻻﺟﺎﺑﺔ‬acei

Hyperkalemia

‫ ﻛﺎﻥ ﻓﻴﻪ ﺳﺆﺍﻝ ﻃﻔﻞ‬nehrotic ‫ﻭﻛﺎﻥ ﺑﻴﺴﺄﻝ ﻋﻦ ﺍﻟﺘﻐﺬﻳﺔ ﺍﻋﺘﻘﺪ ﻋﻦ ﺍﻟﺒﺮﻭﺗﻴﻦ‬

A ‫ﺯﻱ ﺍﺑﻄﻔﻞ ﺍﻟﻄﺒﻴﻌﻲ‬

B‫ﺍﻗﻞ‬

C‫ﺍﻛﺘﺮ‬

‫ ﺳﺆﺍﻝ ﻭﺍﺣﺪ ﻋﻨﺪﻩ‬anal pain

Discharge in under wear

By inspection insignificant

DER tender mass )above sphincter ‫ﺍﻋﺘﻘﺪ ﻛﺪﺓ‬

DX

A.perianal fistula

B.inter sphincteric abscess

C...

D....
‫بسم هللا الرحمن الرحيم‬

1- 9 Years old boy brought to clinic with his parents complaining that he is short. He appears healthy,
less than 50% percentile for height on growth chart. His profile revealed progressive increase in
height, his labs was normal, best indicator of his future height is?
A. His weight vs height
B.parental height
C.parental weight
D. Can't be expected

2-56 yes postmenopausal her pap smear hx was normal except one month ago revealed ASCUs , then
given local estrogen for one month then pap smear repeated revealed intraepithelial lesion,Next step is

A.conization
B.hysterectomy
C.colposcopy
-Any high risk lesion on pap smear should be followed by colposcopy -

3-Pap smear screening is done every

A.2 years
B.3 years
C.5 years
-Based on the age and HPV testing -

4-64 years post menopausal bleeding since 5 days HB 11 Pt ,PTT normal,tvus revealed normal except
endometrial thickness 13mm
Best management

A.progesterone
B.endometrial biopsy
C.hysterectomy
D.reassure
• Any thickening more than 5mm , you should do biopsy -

5- 29 years old male did sleeve gastrectomy 6 days ago, came to the ER complaining of persistent
vomiting since operation, on exam nothing significant, Abg normal ( there were numbers ) , US
normal.
Best management?

A.laparoscopy
B.Admissions
C.laparotomy
D.reassure

6- child admitted in the ER, due to DKA severely dehydrated, managed with fluid and insulin , bad
management complication can occur ?

A.hyperkalemia
B.hypoglycemia
C.brain edema
D.hyperglycemia

7- 43 year old she has 3 children the first child was by C -section the other 2 were vaginal birth, now
she is pregnant (36 weeks) comes with Rupture of membrane , by us placenta previa, AFI 4 cm,CX is
2 cm 50% effaced
What is the Absolute CI for ECV ?

A.previous CS
B.her age
C.us result

8- 31 year old pregnant ( 37 weeks ) ROM in labor on exam breech presentation , CTG reactive PV
CX 2 cm , 50% effaced , mid anterior Cx
Management ?

A.CS
B oxytocin
C.vag prostaglandin
D. IV abx
-Breech presentation is one of the indication of cs-

9 - baby (2 months old ) was diagnosed with congenital heart disease came to the ER with bluish
discoloration , the mother said he was crying what you will do ?

A.immediate heart surgery


B.reexamine after baby calmed
C.go home
D.refer to another doctor

10- 1 month year old baby has yellowish discoloration( body and eye ) he was diagnosed with
physiological jaundice. Total bilirubin 25.
What is the dx?
A. Rh incomputable
B.ABO incompatible
C.c.najjar
D.gilbert
-Indirect hight -
11- 15 yrs female not menstruating yet at clinic by her parents ,she is short stature , short neck ,
hypertensive
Most appropriate diagnosis?
A.Turner
B.down
C.TFS
D.normal variant
12. 3 years old baby with single palmar crease - long tongue - straight hair -
What is the dx ?

A.turner
B.down
C.marfan
D. patue

13. Child with high TSH ,low T4 what is the next step ?

A.start thyroxine
B.Abd Us
C.brain MRI
D.barin CT

14. Someone with HF on lasix, BB, captopril he has chronic dry cough
What is the cause ?

A.BB
B.captopril
C.lasix
D.drug irrelevant

15. 12 weeks pregnant has gallbladder stone since 5 years on medication and now she is having pain
when to do cholecystectomy ?

A.now
B.2nd trimester
C.3rd trimester
D. after delivery
16. 21 year old male complains of pain around umbilicus then traveled to rt iliac quadrant tenderness
,rebound tenderness,positive rovsing sign Vomiting, constipation
What is the DX?

A.polyposis
B.appendicitis
C.intestinal obstruction
D.intussusception

17.baby elevate his head when pronated, turn his head left and right coos his age ?

A.3 months
B.4 months
C.5
D. 6

18. 39 weeks pregnant her BP - 150/90 , urine analysis normal , no headache , no vision changes,
no epigastric pain , her previous BP was below 110/70
What is dx?

A.preeclampsia
B.eclampsia
C.gestational hypertension
D. preeclampsia superimposed HTN
-No proteinuria so C-
19. Pregnant women has hard lump at epigastrium , FHS is heard at umbilicus
Fetal presentation is

A.face
B.Brow
C. Breech
D.OP

20. Someone has gallbladder stones came complaining of yellowish discoloration and epigastric pain
, his labs ALT, AT , ALP , bilirubin , amylase are high.
What are next step ?

A.US
B.CT
C.ERCP

21. Pregnant women not complaining of anything , her urine analysis shows nitrate, leucocytes, then
culture revealed 50.000 organisms what is the dx?

A.cystitis
B.pyelonephritis
C. asymptomatic bacteriuria
-She must be treated-

22. male has fever with chills , low back pain , his urine +leukocytes +nitrates.
how to manage ?

A.oral antibiotic
B.iv abx at home
C.admission

23. divorced women has a baby with sickle cell anemia , she want to get married again but scared of
having baby with SCA , what she can do ?

A.karyotype for the husband


B.karyotype for herself
C.HB electrophoresis for her
D. HB electrophoresis for husband

24. Pregnant women during vaginal delivery , what can make her has fourth degree perineal tear ?
A. unrestrained legs and squatting position
B.unrestrained legs and sitting on chair
C.restrained legs and use of forceps and other metallic instrument

25. Child less than 1 year he has vomiting and diarrhea, he is oriented aware , dry tongue , sunken eye
, normal pulse . what is the dehydration degree ?

A.less than 1 %
B.5-9%
C. more than 10 %

26.42 weeks + 3 days pregnant women bishop score 8 CTG reactive , No CPD how to mange ?

A.expectant
B.CS
C.iol
D. decide after one week
• Bioshop above 4 = go for induction
• If she was 39 or 40 weeks the answer will be A-
27.child has headache and can't concentrate on Study , Lap Hb= 7.9 MCV less than normal ,
TIBC high , how to manage ?

A.oral iron
B.oral folate
C.orla vit B 12
D.packed RBCs transfusion

28.diabetic pt on insulin has extremities paresthesia and forget most of the time , Hb 10 ,
MCV high, how to manage ?
A.oral iron
B.oral folate
C.oral vit B 12
D.packed RBCs transfusion

29. child with polyuria and polydipsia OGTT normal , Na + Cl = high


Other electrolytes are normal, what is your dx ?
A.DM
B.DI
C.siadh
D.conns- High Na and Low K + metabolic alkalosis +HTN

30.CTG ( 3 contraction without deceleration ) what is your dx ?


A.head compression
B.cord compression
C.uteruplacental insufficiency
D. none of the above

31. 34 weeks pregnant doses not feel her baby movement, she goes to the doctor and did
CTG was reactive, what is your management?
A.urgent CS
B.Bpp
C.refer for admission
D.Reassure

32.9 year old child , her mother notice that he is having a mass on his abdomen , she goes to
the doctor and did US which shows a mass, what is the dx?
A.wilms

33. 7 year old sick baby, not complinet to his medication your job as a doctor is to ?
A. Explain to the parents the importance of the medication compliance
B. call the police
C. involve the parents and the child in the medical plan
D.refer them to the Ethics Committee
34. ‫مريضة جات العيادة البسة لبس شفاف واثناء الكشف قربت من الدكتور وهمست بكالم بلغة ال تفهمها الممرضة‬
‫ماذا ينبغي ع الطبيب فعله‬، ‫الموجوده مع إيحاءت غير الئقه‬
1-‫يصرخ عليها‬
2-‫يغير لممرضه أخرى تتكلم نفس اللغه‬
3-‫يبلغ آمن المستشفى‬

35. women got pregnant after trying 10 years, now she is 10 week pregnant, she has abdomen
pain and vaginal bleed, she went to the ER , the dr said the was not fetal heart sound and this
is abortion ( they were shocked ) what the doctor should do ?
A.Sympathy
B.Admission after evacuation as post partum care
C.Search for cause of abortion

36. 75 years old has terminal lung cancer , he can’t breathe , he can’t make decision very
well, the doctor want to do debulking of cancer to help him breathing well, and the success of
this operation will improve his live , but the dr also knows that can develop the complication
of the operation and the anesthesia.
What the dr should do?
A.go ahead surgery
B.informed consent pt
C.search alternative decision maker consent

37.Case Scenario for pt with Mitral Valve prolapse, the question is WOTF is true regarding the
surgery ?
Answered: Don’t give the Ab before surgery

38.Case Scenario for child present with Cyanosis for one hour with crying , Physical Examination
reveals murmur in left sternal border , X-ray shows increased pulmonary vascular marking and small
hearts, what is the next step in treatment?
A- give Abs
B- Cardiac Catheterization
C.calm the baby first

39.Case Scenario for 40 years old female with history of travel/ trauma presented with swelling in one
leg , non-pitting edema , skin color is blue, pulse is intact, How to manage ?
A- lymph pain
B- massage and stocking
C- Anticoagulant

40. man with HTN Lab investigation normal but CT scan shows small mass - - - lipid change???
A.adrenalectomy
B.MRI adrenal
C.observation
-this is chusing by adrenal hyperplasia which cause HTN and dyslipidemia –

41. Patient with necrotizing pancreatitis drain done and patient improved what is the metabolic effect
A.insulin resistance
B.hypoglycaemia
C.inhibit gluconeogenesis
D.lipidolysis

42. screening test for diabetes


A.90% sensitive
B.80% specific
C.20% DM , 80% not
D.90 was....

43. 7days newborn girl born with ambiguous genitalia with hypotension on investigation: low Na,
how to treat ?
A- Saline + glucose
B- Hydrocortisone with saline
-CAH-

44.Case Scenario For man present with mass in the abdomen, US show calcified lesion, your Dx ?
A- Carcinoid
B- lymphoma

45.Case Scenario for child with nephrotic syndrome treated by steroid, what is the first thing to
improve?
A- edema
B- protein in urine
C- urine strip for pt

46.Case Scenario for Child with DM type 1 present in E.R with symptoms and signs of DKA, what is
the next step ?
A- provide management plan
B- wait family and discuss
C- concern child for DM type 1
D- discuss with child only

47.Pregnant women in 38 weeks during her C.S surgeon noted bleeding came from the upper
abdomen, what the source of bleeding ?
A- liver hemangioma
B-mesenteric aneurysm
C- aortic aneurysm
D-perforated peptic ulcer

48.Child with peri pleural and central cyanosis , Dx ?


A- TOF
49.Man with blunt trauma , what site of aorta is injured ?
A- aortic arch
B- distal part of aorta in subclavian site
C- proximal part of aorta

50.Man in ER with pain in the right upper abdomen with vomiting ultrasound shows thick wall of
gallbladder and fluid accumulation around management
A - lap cholecystectomy after 12 weeks
B - percutaneous drain
C - open cholecystectomy

51.Appendicitis without pus management


A- lap appendectomy after 12 weeks
B- open appendectomy after 12 weeks
C - immediate appendectomy

52.Symptom confirm diagnosis of eosinophilic esophagitis


A- dysphagia
B- stucking of food in esophagus
C - chest pain centrally located not respond to antacid
D - confirmation by biopsy and endoscopy

** MI types of ECG changes


** 2 nodules in thyroid management

53.Most important prognostic and preventing factor post MI :


A- Decrease lipid
B - stop smoking
C- decrease weight
D - exercise

54.Patient with fever right upper abdominal pain +ve serology of echinococcus ( hydatid
disease) **picture management
A- albendazole
B- cyst. ???
C- Laparoscopic cystectomy
D - percutaneous drainage

55. pt. -> appendectomy


Most common infection post op.
A- fragilis bacterial
B- staph
C- shigella
D- pseudomonas
- The organisms most frequently isolated from the wound after appendectomy are Bacteroides
(especially B. fragilis)

56.protin +2, ketonuria, glucose high, pregnant 37 wks + 8 days,


management ?
A- induction of labor
B- CS

57. treatment of chlymedia


A- azithromycin
B- doxy cyclin
- Azithromycin cuz it takes 1g once daily rather than doxycycline who taken twice daily

58. child frequent eye blinking, normal all


A- Tic disorder
B- eye dryness
- Oral cuz there is no cutaneous involvements

59. 6 months baby have oral candida


A- oral anti fungal
B- systemic anti fungal
C- oral and systemic anti fungal

60.pregnant 2nd trimester -> BHCG, indicate what?


A- normal pregnancy
B- complete …?
C- Down

61._ ‫ وحده شربت لبن وبعده‬fever, abdominal pain, low back pain
Brucella, ttt 6 weeks ??
- triads of brucellosis is fever which increase afternoon, back pain , sweating,
The recommended treatment for brucellosis is given combination of rifampin and doxycycline for 6-8
weeks

62. Hepatitis c needle stick , how much the risk to get infection?
A- 0.03 %
B- 3 %
C- 30 %

63. pt will do thyroidectomy suffer from MR give?


A- amoxicillin for endocarditis
B- cefazoline for wound infection
C- amoxicillin & cefazoline together

64. pt with constipation for 2 days suffer from sever anal pain ex at 6 & 12 o’clock
A- not laceration
B- internal sphincter abscess
C- anal fissure
D- fistula

65. post menopausal flushing cause


A- high FSH
B- Vasodilation
C- Vasoconstriction

66. You was talking to patient to get details on the history but the patient interrupted u and start to talk
about his point of veiw regarding his diagnosis and the reason of his disease . How u would act :
A. Refer him to another doctor
B. Let him finish his point of veiw
c. Apologize to him and start with close ended questions

67. hormonal replacement therapy ?


We should know if she have uterus or post hysterectomy
- You must know does she have uterus or already removed to assess her for endometrial cancer
risk

68. how to diagnose recovery from preeclampsia


A- BP
B- protinurea 3 test
C- protinurea 5 test

69. hypertensive lady controlled in pregnancy may complicated by


A- abnormal fetal presentation
B- abnormal placenta
C- preterm

70. 40 yr parathyroid adenoma


‫ متى اضطر أعمل‬-> parathyroidectomy
A- age > 50
B- osteoporosis
71. _ ‫مريض بيعمل صبغة‬
precaution?
A-Diuretic (before )
B-Nacl (After )

73. swelling med line under thyroid bone, motile


thyroglossal cyst

74.Neck mass ; biopsied showing (thyroid folicullar cells)


Answer : ectopic thyroid

75. thyroid mass cytology -> medullary thyroid carcinoma, rt lobe ‫سليم‬
A- total
B- subtotal
C-radiotherapy
D-chemotherapy
- total thyroidectomy to avoid the recurrence in other lobe

76. lesion in upper thigh ( irregular margin )


A- excision biopsy
B- punch biopsy

77. ‫ بيقولك غالبا هنا‬old pt had bilateral shoulder pain now bilateral hip pain
polymyalgia rheumatica
- it is a chronic inflammatory condition affect the elderly, pt will suffer from sever pain and
stiffness in the shoulder and hip but may affect all the body

78. Celiac disease biopsy done , showing villous atrophy ,


What else could you finding?
B- subtle crypt lengthening
C- high lymphocyte

79.laparoscopic cholecystectomy come with discharge from middle


mangement ?
A- daily dressing
B- wound inspect
C- exploration

80.sickle cel anemia, splenomegaly, hypersplenism


CBC ‫قليل حاجة كل‬
A- exchange transfusion
B- hydroxy urea

81.meningitis -> LP
G+ve, b B hemolytic +ve, coagulase +ve
ttt?
A- Ampicillin
B- cefapime
82. tension pneumothorax, best ttt ?
A- thoracocnthesis
B- tube thoracotomy

83.pulmonary edema
CHF with poor prognosis
best ttt with diuretic

84.Parkinson disease Dx ->


A- Clinical diagnosis
B- CT
- Parkinson disease is a clinical diagnosis. No laboratory biomarkers exist for the condition,
and findings on routine magnetic resonance imaging (MRI) and computed tomography (CT)
scan are unremarkable. Positron emission tomography (PET) and single-photon emission CT
(SPECT) may show findings consistent with Parkinson disease, and olfactory testing may
provide evidence pointing toward Parkinson disease, but these studies are not routinely
needed. (Olfactory testing can reveal hyposmia, which may precede the motor signs of
Parkinson disease by several years. Medscape

85.cystic fibrosis or celiac


‫ التعامل كيفية يعرفوا عشان الدكتور مع اجتمعوا‬+ prognosis
‫الجلسة اسمها ايش؟‬
A- summarisation
B- verbal communication

86.- pic of fracture -> debridement and surgery

87._ pic of HSP


Ttt ? Suppurative treatment
- Immunoglobulin associated with it is IgA, Most complication is nephropathy
H ( hematuria)
S ( skin purpura)
P ( pain i abdomen ,pain in joint

88. pic of vulva and papule ->


A- reassurance
B- More investigation
- We should investigate for STDs

89. 3 yo child :
A) draws triangle
B) use stairs
90.smoking + pregnancy … -> IUGR

91.pic or discerption -> cauliflower -> condyloma accuminata

92.case of hyperkalemia.. ttt -> Ca gluconate


- Yes initial ttt of hyperkalemia is Ca gluconate to avoid cardiac complication then give the pt
insulin

93.Physical activity in adult


A- 1/2 hour in 5 days
B- 1/3 hour in 6 days

94.hematemesis -> chronic liver disease


Esophageal varices
- Most common of hematmesis is peptic ulcer then esophageal varices

95.hematemesis -> endoscope -> resuscitation


Before endoscope what you’ll give
A- vasopressin

96.hematemesis, no h/o
peptic ulcer
A- Erosive gastritis
B- Mallory weiss syndrome
- No hx of peptic ulcer rule out the erosive gastritis
97.CTG, deceleration
Late -> placenta insufficincy
Early -> head compression
Variable -> cord compression

98.seizure in pregnancy
A- Mg sulfate

99.child have abdominal pain, testes tender, horizontal testis above the other one
A-US
B- scrotal exploration
C- angiography

100.ttt of testicular torsion ?


Surgery

101.egg allergy vaccine -> yellow fever


1. 102. Which vaccine cause GBS ? Polio

2. 103.vaccine contraindicated in pregnancy ? MMR, Varicella

104.child with low immunity, which vaccine we shouldn’t give ?


Varicella

105. ‫ عنده وصار طويلة فترة من ستيرويد ياخذ طفل‬cushinoid app


‫؟ ايه نعمل تطعيمة نعطيه وعايزين‬
A- no problem to give him vaccine
B- give immunoglobulins
C- give him vaccine after 1 month

106. .case of shock ( cardiogenic )


2 different quistion
One with increase capillary refill? septic shock
One with fever ? bacteremia sepsis

107. Doctor asked the resident to give antibiotic to the patient but resident think that the patient had
sensitivity to it. What to do as a resident?
A-Ask doctor to sign the order. √
B-Give the patient the antibiotic.
C-Neglect the doctor order.

108. Family think vaccine harm son. What to do?


A-Counseling and Explain to family the advantages and importance of vaccines. √

109. A patient will undergo elective surgery. Pt don’t want to know the surgery details but
anesthesiologist insist to tell her. What to do?
A- Tell pt anesthesiologist decision. √
B- Reschedule surgery until see anther anesthesiologist
N.B: U have to tell pt why u delay surgery, can't choose B without explain to pt.
110. 15 years old male come to ER and LP should be done to r/o meningitis. What is true regarding
consent?
A- Take Consent from parents √
B- No consent it is an ER case
C- Consent from patient
D- Consent parents and ancent pt .

111. Pt female insisted to take HRT and you told her it is not fit for her. What to do?
A-Refuse to give her HRT. √

112. Couples ask for surrogacy :


A-Tell them it might be illegal in KSA. √

113. Increase domestic violence in ?


A- pregnancy √
N.B: Pregnant women are at high risk of domestic violence in all cultural groups.

114. When to start education about breast feeding ?


A-2nd trimester
B-3rd trimester
C-after delivery
D-before pregnancy √

115. Asthmatic patient not controlled on SABA. What to do next ?


A- Add ICS. √
N.B: BA management started by SABA, then add ICS, then LABA if not controlled.
116. WOTF is a sign of acute severe asthma?
A-RR > 25. √
B-HR > 100.
C-O2 saturation < 95%.
D-Peak expiratory flow < 300.

117. Pt known COPD , come to ER drowsy , o2 sat 84 % , ABGS shows I think respiratory acidosis.
TTT?
A-o2 by mask
B-mechanical ventilation (Not sure)
C-Nubelizer

118. Type of ABG abnormality in excessive vomiting ?


A-Metabolic alkalosis with hypokalemia √

119. Pt known case of PUD diagnosed 8 years ago come with vomiting 2 weeks ago. On exam
dehydrated and abdominal Splash test +ve. What expected on ABG?
A-metabolic acidosis
B-Metabolic alkalosis. √
C-compensated
N.B: This is about Gastric outlet syndrome due to pyloric hypertrophy obstruction complicated a long
standing PUD. Suction Splash test is positive in gastric outlet obstruction.
120. Umbilical hernia in a 10 months old baby. Management?
A- observation √
N.B: Most of UH don't need any ttt. Usually the hole heals on its own by time your child is 4 or 5
years old. If not so do the surgery before the child enter the school.

121. 2 days old baby with seizure. LP done and reported normal. Dx?
A-neonatal sepsis
B-asphyxia √
N.B: Hypoxic ischemic encephalopathy ''birth asphyxia'' is the most common cause of neonatal
seizures, with LP r/o meningitis so most likely it is birth asphyxia.

122. Child with pneumonia (in Right middle lobe) with x ray, vital signs stable. Management?
A-admission for iv abx
B- prescribe amoxicillin for 7 days and discharge

123. Pt had retained placenta and has bleeding what type of PPH ?
A-primary √
B- secondary

124. Pt in labor, dilatation 5cm since 2 hour , effacement 100 , station 0. Management?
A-observe √
B-give oxytocin

125. Pt in labor, dilatation 5cm since 2 hour , effacement 100, station 0. Management?
A-observe √
B-give oxytocin

126. Case of PPH. What to give?


A-oxytocin. (First drug to be given). √
B-ergot
C-carboprost

127. Pt want to stop smoking, he has HTN and epilepsy. what is contraindicated?
A-bupropion √
N.B: U might be asked about S/E of bupropion? Headache.

128. Pt female has history of recurrent PPH. What to do ?


A-active management of 3rd stage of labor. √

129. Pt had a previous C-S. Now pregnant at 36 weeks with abdominal pain. Vitals: hypotensive,
tachycardia. Dx?
A-uterine rupture
B-abruptio placenta. √
N.B: Hypotension + tachycardia are signs of abruption of the placenta.

130. Yellowish greenish vaginal discharge increase with intercourse Dx?


A-trichomonas vaginitis. √
B-bacterial vaginosis.

131. Pt had menopause at age of 40. She is at increased risk of?


A- increase risk for osteoporosis. √

132. Anal itching and pain after defecation ?


A- anal fistula
B- anal fissure. √
N.B: features of anal fissure are: visible tear around anus, skin tag or lump near tear, sharp anal pain
related to defecation, streaks of blood on stools, burning or itching in anal area.

133. Newborn with hypoglycemia. What to give?


A-2ml/kg of 10% dextrose. √
B-2ml/kg of 12.5% dextrose
C-4ml/kg of 10% dextrose
D-4 ml/kg 12.5% dextrose
134. Newborn of diabetic mother. From where take sample?
A- central venosuline. √
B- peripheral
C- nasogastric
D- orogastric

135. 20 years old male with meningitis and seizure. TTT?


A- Vancomycin and Ceftriaxone.
B- vancomycin + ceftriaxone + steroid. √
136. 50 years old male with meningitis, most common pathogen?
A-Streptococcus pneumoniae. √
B-listeria.
N.B: Listeria in 60 and above.
137. 22 weeks gestation , os dilated 1.5 cm. Dx?
A-cervical incompetence. √
138. 1st trimester , Os closed , mild spotting. Type of abortion?
Threatened abortion. √

139. 1st trimester, Os closed and speculum revealed heavy bleeding. Type of abortion?
Threatened abortion. √

140. Ectopic pregnancy treated with salpingostomy, what next ?


A-no follow up needed
B-follow up Bhcg after a week and till zero. √

141. Pt has MI 6 weeks ago come for elective laparoscopic cholecystectomy. What to do?
A-do it now
B-delay surgery until 6 months from MI. √
142. Child with jaundice and anemia ,blood film , coombs direct and indirect are positive,
reticulocytes high. Dx?
A-autoimmune hemolytic anemia. √
B-spherocytosis

143. Inguinal hernia reducable, TTT?


A-simple repair
B-Elective open hernioplasty with mesh repair. √
C-laprascopic
D-observe
N.B: Best for hernia is open surgery except: female for cosmetic reason + bilateral + obese so here it
is better for lap surgery.

144. what’s hemosiderin disposition in macrophages in the lung is indicative of?


A-CMV
B-chronic lung infection
C. Heart failure. √
D-pneumocystis jiroveci

145. which group of drugs is contraindicated in diabetic patients with heart failure?
A-biguanides
B- dipeptidyl peptides IV inhibitors
C- GLP-1 analogous
D- SGLT-2 inhibitors
E- Thiazolidinediones. √

146. Management of late deceleration?


A-change position. √

147. Hemorrhoidectomy is a treatment for?


A-internal piles.
B-external lateral hemorrhoids. √
C-fissure.

148-Child with viral diarrhea ?


A-supportive rx √

149-Child with abd pain postprandial, distention , diarrhea , ?


A-carb intolerance
B-celiac √
C-IBD
D-CYSTIC FIB

150-Child with abd pain , distention , diarrhea , failure to thrive , buttock muscle wasting ?
A-celeic √
B-Ibd

151-Pt has abd pain , father has hx of h pylori , her urea breath test +ve , dx ?
A-h pylori infection √

152-Asthmatic child mother said cannot be sure of proper technique , what the cause of uncontrolled
asthma?Bneed step up ?
A-improper technique √
153-Dermatology in celiac disease ?
A- dermatitis herpetiformis √

154-Female with hypotention , hypok+ , hyponNa , dark line in hand scar what next :
A-dexa supprestion test
B-cortisol level√
C-syntchan test

155-Turner syndrome ass with ?


A-DM
B-hypothyroidism √
C-hyperprolactinemia ?

156-Pt 660 years ask for cardiac assessment : ASCVD RISK 6.5 % WHAT ELSE ?
A-highly sensitive CRP
B-stess echo

157-Pt has multiple multiple liver lacertion what to do ?


A-prehepatic packing
B-rt hepatectomy
C-vessels ligation

158- Pt had rt appeindeceal abcess what pathophysiology ?


A-vasoconstriction
B-redistribution of blood vessels

159-Pt had chest trauma nad 3⁄4 rib fracture , paradoxical chest movement ?
A-chest tube
B-needle aspiration

160-Post partum has dvt started on enoxaparine then developed PE :?


A-same
B-give thrombolytic
**PE treated by anticoagulant Thrombolysis in PE has indications in unstable patient with
hypotension

161- meningitis prophylaxis ?


A-oral rifampicin .
depends on age (child oral rifampicin ,If adult Ciprofloxacin )

162- vaccine can pregnant take ?


A-influenza

163 – pt had low immunity vaccine not to give ?


A-varicella

164 – about x linked agammaglobulinemia


Difiencny in Cd19 and CD 20
Defect in tyrosine kinase that prevent the development of mature B cells from pre B cells in bone
marrow (B lymphocytes= total lymphocytes- T lymphocytes) Predisposed to recurrent infections with
encapsulated organisms like H. influenza, S. pneumoniae due to
impaired humoral immunity

165-Pap smear ?
A-3 years continue

166-Pt 40 years old what to tell about pap smear ?


A-5 years
B-3 consecutive neg results no need for further

167-Question about hemorrhagic shock


168-Question cardiogenic shock

💯 Best vassopressors :

Septic shock : nor epinephrine


Anaphylactic shock : epinephrine
Neurogenic shock : dopamine
Cardiogrnic shock : dopmamine
Hypovolumic shock : epinephrine
1.scenario pt with resting tremors mask like faces ?
A-Parkinson’s disease
B-tardive dyskinesia
C-hypothyroidism

169- a scenario of pt describing rash on palms and soles and syphyllis features causative organism ?
A-treponema pallium

170- charcoats triad ?


A-acute cholangitis
B-acute cholecystitis
C-acute appendicitis

171- duodenal perforation treatment ?


A-simple closure with omental patch “Graham omental patch”

172- anal fissure treatment 2 mcqs related to this ?


A- Lateral internal sphinterectomy
B-external sphincterectomy

173- aggressive treatment of Sle ?


A-prednisolone and mycophenolate mofetil
B-prednisone and hydroxychloroquine
C-prednisone and cyclophosphamide

174- Another scenario of Sle pt on hydroxychloroquine developed some additional symptoms wat to
do ?
A-add prednisone
B-increase the dose of hydroxychloroquine

175- a pt with history of Ocp use wat is associated with it?


A anemia
B history of fatigue
C history of DVT

176- third and fourth degree perineal tears due to ?


A unrestrained legs and squatting position
B unrestrained legs and sitting on chair
C restrained legs and use of forceps and other metallic instruments

177- ECV contraindicated in ?


A-AFI less than 22
B-Cord length greater than 36 or 38mm
C-Placenta Previa

178- a scenario of pt with 11 months history of fatigue developed jaundice 1 to 2 wks ago with raised
lfts ?
A HCC
B AUTOIMMUNE HEPATITIS

179- a pic of ultrasound showing chocolate cyst (endometrioma) ?


A associated with increased risk of infertility

180- a pic of child with crusting on chin ?


A impetigo
B cellulitis
C erysipelas

181- multiple draining sinuses ?


A carbuncle

182- a scenario of cardiac temponade muffled heart sound distended neck veins on ecg specific
finding
A- diffuse st elevation
B- pr depression
- The answer is low voltage or sinus alternans

183- ecg showing heart block


A- first degree
B- 2nd
C- 3rd
D- 4th

184- a pt rescued from a burning building ....he is conscious with singed nasal hairs
A- intubation
B- respiratory support
186- a pic of urine dipstick (coloured) showing +3 proteinuria ???
The answer should be (300-1000 mg/dl)

188- saag less than 1.1g/dL Cause


A-peritoneal tb
B liver cirrhosis

189- contraindication of liver transplant


A acute hepatitis
B chronic
C liver cirrhosis
D end stage liver disease
190- milestone of 5 months old child
A sits without support
B coos
C reaches for object
D pincer grasp

191- baby with absent red eye reflex otherwise healthy


A reassurance of pt that it’s normal
B consult ophthalmologist
- Uni : retinobalstoma
Bi: congenital

192- baby after feeding spills the milk examination and labs normal
A reassurance of mother and position during feeding

193- playtime for the kids


A 30 min
B 60 min
C 120 mins

194- exercise for Diabetic obese pts (I guess )


A 150 min per week
B 180 min/ek

195- a 10 kg child with severe dehydration calculate maintenance & deficit


A 46L
B 54
C 76
D 90
- If the question only asked about Maintenance >> the answer is 1000 ml/day or 41.6 ml/h
- If the question only asked about Deficit >> the answer *could be the same as above *coz
severe dehydration means weight loss more than 10% >> the equation is percentage of body
weight loss multiply by the total weight multiply by 10 >> 10*10*10= 1000 ml/day or 41.6
ml/h
- If the question asked about maintenance and Deficit >>the answer is 1000 + 1000 = 2000
ml/day or 83.3 ml/h . Or more

196- a pt with biliary duct stone all values normal except alk phosphatase quite raised
A ERCP
B MRCP
C CT abdomen
- if the Question contain stones either in gall bladder or CBD , so the next step you do is ERCP
, cuz pt are diagnosed with stone So no need for the MRCP
- if The Question not contain stones but with abnormal Lab result only such as Increase ALk,
next step is MRCP to confirm the Diagnosis
198- a child with about 3 yrs hx of steatorrhea abdominal bloating and diarrhea amylase was normal
what to do?
A stool for fat
B ultrasound
C anti endomysial antibodies

199- predisposes to cancer


A barret esophagus (Precancerous lesion)

201- causative organism for acute brochiolitis


A RSV

202- a case of GB syndrome with features showing ascending paralysis and hx of URTI

203- treatment of GB syndrome


A plasmapheresis
B IV immunoglobulins

204- a pt with all cell lines depressed


A aplastic anemia
B hemolytic
C Sca
D IDA

205- a male pt with normal T cell count but B cells almost diminished
A bruton agammaglobulinemia

206- a baby with abdominal distension and palpable bladder


A prune belly syndrome

207- a 60 yr old pt fights and argues with every 1 sth related to his cousin was also mentioned which I
hv forgotten
A Alzheimer’s dementia
B vascular dementia
C Huntington’s disease
D frontotemporal dementia
208- a scenario showing features of celiac disease
A celiac disease

209- in psoriasis
A clinical hx and examination is adequate to reach the diagnosis
B skin biopsy is confirmatory

210- fixed splitting of S2 bicuspid ventricular hyperteophy


A AVSD
B ASD
C VSd

211- treatment of RHF in which pulmonary edema was developing


A IV frusemide

212- /a pt after carpel tunnel release presented with pain what to give for pain relief
A triptans
B opioids
C NSAIDs

213- H pylori pt treated with clarithro amoxicillin and omeprazole for 10 days not improved what to
do?
A same regimen for 14 days again
B switch to doxycycline metronidazole and PPI.
- Look for other options
10 days ok need to switch but not doxy. Switch to levofloxacin amoxcillin omperazole regime

214- a pt with hx of H. Pylori infection along with maltoma ( 2 queries related to this)
A eradicate H.pylori first
B treat maltoma first

215- baby having unformed stools


A ORS
B lactose free diet
C formula feeding
216- a 60 yr old lady with features of osteoarthritis and varus force was mentioned in the scenario but
there were no options of medial or lateral meniscal tears

217- beta hcg positive but on ultrasound no gestational sac seen


A extrauterine pregnancy.

218- gestational diabetes is


A normal pregnancy associated with diabetogenic condition due to
A progesterone
B same as above due to estrogen
C due to FSH

219- a pt with short neck short stature both parents short


A turner
B familial

220- treatment of hyperkalemia( 2 queries related to this)


A ca gluconate
B insulin

221- a young boy with newly diagnosed diabetes controls his sugar on diet with raised bsl during day
( sth like that was in the scenario)
A brittle diabetes
B honeymoon period
C dawn phenomenon
D somogyi phenomenon B insulin

222- a mother knows that her child has diabetes and was giving him insulin then she suddenly stops
giving him insulin by saying his pancreas r working and he has no disease anymore
A anxiety
B neglect
C denial

223- a couple refuses for vaccination of their child as they think vaccination is bad for their child
A explain the benefits of vaccination to him
B don’t vaccinate the child
C if couple doesn’t agree forcefully vaccinate the child

225- SVD with 800ml blood loss visible to eye


A prim PPH
B SEC PPH

226- best presentation for NVD in case of twins


A breech cephalic
B cephalic breech
C cephalic cephalic
D breech breech
- The best is : cephalic cephalic , also its the most common in twins pregnancy

227- a female pt. wearing see thru dress talks to a male doc in seductive way wat shud the doc do?
A behave rudely and say get out of my room
B refer to other doc
C call the security
D call the nurse use the same language

228- a pt developed fever after 17 days of chemo


A send CBC blood culture urine culture and give antipyretics
B send CBC blood culture urine culture and start broad spectrum antibiotics

229- pt working in some factory develops symptoms of asthma wat to do ist?


A CXR
B PFTS
C ABGS
D CBC

230- a young child newly diagnosed asthmatic mother also asthmatic father is a chain smoker
A stop smoking in the house
B give short acting beta agonist to the child

231- pfts in COPD or Asthma🙄


A increased TLC decreased Fvc and decreased fev1/fvc
B decreased TLC INCREASED FVC AND DECREASED FEV1 to Fvc
C increased tlc normal Fvc and decreased fev1/Fvc

232- ca breast tumor marker


A BRCA
B CA 125
C Cea
D p53
233- scenario of complete molar pregnancy

--
MAY 2019 question

1-Baby delivered at home presented 65 days later with rt thigh bruises other exam unremarkable ( PT
high , PTT high , other normal ) what is the dx?

A. hemorrhagic diseases of newborn


B. factor x deficiency

2-Baby delivered at home with umbilical 1-bleeding what the cause of this bleeding?
A-factor Vlll
B- factor X
C- factor XIII

Note : if baby born in home and comes with bleeding its due to ( vit.K ) deficiency
Note: Vit k depended factors (2,7,9,10)
Note : bleeding from umbilical after clamping after birth due to ( factor XIII ) deficiency
Note : bleeding after circumcision due to ( factor VIII ) deficiency : Hemophilia A

3-Surgeon diagnosed a pt with appendicitis then he decided to do


surgical appendectomy. Intraoperatively he found appendix normal.
Then doctor removed the appendix. What is the most appropriate?
action with regards to this decision?
A- Tell the patient that Dr found appendix normal and removed it
according to guidelines
B- Don't tell patient
C- Call another Surgeon to tell him
D- Inform the ethical communication in Hospital

4- Surgeon diagnosed a patient as a case of appendicitis, he decided to


do appendectomy, when he opened the patient, he didn't find the
appendix. How to identify appendix?
A- Follow tenia coli
B- Follow right colonic artery

-The appendix is located at the proximal root of the outer tinea coli of the cecum. Because the anterior
tinea coli of the cecum is generally quite prominent, it serves as a guide to locate the appendix.-

5-A Surgeon decided to do resection anastomosis surgery for colon


cancer. During the operation the Surgeon noticed the surgical field
became red (bleeding) and BP dropped. Surgeon called the vascular
Surgeon to interfere. What is the injured artery in such case?

- Depend on the location of the cancer –


6- A Surgeon decided to do resection anastomosis surgery for colon
cancer. He called the vascular Surgeon to attend cuz he was excpect to
have a bleeding during operation. During the operation the Surgeon
noticed the surgical field became red (bleeding) and BP dropped.
Surgeon and vascular Surgeon tried to control the bleeding and they
did packing. Then after finishing the anastomosis, BP dropped again.
What is the next step in management?
A- Remove Packing and suture injured vessel
B-Clamp aorta proximally
C- Clamp aorta distally
D-Stabilize patient on table

7- Case scenario of female patient gets menopaused (cycles stopped).


Then she developed dysfunctional uterine bleeding for the last 3
months. TVU found endometriosis (Jnadi think this is endometrial
thickness not endometriosis) with thickness about 19 mm. What to offer
for this woman?
A- Ablation of endometriosis
B- Total abdominal hysterectomy- best management -
C- Laparoscopic hysterectomy

-If ask next step, do Endometrial biopsy-

8-Case scenario of female patient menopausal now 58 years old. She


developed dysfunctional uterine bleeding for the last 2 months. TVU
found multiple endometriosis. What to offer for this women?
A- Ablation of endometriosis
B- Total abdominal hysterectomy- if old pt-
C- Laparoscopic hysterectomy

9-Female patient diagnosed as a case of ovarian cancer. What is the


most important tumor marker for follow up?
A- C125
B- AFP

10-Question directly related to Saudi vaccination guidelines, child aged


1 years, asked about what vaccine in this age?
-MMR , PCV, MCV4,OPV-

11- Many Qs about Ulcerative colitis.


12- Many Qs about Crohn disease.
13- Many Qs about biliary diseases.

14- Pt was self dependent in eating, drinking and wearing clothes. Then
he developed dementia and some abnormal behaviors and agitations.
How to manage?
A.Haloperidol
- Frontotemporal dementia-

15- Q about erythema nodusum and pyoderma gangrenosum of CD and


UC, with their pictures.

16- Child aged 3 years old brought by his mother with episodes of
crying, fever, productive cough and drooling of saliva. 1-2 weeks ago
mother reported that her child was complaining of bilateral
conjunctivitis. Which of the following is the most causative organism?
A- Mycoplasma pneumonia
B- Adenovirus
C-Chlamydia trichomonas pneumonitis

- Viral infection from age 3 month to 5 years is the most common organism-

17- 36 week pregnant with history of smoking came complaining of vaginal bleeding with uterine
tenderness.Diagnosis?
A- Abruptio placenta
B- Placenta previa
C- Vasa previa
D- Placenta accrete

- VB + tenderness + smoking history-

18- the daughter of an old pt complain of progressively decreasing in memory and change in
personality pf her father, ttt?

A.refer to the geriatric clinic

19- Contraindications of ECV?


Placenta previa

20- Young married female came to hospital complaining that her


husband bite her. What do you think the cause of this problem?
A- Husband culture
B-Husband socioeconomic state
C- Husband education
- Eikenella corrodens , gram negative anaerobe – human bite
21-
••Measure cortisol level!!!
••CT brain!!!
••Low dose dexamethasone!!!
••24h!!!
If Q about cushing?

22- Pregnant women at 15 weeks presented with nausea and


headache. On exam the BP was high "168/100", otherwise healthy. Dx?
A- Preeclampsia
B- Gestational HTN
C- Pregnancy induced HTN
D-chronic HTN

23- Pregnant women at 22 weeks presented with nausea and


headache. On exam the BP was high "168/100", otherwise healthy
without protein in urine. Dx?
A- Preeclampsia
B-Gestational HTN
C-Pregnancy induced HTN

24- pregnant women at 22 weeks presented with nausea and headache. On exam the BP was high
"168/100", otherwise healthy without protein in urine. TTT?
1-Hydralazine
2-Methyl dopa
3-Nigedipine
4-Nitrates
25- Most common site of colon cancer metastasis?
A- Liver
B- Lung
C- Prostate
D- Stomach

26- Surgeon did laparoscopy procedure in the liver for a certain reason. Then 10 days following
procedure the patient presented with GERD like sensation, air or gases coming out of the mouth
,What could be the injured structure due laparoscopy
A- Trachea
B- Esophagus
C-Stomach
D-Duodenum

27- Pt has thigh swelling, diagnosed as sarcoma. How to know metastasis?


A- MRI
B- CT
C- CT angiogram
D- Bone scan

28-Child can sit in tripod position, can move from back to abdomen and from abdomen to back, can
hold things in hand but can't control thing in his hand between 2 fingers "no pincer grip".
Developmental milestones age?
A- 4 months
B- 6 months
C- 9 months
D- 12 months

29- Healthy 35 years old male doing routine investigations then accidentally solitary nodule found on
CXR. What to do?
A-Follow up after 6 months!!!
B-CT
C- Take biopsy
D- Resection

30- The most important precaution in patient with Pulmonary TB is?


A-Airborne infection
B-Droplets infection

31- Neoborn delivered at hospital, healthy, but mother reported that the previous baby died due to
immunodeficiency. What is the best action regarding Saudi immunization program?
A-Don't give BCG
B- Give BCG with close observation
C- Refer baby
- we cannot give baby BCG vaccine if there is history of immunodeficiency-
32- A child has immunodeficiency. Which vaccine is contraindicated?
A- Varicella
B- Pneumococcus
C- Influenza
D- Injectable polio- IPV is inactive-

33- Pregnant women, her child is in school, she is afraid of flaring, what next?
Take influenza

34-Mother came to hospital with her healthy normal weight 6 weeks old child, she said that with each
feeding the child passing informed stool. What to do?
A- Change Milk formula
B- Give rehydration solution

35- Child 1 years old on normal formula but passing bloodless diarrhea, vomiting and abdominal pain
since. What to do?
Change formula to cow free milk

36- Child 2 months, cry 1-2 days per week, not sleep well for 1 month. Normal weight. Otherwise
healthy. Dx?
Infantile colic

37-Male patient with left leg pain and swelling, edematous, non pitting edema, intact pulse. Most
benefit test to confirm diagnosis.
A- CT angio
B- MRI venography
C-Duplex US

38- Elderly male came to ER with SOB and difficulty sleeping at night. He mentioned Hx of heart
surgery few years ago. You did investigations and ECHO. What is the most important thing you will
be afraid of that might kill patient?
A-Pulmonary HTN
B- Stroke
C-PE
D- Cardiogenic manifestations

39- Elderly male came with weight loss, headache, RUQ tenderness, LFT all high. Dx?
HCC

40- A case of acute pancreatitis in which after 10 days from acute pancreatitis the pt came with
epigastric vague pain and fullness. Dx?
Pancreatic pseudocyst

41- Pulsating epigastric mass. Dx?


AAA

42- AAA. Best to confirm?


A- CT – with contrast -
B- US – screening -
C- MRI
D- X-ray
43- Bilateral reducible uncomplicated inguinal hernias. Pt not complaining of any complications.
What to do?
A- Lap mesh repair
B- Open mesh repair

- Pantaloon hernia-

44- Pt with Normal sodium, low potassium, and has HTN. Dx?
A-Pheochromocytoma
B- Hyperaldosteronism

45- Pt known case of DM and HTN. He has uncontrolled HTN on multiple medications. What is the
cause of uncontrollable HTN?
A-Taking food salts
B- Hyponatremia

46- Child diagnosed as a case of nephrotic syndrome. Anyway, now the only complaint is generalized
edema. Management?
Frusemide

47- A Case scenario of K/C DM pt came complained of fatigue, lab shows low bicarbonate, normal
Na and low k. What to give?
A-Give bucarb infusion
B- Give IV insulin with dextrose
C- Give normal fluid

48- A case scenario of patient K/C of non hodgkin lymphoma came with
feathers of tumor lysis syndrome. What electrolytes abnormalities you
will find?
A-HypoCa!!!
B- HyperCa!!!

49- A 12 years old girl who is a known case of DM 1 diagnosed 2 years


ago. Now came to do routine checkup. She has no features of puberty.
What test to do annually?
A- Ophthalmology!!!
B- GH assay!!!
50- A young female complaining of abdominal pain that increased with
cycles, stay for 5 days, first 2 days of cycle she missed school due
severe pain (dysmenorrhea). First next step in TTT?
NSAID is the best for primary dysmenorrhea

51- Women with PCOS, not planning to get pregnant soon. TTT?
••Metformin!!!
••COCP!!!
••Progesterone only pills!!!

52-years old female morbidly obese want to do surgical weight loss


operation. What investigation you should do before surgery?
••Abdominal US!!!
••CT!!!
••MRI!!
••C-ray!!!

53- pt with all the viral hepatitis markers negative except positive HB
surface antibodies. Dx?
A-Previous immunization
B- Acute resolved infection

54-year old child not given any vaccine before because mother
refused due fear of harm to her child. What to do?
Counseling-
- Talk with parents about benefits and side effects of vaccines-

55-What is the most common organism to cause Post prosthetic valve


(following valve replacement) endocarditis?
A-Staph epidermedis!!!
B-Staph aureus!!!
C-Strept viridans!!!

56-Case scenario of patient complaining of SOB, with Severe MR + mild LVH TTT?
A-Mitral valve replacement!!!
B- Give medication!!!

57-Case scenario of patient with syncopy and SOB. Dx?


••Aortic stenosis!!!

58-When to do surgery for child with congenital atrial septal defect?


••depends on the size!!!
••depend on the site!!!

59- 12-mother brought her child to hospital. Surgeon discovered hernia


and decided to do surgery immediately. Mother refused. What to do?
A-Do surgery whatever mother decision!!!
B-Inform hospital police!!!
C- Don't do surgery!!!
D- explain to the mother
-If she refused let her sign Discharge Against Medical Advice (DAMA)-

60-Rheumatoid arthritis patient with multiple joints pain, given NSAID


but not responding, best drug?
••Methotrexate!!!
61-Case of child with skin rash in lower limbs, ankle and knee joints
pain, fever. Dx?
••HSP vasculitis!! Supportive ttx

62-Case of abdominal pain, bloody diarrhea, Hb low. Dx?


••HUS!!!

63-Low Hb, MCV low, RDW high. Dx?


A-IDA!!!
B-Aplastic anemia!!!
C-Thalassemia!!!

- If Hx of methotrexate > folate


If pt on phenytoin > folate

64-Pt with recurrent chest infections and low platelets. Family history
of similar condition. Dx?
••Weskot Aldrich Syndrome “X linked thrombocytopenia”
- X linked recessive-

65- You are in OR with consultant and he ask you to prepare Lt. kidney
for surgery but you are sure it is the Rt. Kidney. What to do?
A-Inform chief intern!!!
B-Run outside the OR and check the CT image!!!
C-Inform the consultant about your concern!!!

66-Case of male patient with joint and abdominal pain, splenomegaly,


not responding to NSAID. Dx?
Felty syndrome
- triad of RA , splenomegaly neutropenia-

67-What is the contraindication of liver transplantation?


••End stage liver disease with ascites!!!
••End stage liver disease with encephalopathy!!!
••Acute liver hepatitis !!!
••Alcoholic liver cirrhosis!!!

68- Contraindicated vaccine in pregnancy?


••HPV!!!
••Zoster!!!
••Pneumococcal!!!
••Influenza!!!
SMLE
GENERAL SURGERY
• Fibroadenoma → most common < 30 y.o.
→ firm, rubbery, non-tender, well-circumscribed, mobile,
hormone-dependent.
**Aspiration→ no fluid main difference with Cysts.
• Diagnosis→ U/S q6mo x 2 y. & FNA
→core or excisional biopsy if suspecting malignancy
• Treatment→ Generally Conservative
→ Excision if: * >3 cm
*growing upon serial U/S
*Symptomatic
*formed >35 y.o.
*core biopsy suggesting Phyllodes
• IntraDuctal Papilloma → solitary benign intraductal Polyp
** MOST COMMON CAUSE OF BLOODY NIPPLE DISCHARGE
• Diagnosis → U/S will show Nodule
• Treatment→ Excision of involved duct
• Fat Necrosis → Hx of trauma (50%)
→ Firm, ill-defined mass, ± tenderness
→ Regress spontaneously
→Rule out malignancy
• Mammary Duct Ectasia → Obstruction of subareolar Duct
→ Inflammation & tenderness
→ Resolve spontaneously
→ 2ndry inf. Mastitis & Abscess
• Mastitis → lactational vs. non-lactational
• Breast cancer Screening Guidelines Jan, 2016: US & Canada
**Mammography → 40-49 (weak Recommendation)
→50-74 (Bieannial)
→75+ (weak recommendation)
**BSE→ Against Teaching BSE
However the BSE can be performed monthly but with no clear evidence
of benefit.
**BRCA1 & BRCA2 → high risk patient (+ve family history)
• Invasive Ductal Carcinoma → 80%
• Invasive Lobular Carcinoma → 15%
→ 20% bilateral
• Radical mastectomy (rarely)→ removes all breast tissue, skin,
pectoralis muscle, axillary nodes
• Modified radical mastectomy (MRM) → removes all breast tissue,
skin, and axillary nodes
• Simple mastectomy → removes all breast tissue and skin
5/The most common cause of breast bloody discharge?
• Fibroadenoma
• Ductal Papilloma
• Mammary Duct Ectasia
Answer: B
21/23 years old woman with cyclic bilateral nodularity in her breast since 6 months. On
examination there is tender 3 cm mobile subareolar mass on her right breast. What
will you do next?
• FNA with cytology
• Mammogram
• Follow up for next cycle
• Observation
Answer: A
23/Pectoralis major muscle was removed during mastectomy. Which action will be lost?
• Abduction
• Adduction
• Extension
• Internal rotation or flexion
Answer: B
32/ 24 years old girl came to you with a painless mobile breast mass that does not
change with menstrual cycle. It started increasing since 3 months. Now it’s 3 cm
in size. What is the most likely cause?
• Fibroadenoma
• Ductal Papilloma
• Paget’s disease of nipple
Answer: A
33/ They want to stop screening for breast cancer for women under 47 to decrease
unnecessary anxiety to the public. You didn’t agree because there is a gene that
cause cancer in young women. What is the gene?
• A) APC
• B)BRCA2
• C)CA125
Answer: B
34/ What will tell you tell a young lady in regard to breast cancer?
• a. Self exam is obsolete now
• b. Self exam as well as mammography are important
• c. Only mammography
• d. CT scan
Answer: A
38/ Which of the following suggest ductal papilloma on breast examination?
• A)Blood stain on nipple
• B) Hemorrhagic discharge
• C)Serous discharge
• D)Pus from nipple
Answer: A
40/ What is the best study to determine cystic breast lesion?
• Biopsy
• FNA
• Mammogram
• U/S
Answer: B
42/ A patient presented with hard mass on the outer upper area of the breast. which
lymph node you have to examine?
• posterior axillary
• anterior axillary
• lateral axillary
• medial axillary
Answer: B
44- What is the most common site of breast cancer?
• Subareolar
• Upper Lateral Quadrant
• Upper Medial Quadrant
Answer: B
67/ a lactating woman 10 days after delivery complaining of fever and rigors. On
examination: tender left breast and nodules in upper outer area , What is the
most likely the diagnosis?
• a. Postpartum sepsis.
• b. Breast abscess.
• c. Inflammatory breast cancer.
Answer: B
75/A female patient who has high risk for breast cancer. Which gene is responsible
for that?
• APC gene
• CEA gene
• BRCA 1 &BRCA2
Answer: C
78/ What is the cause of skin dimpling in breast cancer?
• Cooper ligament
• Lactiferous duct
• Peau D’orange
Answer: A
85/30 years old female dancer with breast mass that disappeared by aspiration ?
• Fibroadenoma
• Fibrocystic Change
• Ductal papilloma
• Galactocele
Answer: B
92/ what are the gene mutation most likely leading to breast cancer?
• APC
• BRCA1 & BRCA2
• CA125
• CA19.9
Answer: B
93/30 years pt with cyclic breast pain and tenderness . pain is relieved after the
cycle. By examination there is periareolar nodular mass 3cm .what is you action?
• A.FNA
• B. mammogram followed by US
• C. U/S
• D.follow her to next period
Answer: C
147/pt noticed lump for three months which is freely mobile with no discharge and
not related to menstrul cycle?
• cystic
• fibroadenoma
• ductal papilloma
Answer: B
190- Bilateral breast cancer with :
• 1.paget disease
• 2.lobular carcinoma
• 3.mucinus carcinoma
• 4.ductal carcinoma
Answer: B
219- 48 y.o female, without any family history of Breast CA with Normal
mammogram when to repeat it?
• 2 yrs
• 3 yrs
• 5 yrs
Answer: A
220- Patient with breast lump which is painless, firm , freely mobile. U/S showed a
>3 cm in size. What’s most likely the diagnosis?
• 1. fibroadenoma
• 2.cancer
• 3.fibrocystic changes
Answer: A
236-Case scenario about patient with breast lump in upper outer quadrant which is
tender and increase within days from her period asking about the diagnosis?
• Fibroadenoma
• Ductal papilloma
• lactional mastitis
Answer: A
267-female came for breast screening, her mother and sister had breast cancer at 45
and 48 year. What are you going to for her now ?
• Perform U/S
• Schedule her for a CT scan
• Advise her to start Mammogram screening
• Refer her for genetic studies
Answer: D
281- in mastectomy what structures are to be excised ?
• all breast tissue, skin, pectoralis muscle, axillary nodes
• all breast tissue, skin, and axillary nodes
• all breast tissue and skin
Answer: C
284- self-breast examination decrease breast cancer by years:
• one year.
• two year.
• three year.
• four year.
Answer: B
297- 40 years old female with 3 months mass 2.5 cm, firm and irregular, patient
report no history of trauma. mammogram and US was normal. what to do next?
• A) follow up after two cycles
• b) obtain biopsy
• C) Reassurance
Answer: B
302- Patient came to you her mother and her sister were diagnosed with breast
cancer, came for counseling, what to do now?
• A) BRCA
• B) MRI
• C) PET scan
Answer: A
305- Patient underwent right radical mastectomy. Which of the following will be
affected post operatively?
• Right arm abduction and internal rotation
• Right arm abduction
• Right arm adduction
Answer: C
315-best imaging for cyst in the breast
• US
• Mammogram
• Tc99m
Answer: A
317- years old lady, found a lump in the upper outer area of her right breast, it was
large, firm with irregular border, not tender, no palpable lymph node, what
should you do?
• A) FNA.
• B) lumpectomy and radiation.
• C)mastectomy.
• D) Biopsy
Answer: D
330 - young female noticed a mass 1 week after her menstrual cycle and
• persist for three cycles , the mass is rubbery , mobile what is your diagnosis?
• fibroadenoma
• Fibrocystic change
• Duct ectasia
Answer: A
343- mamogram dectect breast ca before self examination by how many years?
• A. 1 year
• B. 2 years
• C. 3 years
• D. 4 years
Answer: A
369-A patient was found to have a bilateral hypo-echoic cysts; by ultrasound. What
is the next step?
• MRI
• CT
• Biopsy
• FNA
Answer: D
376- a female patient with breast lump in upper outer tender and increase within
days from her period asking about the diagnosis ?
• Fibroadenoma
• Lactational mastitis
• Galactocele
Answer: A
389-papillary tumor non invasive 2cm, next step?
• Duct excision
• Radiation and chemotherapy
• Breast Conservative surgery
Answer: A
407- women with mastitis what the most appropriate action?
• stop breast feeding
• clean nipple with alcohol
• surgical drainage
• continue breast feeding
Answer: D
441-patient after radical mastectomy can't raise her hand above the shoulder?
• due to excision of Pectoralis muscle
• Due to injury to the axillay nerve
• Due to injury the long thoracic nerve
Answer: C
442-upper outer quadrant breast lump , first metastasis to :
• Anterior lymph node
• posterior LN
• Lateral LN
Answer: A
454-Most common cause for a stained-blood nipple is?
• Paget’s disease of nipple
• Mastitis
• Ductal Papilloma
Answer: C
467-adult with breast mass the become large with menses came to you asking for dx
FNA shows yellowish secretion which now disappeared what is your diagnosis?
• 1. Glactocele
• 2. Ductasia
• 3. Normal variant
• 4. Nanovulatory
Answer: C
481-a patient post mastectomy has been scheduled for reconstruction of the pectoralis
muscle from the rectus muscle what is the vessel may be injured or affected during
this operation?
• superior epigastric artery
• inferior epigastric artery
Answer: B
490-how often to do self breast exam ?
• 1 month
• 3 month
• 6 month
Answer: A
514-Best prognostic indicator for breast cancer?
• Estrogen receptor
• Stage and grade
• Involvement of axillary LN
Answer: C
518-Most specific predictor of local recurrence in breast cancer.
• size.
• Lymph Node Metastasis
• estrogen receptors
• progestrone receptors
Answer: B
• Neck Masses
• Congenital→ <20 y.o
→ Lateral: Brachial cyst or venolymphatic malformation
→ Midline: thyroglossal /dermoid cyst or laryngocele
• Infectious/Inf. → <20-40 y.o.
→Lymphadenopathy reactive to Tonsillitis or phatyngitis
→ Infectious MonoNucleosis/kawasaki/cat scratch
→ Salivary gland Calculi/Sialoadenitis
→Thyroiditis
• Neoplastic → >40 y.o.
→Thyroid tumor
→Lymphoma
→ Metastatic Malignancy
• Investigation→ History & Physical Examination
→ Laboratory (WBC → Infection vs lymphoma)
→ Imaging: U/S & CT Scan
→ Biopsy: FNA / Open Biopsy→lymphoma
• Brachial Cyst→ smooth & Painless
→ Appears after URTI
→ Surgical Removal if infected start with antibiotics.
• Thyroglosal Cyst → 20-40 y.o.
→ midline
→ w/ swallowing & tongue Protrusion
→Complete excision (SIS-TRUNK Operation)
• Lymphoma →Hodgkin Lymphoma
→Non-Hodgkin Lymphoma
**NHL(chest&abdomen) is the 2nd most common tumor in HIV patients after Kaposi
Sarcoma (skin&oralmucosa)
• Hodgkin→ bimodal incidence 15-34 & >50 y.o.
→presistent/painless/firm/paraneoplastic Pruritis
→cervical or supra-calvicular (sequential Involvment of LN)
→ both Chemotherapy & Radiotherapy
→ >90% 5 year survival rate
• Staging →
stage I stage II stage III stage IV

• Add (B) to the stage if: Night sweating or High fever or


unexplained weight loss
• Non-Hodgkin→ 7-11 y.o.
→ lymphoblastic/ large cell/ Burkitt's like
→ Rapid growing in Pediatrics
→ Slow growing in Adults
→ most commonly: Abdomen/ chest/ head & neck
→ Chemotherapy no benefit of radiotherapy in Pediatrics
• Staging → Stage 1 : one group of lymph nodes
or one organ (extranodal lymphoma)
Stage 2: In 2 or more groups of lymph nodes
or In 1 or more groups + an organ

Stage 3: both sides of the diaphragm


Stage 4: one or more organs
or distant metastsis

Add (B) to the stage if: Night sweating or High fever or unexplained weight loss
• Burkitt’s Lymphoma → B-cell Lymphoma
→ C-myc gene dysregulation
→ Starry Sky Pattern on Microscope
→ Ass. w/ EBV infection
• Thyroid Nodule → TSH & U/S for initial evaluation
→ FNA → > 5mm with Suspicious U/S
→ >1cm all nodules
→ Radio-Iodine therapy → hyperthyroidism
→ Post-op (follicullar & papillary )
→Chemotherapy & Radiotherapy→ Anaplastic CA or Thyroid Lymphoma
→ Surgical Excision → FNA suspicious
→ >3-4 cm even w/ normal FNA
→ increasing size on serial U/S
→ Hyperthyroidism unmanageable with Radio-iodine therapy
→malignancy other anaplastic CA or thyroid lymphoma
• Thyroid CA→ Papillary→Popular (70-75%)
→Palpable lymph nodes
→ Postive I131
→Postive Prognosis
→ Follicular → 10%
→ Far away metastasis
→Female (3:1)
→ -ve FNA
→Favorable prognosis
→Medullary → 3-5%
→ MEN IIa or MEN IIb (also Papillary)
→aMyloid
• Oral Cavity SCC → Asymptomatic neck mass 30%
→ ± Bleeding
→Dysphagia/ Sialorrhea/ Dysphonia
Q16/What is the commonest nerve injured after thyroidectomy?
• Superior Laryngeal Nerve
• External Laryngeal Nerve
• Recurrent Laryngeal Nerve
Answer: C
Q37/ 45 years old female presented with neck swelling and anxiety. On examination
the swelling is moving with swallowing and lateral to midline. What is the most
likely diagnosis?
• Thyroglossal cyst
• Branchial cyst
• Thyroid nodule
Answer: C
Q69/ (long scenario) man with solid thyroid nodule, what is the most appropriate
thing to do?
• a. Incisional biopsy.
• b. Excisional biopsy.
• c. Fine needle aspiration.
Answer: B
Q86/ which thyroid cancer have the best prognosis?
• Papillary Carcinoma
• Follicular Carcinoma
• Atypical Carcinoma
• Medullary Carcinoma
Answer: A
Q88/ mass in the neck in cervical area, the best initial investigation?
• Chest X-ray
• U/S
• FNA
• CT
Answer: B
Q98/ sign of hyperthyroidism with nodule in lateral to midline?
• Brachial Cyst
• Thyroglossal Cyst
• Thyroid Nodule
• Lymph Node
Answer: C
Q123/Single thyroid nodule iodine sensitive. Best management?
• A.Lobectomy
• B.Radio iodine therapy
• C.Anti thyroid drug
Answer: A
Q161- In thyroidectomy which nerve injury lead to horseness of voice?
• Superior Laryngeal Nerve
• External Laryngeal Nerve
• Recurrent Laryngeal Nerve
Answer: C
Q200- Generalized cervical lymphadenopathy + mild tenderness + low grade fever.
What's the most likely diagnosis:
• Viral Infection
• lymphocytic lymphoma
• hodgkin's lymphoma
Answer: A
Q216- Pt with swelling in the thyroid , She is euthyroid what will you do ?
• Thyroid lobectomy
• FNA
• Biopsy
• Excisional biopsy
Answer: B
Q222- patient presented with cervical mass not in the midline and move with
swallowing, what is diagnosis:
• thyroid nodule
• thyroglossal cyst
• brachial plexus
• cervical lymph nodes
Answer: A
Q223- patient presented with cervical swelling associated with weight loss, night
sweating and fever but no respiratory or neurological symptoms, he was
diagnosed as non hodgkin's lymphoma. On CT scan, there are paraaortic and
inguinal lymph nodes enlargement what is the stage:
• IB
• IIB
• IIIB
• IVB
Answer: C
Q243-thyroid disease associated with papillary cancer ?
• Hashimoto
• Riedel thyroiditis
• De Quervain’s disease
Answer: A
Q257- Parafollicular cell produce which type of tumor ?
• Follicular
• Medullary
• Papillary
Answer: B
Q261- Lump in throut barium enima and endoscopy normal?
• Squamous cancer
• Gerd
• Peptic ulcer
• Globus pharyngeos
Answer: D
Q275-old patient in 60s of age smoker presented with history of sore throat
hoarseness of voices for 3 month with enlarged cervical lymph nodes : on
examination the was large mass in the throat ( the history was highly suggestive
of larynx Ca ) what is the most probably your action :
• saline water gargles
• analgesia and antibiotic
• surgery and radiotherapy
Answer: C
Q278- pt presented with dysphasia , he has hoarseness and excessive salivation and feels
that there is a lump in his throat , what is your diagnosis?
• Achalasia
• Diffuse esophageal spasm
• Cricopharengeal dysfunction
Answer: C
Q300. Pt with swelling in the thyroid , She is euthyroid what will you do ?
• Thyroid lobectomy
• FNA
• Excisional biopsy
Answer: B
Q321- man with hyperthyroidism, presented with thyroid nodule showing high
radioactive uptake, how would you manage him?
• antithyroid medication.
• radioactive iodine.
• thyroidectomy.
Answer: C
Q346- NHL staging (Ann arbor)Patient with B symptoms and lymph nodes from the neck
to the para aortic with spleen involvement?
• A. Stage 1
• B. Stage 2
• C. Stage 3
• D. Stage 4
Answer: D
Q349- Thyroid nodule treatment in the right side only?
• Lobectomy
• Thyroidectomy
• Radioactive Iodine
Answer: A
Q388- pt has high radioactive uptake what is the best treatment :
• Antithyroid
• radioactive iodine
• Thyroidectomy
Answer: A
Q402-Solitary thyroid nodule . what is the definitive diagnostic test?
• FNA
• Excisional biopsy
• Incisional biopsy
Answer: B
Q470-patient with thyroid nodule and tortous dilated artery that compress the
external laryngeal nerve, which of the following will be affected
• tone of the voice
• lateral adduction of vocal cord
• abduction of vocal cord
• sensation above the vocal cord
Answer: A
Q486-mass moving with tongue protrusion?
• Brachial Cyst
• Thyroid Nodule
• Thyroglossal Cyst
Answer: C
Q486-pt with thyroid goiter compress external laryngeal nerve . what is the action
affected:
• tension of vocal cords
• abuduction of vocal cords
• adduction of vocal cords
• Loss of sensation superior to vocal cords
Answer: A
Q520-Old man smoker and alcohol drinker developed oral ulcer that bleeds when
Touching?
• Squamous cell carcinoma
• Aphthous ulcer
Answer: A
Q528-25 y o woman came to the clinic with her 6 weeks old baby, complaining of
irritability , weight loss, and inability to sleep?
• Post Partum thyroditis
• hyperthyroidism
• hashimoto thyroditis
Answer: A
Q530- A specific sign of hyperthyroid ?
• retraction led
• prietabial myxedema
Answer: A
Q289 -most common parotid gland malignancy?
• mucoepedermoid carcinoma
• Adenocarcinoma
• Squamous cell carcinoma
Answer: A
Q322-Pt with starry sky pattern on biopsy > burkhits Wts the mutation:
• C-myc genE
• abl2
• Bcr- abl
Answer: A
Q355- Which of the following is associated with Burkitt’s Lymphoma ?
• EBV
• HIV
• cocxacki
• HBV
Answer: A
172/ Pt with past hx of hodgikon lymphoma , but cured completely, Presented with
back pain . Examination and evaluation show paraspinus Edema and fluid
collection -ve burecella titer and tuberclin test , what the cause ?
• burecellosis
• breast cancer
• recurrent hodgkin lymphoma
Answer: C
Q405- young pt, with neck mass and itching, Diagnosis?
• leukemia
• Hodgkin lymphoma
• Non-hodgkin lymphoma
• Lyme disease
Answer: B
Q459-Pt came to you with history of ingestion of 2 safety pin what is your action?
• Reassurance
• Surgery
• Follow up with x-ray
Answer: C
Q502-HIV patient (symptoms of intestinal obstruction )did intestinal resection, They
found tumor white in color nearly encircling the wall,What is the tumor?
• hodgkin
• non hodgkin
• Adenocarcinoma
• plasmacytoma
Answer: B
• Achalasia → degeneration of the AuerBach’s plexus
→CXR → Dilated Esophagus
→Barium → Bird’s beak
→Balloon Dilation can be repeated
→ Botulinum Toxin temporary
→ Myotomy
• EsophgealVarices → Major bleeding (10-30% of UGIB)
→Ass. w/ liver Cirrhosis (Alcohol gastritis→20%UGIB)
• Peptic Ulcer → Ass. w/ H. Pylori infection (20% OF UGIB)
→ Eradication of H. pylori & antacids
→ Surgery if; perforation/failure of medication

**All casesof Upper GI Bleeding must be investigated with Upper


Endoscopy after patient stabilaziation
Q10/A patient with achalasia. Repeated attempt of balloon expansion relapses again.
What is the management?
• stent insertion
• myotomy
• nasogastric tube
Answer: B
Q191- Case of RLQ pain and mass , what is the defenitiveTreatment?
• conservative .
• surgery .
• Antibiotics
Answer: B
Q6/A mass at the middle of the Esophagus:
• Adenocarcinoma
• Squamous cell Carcinoma
• Barret’s Esophagus
Answer: B
Q415-male patient complaining of dysphagia for solid and liquid ,radiograph shows
dilated esophagus with tapering at end , what is ur dx?
• Esophageal spasm
• Esophageal cancer
• Achalasia
Answer: C
Q476-pt perform surgery to stomach wrapped it around the esophagus which nerve
will be injured?
• Phrenic Nerve
• Vagus Nerve
• Celiac Plexus
Answer: B
Q24- A patient presented complaining of dark stool and vomiting blood. He is stable
now. What is your investigation?
• Barium studies
• Upper GI endoscope.
• Rectal Endoscope
Answer: B
Q14- A 32 years-old alcoholic male patient is brought to the emergency department
with the history of vomiting large amount of bright red blood. Physical
examination revealed splenomegaly and ascites. Which of the following is the
most likely source of bleeding?
• Duodenal ulcer.
• Proton pump.
• Esophageal varices.
• Gastric cancer.
Answer: C
Q50- Patient had Upper GI bleeding. What is the diagnosis?
• Peptic Ulcer
• Gastritis
• Esophageal Varices
• Mallory Weiss Tears
Answer: A
Q71- What is the common presentation of perforated duodenal ulcer?
• Answer: The most characteristic symptom is the suddenness of the onset of
epigastric pain. The pain rapidly becomes generalized although occasionally it
moves to the right lower quadrant.
Q350- Peptic ulcer disease treatment the kid is on PPI and metronidazole What to
add?
• Clarithromycin
• Esomeprazole
• Rabeprazole
• Lansoprazole
Answer: A
Q399-Complain of lump like sensation, No dysphagea, All examination are normal.
barum and endoscopy showed normal anatomy. what is the possible diagnosis?
• Cricopharyngial Dysfunction
• Pharangial dieverticuli
• Globus Pharangyus
Answer: A
Q535-A 40-year-old male, 2 years history of difficulty swallowing and lump sensation
in throat, excessive salivation, intermittent hoarseness, weight loss:
• Achalasia
• Scleroderma
• Diffuse esophageal spasm
• Cricopharyngeal dysfunction
Answer: D
Q50/ Patient had Upper GI bleeding. What is the diagnosis?
• A-Smoking
• B-NSAID’s
• C-ZollingerEllison $
• D-Ethanol excess
• E-Family hx of PUD
Answer: B
Q509-Alcoholic patient, RUQ pain , jaundice , splenomegly , ascites , caput medusa:
• Portal Hypertension
• hepatitis
• budd-chiarri $
Answer: A
Q24/A patient presented complaining of dark stool and vomiting blood. He is stable now.
What is your next step?
• Administer IV large bore
• Blood match for transfusion
• Upper GI endoscope.
• Reassurance and send patient home
Answer: C
Q14/A 32 y.o alcoholic male patient is brought to the emergency department with the
history of vomiting large amount of bright red blood. P/E revealed splenomegaly and
ascites. Which of the following is the most likely source of bleeding?
• a. Duodenal ulcer.
• b. Proton pump.
• c. Esophageal varices.
• d. Gastric cancer.
Answer: C
• Appendicitis → 6% of population (80% 5-35 y.o)
→ low grade fever
→poor localized periumbilical Abdominal pain then
well localized over Mcburney’s point
→ +ve Mcburney’s sign
→+ve Rovsnig’s sing
→Retrocecal appendix → Psoas sign
→Pelvic Appendix → Obtrator sign
→ Subhepatic Appendix → Pain in RUQ due to non-rotation
→Investigation; Laboratory→ mild leukocytosis
;U/S→ non specific to rule out gyne causes
;CXR/AXR→non specific rule out perforation
;CT→ Optimal method inflammatory changes
→Treatment; Appendectomy → Gold standard
**Hydration & pre-op AB (Cefazolin & Metronidazole or 2nd/3rd cephalosporin)
** no post-op AB unless there is Perforation **post-op histo→ Abundant Neutophil
in Muscularis propria **for elderly colonscopy to rule out neoplasm
• Presentation → Retroceccal 64% →psoas sign
→ Pelvic/subcecal 32% →obtrator sign
→rectal/vaginal tenderness
→Preileal/postileal
→Inverted/Subhepatic/Retroperitoneal Cecum
• Small Bowel Obstruction → 75% adhesions post-surgury
→ intussusception/ crohn’s
→vomiting proximal > distal
→ Abdominal Distention distal > proximal
Q547-case scenario of intussusception what is the gold standard investigation :
• A. abdominal CT
• B. CXR- barium study
• C. abdominal X-ray
Answer:B
Q27/ Child was crying since 4 hours ago. He has fever and vomiting. On examination
there was a mass on the RUQ. What is your diagnosis?
• Appendicitis
• Gastroenteritis
• Pancreatitis
• Intussusception
Answer: D
Q188-abdominal radiological investigation showed the inner and outer surface of
intestine , what is mechanism of this condition?
• Perforation
• Obstruction
• Extravasation
Answer: B
Q283-Pt with low intestinal obstruction most common symptom associated with
absolute obstruction
• diarrhea
• Vomiting
• Distention
Answer: C
Q307- Pt with high intestinal obstruction, what will be the symptom ?
• Absolute constipation
• Diarrhea
• Change of habit
Answer: B
Q4/ 16 years old female complaining of abdominal pain for 2 days. It started in the
periumbilical area then it radiated to the right lower quadrant. It is associated with
anorexia. On examination, right lower mass was felt. What is the management?
• a. surgery immediately
• b. conservative
Answer: A
Q51/ A patient presented with RLQ pain, guarding, tenderness and positive obturator
sign. What is the most likely diagnosis?
Q58/ A patient had appendectomy. The artery that supplies the appendix is a branch
from which of the following?
• Superior epigastric Artery
• Inferior epigastric Artery
• Superior Mesenteric Artery
• inferior mesenteric artery
Answer: C
Q70/ Patient have appendectomy what is the most common infection will be come after
operation?
• a. fragilis bacteroides
• b. Staph aureus
• c. Shigella
• d. Pseudomonas
Answer: A
Q72/ In appendectomy. Which artery you have to pay attention to during the procedure?
A) Gonadal vessels
B) Iliac artery
C) Inferior Epigastric
Answer: B
Q74/ 17 year old complaining of abdominal pain that shifts to the right iliac fossa. what is
the most likely diagnosis?
• Diverticulitis
• Testicular Torsion
• Appendicitis
Answer: C
Q75/ 17 years old boy with acute appendicitis, appendectomy was done. If you take it
under microscope, which of the following will be found ?
• a. Neutrophils in muscularis propria.
• b. Mucus filled lumen.
• c. Neoplastic tumor at the tip.
Answer: A
Q102/Post appendectomy, penicillin resistance? What antibiotic use?
• Cefotetan
• Cefoxitin
• Gentamycin
• Carbapenem
Answer: C
Q132/patient presented with appendicitis appendectomy done and sent to
pathology what you expect to see
A.Neutrophil in wall .
B.Distended lumen with mucus
C. Pleomorphism
Answer: A
Q160/ Chronic alcoholic wake up at 3 am with abdominal pain after a little while the
pain was generalized all over the abdomen and is severely exacerbated with
movement:
A. Peptic ulcer perforation
B. Cholycystitis
C. Diverticulitis
D. Pancreatitis
Answer: A
Q224- Patient with appendicitis but presented as right upper quadrant pain?
A. Retrocecal appendix.
B. Subhepatic Appendix
C. Cholycystitis
Answer: B
Q225- pt present with Right subcostal area pain and you do surgery and you find
inflamed appendix?
A. Retrocecal appendix.
B. Subhepatic Appendix
C. Cholycystitis
Answer: B
Q358- Famale married pt came with all symptoms and signs of typical appendicitis(
periumbilical pain but still didn’t shift) + sever pain on rectal exam + low grade
fever?
• Ovarian torsion
• Ectopic pregnancy
• Appendicitis
• Cholycystitis
Answer: C
Q396-patient came with picture of appendicitis, mass in RIF, what is next step?
• Reassurance
• Surgery
• Obtain Biopsy
Answer: B
Q408- pt with RLQ pain and swelling , loss weight , colonoscopy done showed mass
in RLQ , what is your Diagnosis :
• Appendix tumor
• Cecum tumor
• Rectal Tumor
Answer: B
Q420-Sites of Displaced appendix ?
• Retrocecal
• Subcecal
• Pelvic
• Extra Pelvic
Answer: A
Q438- If the surgeon going to do appendectomy ,So he have to ligate the arterial
branch of which the following major artery ?
• Inferior mesenteric
• Superior mesenteric
• Superior epigastric
• Inferior epigastric
Answer: B
Q451- after appendectomy a patient got abdominal infection by enterococcus
faecium. He is allergic to penicillin what are you going to give him:
• ceftriaxone
• vancomycin
• cefotaxim
• TMP-SMX
Answer: B
Q465-RLQ pain 9 out of 10 , mass felt tender on examination , US mass 6cm:
• ectopic pregnancy
• cystic rupture
• appendicitis
• cystic Torsion
Answer: D
Q473-Right lower Quadrant pain , nausea , guarding in examination?
• Cholycystitis
• Divirticulitis
• Appendicitis
Answer: C
Q474-male with right lower hypochondrium pain , in surgical exploration they found
inflamed appendix , what is the congenital malformation responsible for this :
• reverse rotation
• non rotation
• behind liver
Answer: B
Q318-pic of bowel obstruction with baby 2 week what is the treatment ?
• primary surgery
• leveling surgery
• Barium Enema
Answer: C
Q354- Child came with colicky abdominal pain, vomiting, bloody stool. US showed
doughnut sign. What is the most important step in management of this case ?
• urgent surgery referral
• NGT decompression
• IV Fluid resuscitation
• Barium Enema
Answer: D
Q434-What is the most sensitive and specific investigation to diagnose and localize
bowel obstruction ?
• Erect & Supine CXR
• Erect & Supine AXR
• U/S
• CT scan
Answer: D
• Crohn’sDisease → any part of GIT (Small bowel + colon 50%)
→ Post-prandial Colicky pain
→ fever
→ RLQ mass
→ Endoscope (pathcy ulcers/ psuedopolyps/ cobblestone)
→ AXR (String sign; due to bowel wall thickening)
→ EIM (CD > UC); Dermatologic → ErythemaNodosum 15%
→PyodermaGangrenosum10%
→ PerianalSkin Tags 80%
→ Tx; Surgery → not curative/ recurrence after 5 years
indication → Faliureof medical tx
→SBO/ Abcess/ Perforation
complication of tx→shortbowel $/ gall & kidenystones
→Mortality Rate; 15% at 30 y.
• Ulcerative Colitis → large bowel only (rectum w/ proximal progression)
→Rectal Bleeding 90% & Diarrhea (no Abdominal Pain/no fever)
→Urgency/ Tenesmus
→Endoscpy; Continuous Inflammation /Erythema/psuedopolyps
→Radiology; Lack of haustra/ stricture→ruleout CA
→ Complication; toxic mega colon/ EryhthemaNodosum
→Tx; Surgery→Curative no recurrence post-op
→ perforation/toxic mega colon/ obstruction/ failure to thrive
** more liver Problems and higher risk of colorectal cancer is noted with UC, Thus regular colonscopy &
biopsy of ≥ 8 years disease
• Irritable Bowel $ →functional bowel symptoms w/ normal investigation
→Stress; trigger not a cause
→ Types; with Diarrhea/Constipation or mixed (Alternative)
**Abdominal discomfort >12 weeks in the past 12 mo; relieved by defecation;
change in consistency; change in frequency.
Q94/ Ulcerative colitis skin lesions?
• Erythema Nodosum
• Pyoderma Gangriosm
• Perianal Skin Tags
Answer: A
Q165/ man came with loose stools , history of loose stools before. Its watery like
with mucous not containing blood what is your diagnosis?
• IBS
• Crohn’s disease
• ulcerative colitis
Answer: B
Q166/pt came with history for alternative bowel habits some time loose and other
day constipation for weeks your diagnosis?
• IBS
• Crohn’s disease
• Ulcerative colitis
Answer: A
Q215- 16 years old female. Fever and Chronic diarrhea for 10 months Post meal Para
umbilical pain?
• IBS
• Crohn’s disease
• Ulcerative colitis
Answer: B
Q264- Hx of ulcerative colitis when will do colonoscopy?
• After 1 year
• Every 6 months
• 8y after diagnosis
Answer: C
Q272-main difference between chron's , ulcerative and IBS?

Q314- female pt, k/c of crohn’s disease, presented to the ER with constipation and
vomiting " what is the cause of the symptoms?
• Perforation
• Obstruction
• Short gut syndrome
Q189- If there was inferior mesenteric artery thrombosis. Which artery will not be
affected!
• descending colon Artery
• sigmoidal Artery
• Splenic Artery
• Cecal Artery
• Meckel’s Diverticulum → 2%of population of which 2% symptomatic
→2:1 male: female ratio
→ 2 feet from Illeo-cecalvalve
→2inches length
→2inches diameter
→2types of tissue (gastric/pancreatic)
→2years of age
investigation→AXR
→ Tc99m
• Hirschsprung’s Disease →1 in 5000 births/ 3 or 4:1 m: f
→ migration defect of neurocrestcells (aganglionic)
→ failure to pass meconium/Distal bowel Obst./ Abdominal Dist.
Rectal Biopsy (gold standard) → aganglionosis& Neuronal Hypertrophy in Muscularisexterna
AXR → Narrow Rectum
Tx → Surgical resection of aganglionicpart (anal pull through)
• Hemorrhoids → engorged vascular cushion at 3, 7, 11 o’clock (lithotomypostision)
→Painless bleeding/ anemia/mucus /pruritis/ rectal fullness
→4 grades
Tx→hemorrhoidectomy
• External Hemorrhoids→ sudden painful perianalmass
→Dietary & Medical (surgical if thrombosed)
• Colorectal CA → 4th most common cancer, 2nd most common cause of cancer death
→Age (above 50, mean is70 )
→ Genetic; microsatellite instability (FAP/HNPCC/family hx)
→ Colonic conditions (AdenomatousPolyposis/ IBD/ hxof Colorectal CA)
Q28/ Treatment of hirschprung disease?
• Barium Enema
• Balloon Dilation
• Surgical Rescection
Answer: C
Q43/ What is the most common site of Meckel’s diverticulum?
• Duedenojejunal junction
• Proximal to Illeocecal Valve
• Distal To illeocecal Valve
Answer: B
Q77/ Child was diagnosed to have meckel's diverticulae. What is the best diagnostic test?
• Abdominal X-ray
• Braium Studies
• Tc99m
• CT scan
Answer: C
Q26/Which one of these layers is involved in chagas and hirschsprung disease?
• a. muscularis externa
• b. mucosa
• c. submucosa
Answer: A
Q124/Perianal itching. Most likely?
• Crohn’s disease
• Ulcerative Colitis
• Hemorrhoids
Answer: C
Q231-baby brought by his mother with abdominal distention, bad smell vomiting,
history of delayed passage of muconim after 3 days using enema rectal
examination showed empty rectum (photo of abdominal x ray showing multiple
air-fluid levels). and the doctors start to decompress his stomach. what is the
definitive treatment ?
• Leveling Colostomy
• total Coloectomy
• Partial Coloectomy
Answer: C
Q7/Which of these supplements protect against colorectal cancer?
• folic acid
• vit E
• Vit D
Answer: C
Q168/which of the following has strong association for colorectal cancer:
• familial polyposis
• ulcerative colitis
• chrons disease
Answer: A
Q259- "Microsatellite instability?
• Crohn’s Disease
• Ulcerative Colitis
• Colorectal Carcinoma
Answer: C
• Q287-high fat and low fiber diet increases the risk of which of the following cancers?
• Esophageal Carcinoma
• Gastric Carcinoma
• Colorectal Carcinoma
Answer: C
• Q99/gastrectomy which of the following enzymes will be defecient?
• Lipase
• Pepsin
• Trypsin
Answer: B
• Q194- patient did gastrectomy and need to take vit B12 for life . which cells are
responsible ?
• goblet cells
• chief cells
• Parietal Cells
Answer: C
Q266-Resected end of small bowel and beginning of large colon will lead to deficiency in?
• Iron
• B12
• Calcium
Answer: B
Q288- Colon Cancer stage B2?
• Metastases to LN
• No LN involvment
• Metastases to Distant Organ
• Answer : B
Q203- fecal incontinence which level is affected
• above c2
• below c2
• above t12
• below t12
Answer: D
Q362- 50 yrs man diabetic, well controlled had colon cancer surgery ( coloectomy)
they kept him on insulin and dextrose, after surgery by 2 days he became
irritable, in shock, his electrolyte ( Na 129 ) ( K 3.2 ) urine and serum osmolality
Normal, what's the Dx:
• fluid overload
• addison disease
• SIADH
Answer: A
Q463-Pt with abdominal pain, nausea, vomiting, wt loss. On examination: palpable
mass. Whats your action ?
• PPI
• Follow up
• Urgent referral
• Ultrasound
Answer: C
Q464-premature presented with abdominal distention and air on x-ray ?
• Congenital Diaphragmatic hernia
• ARDS
• Necrotizing enterocolitis
Answer: C
Q471-Heamorids are
• Viens
• Arteries
• Skin Lesions
Answer: A
Q482-Baby with vomiting and abdominal distention Examination gush of stool
History of decrease feeding and constipation, history of barium enema What
next?
• Colectomy
• Hydrostatic reduction
• Leveling colostomy
Answer: C
Q84/ Prevent of hemorrhoid:
• Vit D supplement
• high fiber diets
• Daflon
Answer: C
• Cholelithiasis →imbalance of cholesterol & bile salts
• Cholesterol stones →80%
→ Obesity / age < 50 y. / Estrogens (female, OCP, Multiparity)
→Rapid weight loss
• Pigment stones → 20%
→Cirrhosis/chronic hemolysis/biliarystasis
**protective agents are statins, VitC, Coffe& Exercise
• Treatment → Urodoxycholicacid ; a cholesterol stone dissolving agent
; 50% recurrence after 5 years of discontinuation
;Prophylactic use with rapid weight loss
** Reducing fat in diet reduces billiarycolic attacks but not dissolve already formed stones
→ Pethidine; analgesia of choice to relieve billiarycolic pain
**Morphine is thought to worsen billiarycolic pain
• Investigation → Labs (CBC/ LFT’s/ Amylase & Lipase)
→ U/S (gall bladder wall thickening > 4mm / localization of stone/Murphy’s sign)
→ERCP (billiary& pancreatic ducts/ treat CBD stones/ may cause pancreatitis 2%)
→MRCP (similar to ERCP but non -invasive/ cannot treat CBD stones)
→PTC ( if ERCP failed or not available/ requires Prophylactic AB/ contraind. & compl.)
→ HIDA (tc-99/ visualize billiarytree )
• Acute Cholecystitis → hxof billiarycolic
→ RUQ or epigastricpain/Murphy’s sign/ Boas’ sign/ low grade fever
Investigation→labs (↑ ↑WBC/↑ALT/↑AST/↑ALP)
→U/S (98% sensetive) if –ve→ HIDA scan
Treatment → Cholecystectomy (Open; kocher’sincision or Laparscopic)
Q29/ 40 years old woman with no pain but you noticed jaundice. She has high direct
bilirubin and high ALT what is your diagnosis?
• Gilbert’s disease
• Biliary stone
• Carcinoma in head of pancreas
Answer: C
Q73/ During a cholecystectomy, there was an injury to the cystic artery. The surgeon
applied pressure on the free margin of the lesser omentum. What is the name of
the artery compressed in this maneuver?
• Coeliac
• Hepatic
• Splenic
• Gastro-epiploic/duodenal
Answer: B
Q108/patient post lap-cholycystectomy developed SOB, cough, respiratory distress
What's your diagnosis ?
A.PE
B.Stroke
C.Pneumonia
D. Atelectasis
Answer: D
Q116/what is the most common type of gallstone?
A.cholesterol
B.bile
C. Pigmented
Answer: A
Q118/lap-cholecystectomy injured cystic artery, surgeon put pressure with the free
margin of lesser omentum,which artery is compressed?
A.hepatic
B.coeliac
C.splenic
D.gastro-epiploic/duodenal
Answer: A
Q119/ What is the drug that can make a cholecystitis pain worse?
• Acetaminophen
• Morphine
• Meperidine
Answer: B
Q137/Case Symptoms of cholecystitis what best initial modality ?
• Radiography
• U/S
• CT
Answer: B
Q202-what is the most common type of gall stone?
A.Cholesterol
B.bile
C.Pigmented
Answer: A
Q359- 41 years old pt with recurrent episodes of acute cholecystitis U/S showed
small stones, you will give her: ?
• ezitamib
• Fibramait
• Urodoxycolic Acid
• Answer : C
Q373-most accurate test for acute cholysctitis?
A. Ultrasonography 90-95% sensitive for cholecystitis and is 78-80% specific.
B. CT & MRI The sensitivity and specificity greater than 95%
C. Hepatobiliary Scintigraphy sensitivities and specificities 90-100% and 85-95%
Answer: C
Q375- patient with right hypochondrial pain and US showed echogenic shadow ?
• Billiary stones
• Renal Stones
• Peptic Ulcer
Answer: A
Q392- What is pain killer used in cholecystitis?
• Morphine
• Pethidine
• Acetamenphine
Answer: B
Q397-Women with history 2 times of right hypochondrium pain she took analgesic for it .
Presented now to ER with acute right hypocondrium pain . CT showed Normal
gallblader with small stone not obstructing the blader. U/S showed small stone 3 cm
what is the managment?
• simvastatin
• urodoxycolic acid
• chenodoxicolic acid
Answer: B
Q409- in open cholecystectomy, what is the name of the incision used?
• Kochar ( subcostal)
• Hasson Incision
• Mcburny’s Incision
Answer: A
Q440-Sickle cell patient with recurrent RUQ pain with elevated pancreatic exam,During
U/S there was multiple gall stone (I think 7 gall stones), The largest one was2.5 cm
when you did ERCP there was no stone in billiary tree , what is your management ?
• Conservative
• Antibiotic
• Cholecytectomy
Answer: C
Q443-40 years old woman with biliary colic , what to give medically :
• statin
• urodeoxycholic acid
• Ezitembe
Answer: B
Q457-Case of female with hx of gallstones in the gallbladder she had 2. Attack which
relived her attack by analgisea,In the CT report,Low density 0.3 cm calculi in the
gallbladder What you will perscribe
• Imitadiben ( liped lowering agent )
• Simvastatin
• Urodeoxycholic
Answer: C
Q312- female pt, k/c of sickle cell, experiencing acute cholecystitis attacks from time
to time, on US there is 7 gall bladder stones, one of them is 2 cm large, now she is
asymptomatic, what is your plan?
• cholecystectomy.
• Urodoxycholic Acid
• Statins
Answer: A
Q488-Senario,best site to insert the needle for liver biopsy:
• Midaxillary at 6th intercostal space
• Midaxillary at 7th intercostal space
• Midaxillary at 10th
Answer: B
• Acute Pancreatitis → Idiopathic/ gall stones 45%/ Alcohol 35%/Tumors/scorpion stings
→infection/ Autoimmune/ Surgery (e.g. ERCP) / hyperlipidemia/drugs
→ Activation of proteolyticenzymes → ↑↑↑Amylase & lipase
Treatment →Supportive (hydration / analgesia →90% resolve)
→ ERCP + sphincterotomyif conservative failed
Complication → Pseudocyst/ abcess/infection/necrosis/related vessel thrombosis/
ascites/ pleural effusion/ DM/ ARDS/ DIC/encephalopathy/ hypocalcemia
• Chronic Pancreatitis → necrosis + inflammation + fibrosis
→Alcohol 90%
early stage→attacks of epigastricpain radiating to back
late stage → Malabsorption$ If >90% of function are lost (Steatorrhea)
→DM/Calcification/Jaundice/Wt. loss/ Pseudocyst/Ascites/GI bleeding
Treatment → Alcohol Abstinence + enzyme Replacement + Analgesics
→Pancreatic resection if duct blockade
• HereditryPancreatitis → Chronic or Recurrent Acute (AutosomalDominant)
• Pancreatic Cancer →Increased age 50 -70 y.o.
→Smoking↑riskby 5x
→↑fat↓fiberdiet/ heavy Alcohol use/ Obesity/DM
→head 70% ; painless jaundice →Courvoisier ‘s sign
→body or tail 30% ; continousmid epigastricpain + wt loss
Investigation → CA19.9 (most useful marker)
→↑ALP, ↑Billirubin
→U/S & CT w/ contrast (assess metastasis & resectability)
→ERCP/MRI/MRCP
Treatment →Surgical Resection (only 20-30% are resectable)
→whipple’sresection (pancreaticoduednoectomy)
Q49/ Patient with abdominal pain and fever. Lab showed high amylase. What is the
diagnosis?
• A) Acute pancreatitis
• B)chronic Pancreatitis
• C)Acute cholecystitis
Answer: A
Q167/ Patient came with history of upper abdominal pain the investigation show
high amylase your diagnosis will be:
• A)acute pancreatitis.
• B)gastric ulcer.
• C)acute cholecystitis.
Answer: A
Q258- hereditary chronic pancreatitis is?
• A) autosomal dominant
• B)Autosomal recessive
• C) polygene
Answer: A
Q310- Pt reported that he was diagnosed with pancreatic CA, his medical report was free
and when he confronted by reports , he ranaway. What is your diagnosis?
• Hypochondriasis
• malingering
• Post traumatic stress
Answer: B
Q404- pt male 57 with chronic epigastric abd pain , last wk lost 7 kg +ve stool occult
blood
• Chronic pancreatitis
• Chronic cholecystitis
• Chronic gastritis
Answer: C
Q469-patient with +ve Cullen sign and Grey-Turner sign what is the diagnosis ?
• Acute cholycystitis
• Acute Pancreatitis
• Chronic Pancreatitis
Answer: B
Q521-can't recall the senior very well but it was symptoms and sign of pancreatic disease
that caused thrombophlebitis ?
• acute pancreatitis
• pancreatic tumor
Answer: A
• Hernia → Abdominal wall defect w/ protrusion of intra abdominal content
→m: f , 9: 1
→Ass. w/ ↑Intra Abdominal Pressure (Obesity/cough/pregnancy/constipation)
Types →Groin; Direct →25%
→Old age/ hasselbach’s∆/ medial to inferior epigastrica.
;Indirect→50%
→young age/lateral to Inferior epigastrica.
→Originate at deep inguinal ring descend to scrotumor labia majora
;femoral→5%
→ Pregnancy/ Femoral canal/ below Inguinal Ligament/ medial to
femoral V.
→most common strangulated hernia
→Ventral; Incisional→8-10%
;Umbillical→3-8%, at birth ,observed untill5 years of age
** Laboratory studies →not specific for hernia.
**Imaging studies → not required in the normal workup of a hernia; However, they may be useful
in certain scenarios, as follows:
• U/S→ differentiating masses in the groin or abdominal wall
→differentiating testicular sources of swelling
• Upright CXR/AXR→If an incarcerated or strangulated hernia is suspected
• U/S ± CT (CT required for obtrator/femoral/SpigelianHernia or obese patient)
Q15/A patient with indirect inguinal hernia. What artery runs medial to it?
• A)inferior epigastric artery
• B)Femoral Artery
• C)Testicular Artery
Answer: A
Q17/Patient came with abdominal swelling and weak abdominal wall. What’s the
diagnosis?
• A) Umbillical
• B)direct inguinal hernia
• C)indirect Inguinal Hernia
Answer: A
Q76/ 7 years old boy brought by his parents, he has nausea, severe vomiting for 20
minutes and now semi comatose. The parents mentioned that he has same
episode two weeks ago for 5 minutes without deterioration in consciousness. On
examination there is right testicular mass that does not transilluminate with light.
What is the best action to do ?
• a. Radiology.
• b. ESR.
• c. Surgical exploration.
Answer: C
Q104/during laparoscopic surgery of inguinal hernia you find artery superficial going
upward ?
• A.Anterior Abdominal Artery
• B.Inferior epigastric artery.
• C.Suprior Epigastric Artery
Answer: B
Q121/Ligament in inguinal canal?
• A.Broad
• B.Round
• C. Inguinal
Answer: B
Q173/ 18-year-old healthy male was playing baseball and suddenly he felt abdominal
pain. On examination he has para-umbilical mass. His vital signs are: BP 100/76, RR
30, HR 100, O2 sat is 95% with 100% oxygen mask. What is your next step in the
management?
• A)Abdominal US.
• B)CT scan.
• C)Erect Chest x-ray.
• D)Reassure and send home.
Answer: C
Q214- infant periumblical hernia , what you will do:
• Truss
• reassurance
• Hernioplasty
Answer: B
Q306- Which of the following types has the highest risk for developing strangulated
hernia?
• A) Direct inguinal
• B)Indirect inguinal
• C)Femoral
Answer: C
Q319-mid inguinal point artery posterior to it ?
• A)external iliac
• B)Femoral Artery
• C)epigastric
Answer: A
Q456-Older patient lifting heavy object with mass not reaching the scrotum?
• A)Direct inguinal hernia
• B)Indirect inguinal hernia
• C) Femoral Hernia
Answer: A
Q461-Pt undergone laparoscopic hernioplasty then complains of respiratory
depression.
• A)Atelectasis
• B)Pulmonary Embolism
• C) DVT
Answer: A
Q548-18 boy play basketball he came with abdominal pain without any injury in
match physical exam was tenderness in paraumbilical area what to do next:
• A. Chest xray
• B. abdominal CT
• C. 24 hr recheck
• D. kidney US
Answer: A
Q370- baby with decreased air in the lung, abdomen looks scaphoid, what is the
diagnosis?
• dextrocardia.
• Pyloric stenosis
• diaphragmatic hernia.
Answer: C
• Abdominal Trauma → Blunt; solid organ injury→ Spleen most common
→liver 2nd most common
→Intra-abdominal or
retroperitonealhemorrhage
→ CT scan most specific
→ Peritoneal Lavage most sensitive
→U/S FAST detects fluid in peritoneal cavity
**imaging must be done →equivocal abdomen/altered sensorium(head & spinal injury)/ shock or
hypotension/fracture ribs, pelvis & spine/postiveFAST.
**if hemodynamic ally unstable or persistent high transfusion demands→Laparotomy
→Penetrating; hollow organ injury → Liver most common
→Stab; requires local wound
exploration
→gunshot; always requiring lapartomy
**penetrating wound + shock or peritonitis or evisceration or free air or blood in NG →lapartomy
Q53/ A patient presented with splenic injury after abdominal trauma. The surgeon
decided to embolize the splenic artery. Which of the following will be
compromised?
• Lower end of esophagus
• Stomach fundus
• Splenic flexure
Answer: B
Q140/anterior abdominal stab wound , the omentum is bulging out through wound ?
• A.FAST
• B.CT
• C.Exploratory laparotomy
Answer: C
Q245- A patient was stabbed in his abdomen, o/e he’s vitally stable, and some of the
mesentery is out. what will u do
• exploratory laparotomy
• wound exploration
• Observe
Answr: A
Q295-gunshot with wound bowel perforation , What antibiotics you should give?
Answer: Metronidazole + Gentamycin
Q353- First degree spleen injury, ttt?
• Immediate laparatome.
• conservative in ward.
• conservative in ICU
• Laparoscopy
Answer: B
Q371-Stap wond in the abdomen with bacteroid fragilis what is the best antibiotic to
use?
• Ampicillin
• Metronidazole
• Carbapenems
Answer: B
Q374-Patient with stab wound what to do next ?
• Local wound exploration
• exploratory laparotomy
• FAST
Answer: A
Q406- in Splenectomy what organ might be affected
• Pancreas
• Stomach
• Colon
Answer: A
Q422-Patient had splenic trauma and splenectomy planned, if the surgeon ligate the
splenic artery high up what structure would be affected ?
Lower Esophageal Sphincter
Stomach fundus
Pancreas
Answer: B
Q446-which of these patient perotineal lavage is indicated ?
• severe head trauma
• hypotensive pt with abdominal distention
• Mandatory for all Abdominal Trauma cases
Answer: B
Q479-case of truma pt unconscious with abdomen distension what is the intial
treatment:
• CT
• IV fluid
• FAST
Answer: B
Q543-CT in truma important for :
• to detect diaphragmatic injury
• Intraperitoneal Hemorrage
• retroperitoneal injury
Answer: C
Q82/ Patient presents with blunt trauma that affect the inferior mesenteric artery, Which
one of the following branches NOT affected?
• - Left colic artery
• - Sigmoidal arteries
• - Superior rectal artery
• - Cecal Artery
Answer: D
Q176/ Patient after RTA had spleen injury, he underwent surgical exploration and
splenectomy. Which of the following should be given?
• Antibiotic.
• Meningiococcal vaccine.
• Antiviral
Answer: B
Q186-RTA, splenic injury type 1, what the management?
• Laparotomy
• Diagnostic Laparoscopy with splenectomy
• Conservative
Answer: C
Q230- intra abdominal bleeding can lead to:
• Hypovolemia
• Dehydration
• Peritonitis
Answer: A
Q255-CT in trauma patient is done to detect what?
• Retroperitoneal bleeding
• Intraperitoneal bleeding
• It is most specific for trauma
Answer: A
Q335-we do ct in blunt abd trauma why?
• Retroperitoneal bleeding
• Intraperitoneal bleeding
• It is most specific for trauma
Answer: A

You might also like